Dr.iqbal Orl Cases

November 4, 2017 | Author: Fahad Raja | Category: Ear, Human Head And Neck, Health Sciences, Wellness, Diseases And Disorders
Share Embed Donate


Short Description

ent made easy...

Description

Clinical Scenarios in

Oto-rhino-laryngology A Problem Oriented Approach

First Edition ISBN: 978-969-9340-01-7

PROF. DR. IQBAL HUSSAIN UDAIPURWALA MBBS., DLO., FCPS. Professor and Head of the ENT Department Bahria University Medical & Dental College, Karachi. Fellow and Examiner, College of Physicians & Surgeons Pakistan. Editor, Pakistan Journal of Otolaryngology and Head & Neck Surgery. Visiting ENT Surgeon, Liaquat National Hospital, Habib Medical Centre and Zubaida Medical Centre, Karachi.

Clinical scenarios in oto-rhino-laryngology

Great care has been taken to maintain information contained in the volume. However, in view of the ongoing research and changes in the government rules and regulation and a constant inflow of information, the author can not be held responsible for errors or for any consequences arising from the use of the information contained herein.

Copyright Reserved All rights are reserved with the publisher. No part of this publication may be reproduced, stored in a retrieval system, or transmitted, in any form or by any means, electronic, mechanical, photocopying, recording or otherwise, without the prior permission of the publisher.

ISBN: 978-969-9340-01-7

First Edition, 2011

II

Clinical scenarios in oto-rhino-laryngology

Preface to the first edition It is a moment of great pleasure for me to present first edition of this book. Clinical subjects are always difficult for the medical students because most of the text books are written according to the diseases or systems but when they deal with the patients, who always come with some complaints. To correlate these complaints of the patients with the disease and to make diagnosis is always difficult for them. This book is an endeavor to improve clinical acumen and interpretation of a medical student. All common diseases of oto-rhino-laryngology and head & neck are presented in the form of case discussions. A clinical scenario is presented first and then important points in history taking and clinical examination are described with clinical, provisional or differential diagnosis. How this patient will be investigated and how the diagnosis will be reached is then given. At the end important points related with the diseases is discussed briefly. This book could not have been accomplished without the help and assistance of many people. I would like to express my sincerest gratitude to all my teachers and colleagues who gave me valuable guidance and suggestions in writing this textbook. My special thanks goes to Dr. Muhammad Shuja Farrukh, assistant professor of ENT, Dow University of Health Sciences, Karachi who gave me his utmost support and full time assistance in proof reading of the manuscript several times, as indeed in seeing it in the form of a print. I am thankful to Mr. Rehan Ahmed Khan and Mr. Rehan Zia of Hamdard University Hospital for their technical services and support in title designing and computer work. In the last but not least, I am grateful to my wife Azra Iqbal and my daughters, Fatima, Saba and Zahra for their enormous support and untiring efforts at every step of this work, without them it would not be possible to make this idea into a reality. I hope medical students will find it very useful in preparation of their final examination in ENT as well as in their clinical practice. By no means it is perfect and there may be some ambiguity in the text. Your suggestions and criticism are always welcome to improve the standard of this book.

Karachi, 2011.

IQBAL HUSSAIN UDAIPURWALA

III

Clinical scenarios in oto-rhino-laryngology

Foreward What is special about this book written by Prof. Iqbal Hussain Udaipurwala, who is already author of few books on oto-rhino-laryngology. Real time clinical scenario based book is a smart scientific attempt for teaching and learning, which makes it interesting and exciting. Most traditional textbooks of oto-rhino-laryngology contain basic clinical and scientific facts that forms the foundation of the speciality. While these text can provide an essential cornerstone for the practice of oto-rhino-laryngology, applying this information to a clinical setting relies on sound judgement, presence of mind and clinical experience. Problem based learning for under-graduate and post-graduate teaching is very rewarding and practical. Based on real time clinical cases, which you can witness by actual clinical case, patient pathology and photographs, you get a confidence of trueness. This helps confidence building regarding diagnosis and management. I must commend the author, Prof. Iqbal Hussain Udaipurwala for contributing this scientific book for learners, this is bound to enrich the reader’s mind and skill beyond his expectations.

PROF. SYED TIPU SULTAN MBBS, DA (London), FFARCSI, FCPS (Hon.) Professor of Anaesthesiology, critical care and pain management, Principal, Bahria University Medical & Dental College, Council Member, College of Physicians & Surgeons, Pakistan, President, Pakistan Medical Association (centre).

IV

Clinical scenarios in oto-rhino-laryngology

CONTENTS SECTION I: EAR Case 1- Boil in the ear Case 2- Foreign body of the ear Case 3- Wax in the ear Case 4- Pre-auricular sinus Case 5- Otomycosis Case 6- Maggots in the external auditory canal Case 7- Acute suppurative otitis media Case 8- CSOM with aural polyp Case 9- CSOM with cholesteatoma Case 10- CSOM with facial paralysis Case 11- Dry perforation of ear drum Case 12- Otitis media with effusion Case 13- Otosclerosis Case 14- Presbyacusis Case 15- Noise induced hearing loss Case 16- Meneire’s disease Case 17- Benign paroxysmal positional vertigo Case 18- Congenital deafness

3 6 9 12 14 16 18 21 24 28 31 34 38 42 45 48 51 54

SECTION II: NOSE & PARANASAL SINUSES Case 19- Deviated nasal septum Case 20- Nasal trauma with fracture Case 21- Antro-choanal polyp Case 22- Septal haematoma Case 23- Ethmoidal polypi Case 24- Septal adhesion Case 25- Allergic rhinitis Case 26- Foreign body of the nose Case 27- Boil in the nose

59 62 66 70 73 76 78 81 84

V

Clinical scenarios in oto-rhino-laryngology

Case 28- Epistaxis Case 29- Chronic rhino-sinusitis Case 30- Nasopharyngeal angiofibroma

86 89 92

SECTION III: ORAL CAVITY & PHARYNX Case 31- Chronic tonsillitis Case 32- Post-tonsillectomy haemorrhage Case 33- Quinsy Case 34- Enlarged adenoids Case 35- Papilloma of the cheek Case 36- Carcinoma of the cheek Case 37- Carcinoma of the tongue Case 38- Foreign body of the throat Case 39- Ranula

99 102 104 107 110 113 117 120 123

SECTION IV: LARYNX & TRACHEA Case 40- Vocal nodules Case 41- Tracheostomy Case 42- Carcinoma of the larynx Case 43- Foreign body of the bronchus

127 130 133 138

SECTION V: HEAD & NECK Case 44- Ludwig’s angina Case 45- Branchial cyst Case 46- Retro-pharyngeal abscess Case 47- Sub-mandibular salivary calculus Case 48- Parotid gland pleomorphic adenoma Case 49- Multinodular goiter (MNG) Case 50- Tuberculous cervical lymphadenopathy

143 146 149 152 155 158 162

VI

Clinical scenarios in oto-rhino-laryngology

Section I EAR

Case 1Case 2Case 3Case 4Case 5Case 6Case 7Case 8Case 9Case 10Case 11Case 12Case 13Case 14Case 15Case 16Case 17Case 18-

Boil in the ear Foreign body of the ear Wax in the ear Pre-auricular sinus Otomycosis Maggots in the external auditory canal Acute suppurative otitis media CSOM with aural polyp CSOM with cholesteatoma CSOM with facial paralysis Dry perforation of ear drum Otitis media with effusion Otosclerosis Presbyacusis Noise induced hearing loss Meneire’s disease Benign paroxysmal positional vertigo Congenital deafness

1

3 6 9 12 14 16 18 21 24 28 31 34 38 42 45 48 51 54

Clinical scenarios in oto-rhino-laryngology

Case 1 Clinical Scenario A 28 years male patient came in the OPD with complaint of pain in the left ear for last 2 days. Initially pain was mild to moderate but next day it increased and became severe. He also had deafness and swelling around the ear canal with some pussy discharge. On examination his left pinna was very tender especially over the tragus with yellowish pus coming out. On retraction of the pinna a rounded, soft and fluctuant swelling was visible at the external auditory meatus (fig. 1.1). It was not possible to examine deeper part of the canal and tympanic membrane because of pain.

Fig. 1.1 Rounded, soft, tender and fluctuant swelling in the external auditory meatus

Important points in history taking: 1- Any previous history of discharge from the ear. In this case there was no such previous history. 2- History of diabetes mellitus or other immuno-compromised states. In this case no such history. 3- Habit of scratching the ears with any sharp object. There was no such history. 4- Any history of swimming especially in dirty water. There was no such history. Important points in clinical examination: 1- Palpation of tragus, pinna and mastoid area for tenderness. Tragus and pinna were tender but mastoid area was not tender in this case. 2- Probe test of the swelling. Swelling was very tender and soft. It was not possible to move the probe all around the swelling i.e swelling is arising from the canal wall in its outer cartilagenous part. 3- Tuning fork tests showed conductive type of deafness. Rinne’s test was negative in the left ear and positive in the right ear, Weber’s test was 3

Clinical scenarios in oto-rhino-laryngology

lateralized towards the left and and Schwabach’s test was equal to the examiner in both ears. Diagnosis: The most probable diagnosis of this case was ‘Boil ear’. Differential Diagnosis: The differential diagnosis in this case could be: 1- Aural polyp: In aural polyp there is history of chronic discharging ear for a long time. In addition probe test will differentiate a polyp from swelling arising from the external auditory canal. 2- Osteoma: This is a benign tumour of bony origin and is situated in the deeper bony part of the external auditory canal. It is hard and usually non-tender. Investigations: No investigation was done in this case. Treatment: This was a case of large boil where pus was present along with fluctuation and patient was already taking antibiotic without any relieve. So incision and drainage was planned under local anaesthesia. A longitudinal incision was given parallel to the external auditory canal. Pus was drained and sent for culture and sensitivity. The external auditory canal was packed with antiseptic ointment. Antibiotic against staphylococcus aureus was given parenterally (amoxicillin with clavulanic acid). Pus drained after incision & drainage was sent for culture and sensitivity, which showed heavy growth of staphylococcus aureus. The organisms were found to be sensitive to amoxicillin with clavulanic acid, so the same antibiotic was continued for 7 days. Discussion: Boil or furuncle is the acute infection of the hair follicle by staphylococci. In the external auditory canal hair follicles are only present in the outer one-third part. Boil in the ear is usually single but multiple boils can occur sometimes. The predisposing factors are diabetes mellitus, general debilitating diseases, scratching of the external auditory canal, swimming and poor hygiene. Following points are important to remember in case of boil in the ear: 1- Recurrent boil is common in patients having diabetes mellitus. Thus if any patient comes with recurrent boil, diabetes mellitus should be excluded. 2- During incision and drainage of the boil ear, incision is always given parallel to external auditory canal. The circumferential incision at the external auditory meatus may lead to stenosis later on, so it should be avoided. 4

Clinical scenarios in oto-rhino-laryngology

3- Boil in the ear is a very painful condition because the skin is tightly adherent to the underlying cartilage.

TEST YOURSELF

Read the clinical scenario given at the beginning and answers the following questions 12345-

What is the most likely diagnosis in this case? What are the differential diagnosis in this case? How will you investigate this case? How will you manage this case? What are the important points to remember in a case of boil in the ear?

5

Clinical scenarios in oto-rhino-laryngology

Case 2 Clinical Scenario A mother brought her 4 years old son with the complaint that he had inserted something in his right ear 3 hours back. She tried to remove it with a forceps, which resulted in further pushing of the foreign body deeper in the canal. He was also complaining of mild pain in his right ear. Otoscopy showed foreign body (a bead) impacted deeper in his external auditory canal (fig. 2.1)

Fig. 2.1 Otoscopic finding showing a foreign body in the external auditory canal

Important points in history taking: 1- Inquire about nature of the foreign body, whether it was vegetative or non-vegetative, metallic or non-metallic, smooth or sharp, rounded or irregular etc. In this case nature of the foreign body was not known. 2- Duration of foreign body insertion. In this case it was three hours. 3- Any attempt of removal by family member or family doctor. Unskilled attempt for removal may cause further pushing of foreign body deeper and trauma to the surrounding structures. In this case there was history of removal of the foreign body by patient’s mother herself. 4- Any bleeding from the ear. 5- Pain in the ear. Pain is caused by pressure of the foreign body or trauma to external auditory canal or ear drum. Important points in clinical examination: 1- Assess the site of impact. 2- Confirm the nature of foreign body. 3- Any trauma or bleeding present in the external auditory canal. 4- General condition of the child, whether he is anxious or co-operative. Removal of foreign body in an anxious and unco-operative child may 6

Clinical scenarios in oto-rhino-laryngology

cause more trauma. So it is always better to remove foreign body under general anaesthesia. Diagnosis: This was a case of impacted foreign body in right external auditory canal. Investigations: No investigation is required in otherwise healthy child in such an emergency situation. Treatment: Patient was admitted for removal of foreign body under general anaesthesia, as he was crying and very anxious, even not allowing proper examination. Under general anaesthesia foreign body was removed by passing a ring probe beyond the foreign body and pulling it out (fig. 2.2).

Fig. 2.2 Method of removal of a rounded foreign body by ring probe

Discussion: Foreign bodies in the ear may be animate such as insects or inanimate. Inanimate foreign bodies are usually introduced by children and mentally retarded persons by themselves. Inanimate foreign body may be hygroscopic or vegetative like seeds or non-hygroscopic or non-vegetative like metals, plastic and other materials. A hygroscopic foreign body absorbs water and moisture present in the canal and swells up and gets impacted in the canal. Isthmus of the external auditory canal is the narrowest part and most of the foreign bodies are impacted at isthmus. Method of removal depends upon the size, site of impaction and type of foreign body. Removal under general anaesthesia is essential in children and sensitive adults. Smooth and rounded foreign body is removed by a ring probe. Forceps must not be used in such foreign bodies as it can push the foreign body further in. 7

Clinical scenarios in oto-rhino-laryngology

TEST YOURSELF

Read the clinical scenario given at the beginning and answers the following questions 1234-

How will you manage this patient? What are the options for removal of foreign body from the external auditory canal? Classify foreign bodies of the external auditory canal. What is the narrowest part of external auditory canal?

8

Clinical scenarios in oto-rhino-laryngology

Case 3 Clinical Scenario A 27 years old male patient came with the complaints of blockage of the right ear after swimming in the pool on a picnic 2 days back, which was continous and same. He also had mild pain and discomfort in his right ear. Important points in history taking: 1- Any discharge from the ear. No discharge was present in this case. 2- History of cold or sore throat before going to swimming. There was no such history. 3- Itching in the ear. Slight itching was present in this case. 4- Use of ear plugs during swimming. He had not used ear plugs during swimming. 5- Any history of bleeding from the ear. There was no history of bleeding from the ear.

Fig. 3.1 Otoscopic finding of the right ear

Important points in clinical examination: 1- Ear examination with the speculum and otoscope. Showing accumulation of dark brown wax in the external auditory canal (fig 3.1). 2- Suction cleaning of the ear and inspection of the tympanic membrane. Wax was not possible to be removed by suction as it was impacted and hard. 3- Tenderness over pinna, tragus and mastoid area. There was no tenderness on any area in this case. 4- Tuning fork tests. Rinne’s test was negative in right and positive in left ear, Weber’s test was lateralized to the right and Schwabach’s test was equal to the examiner on both the sides. 9

Clinical scenarios in oto-rhino-laryngology

5- Examination of the nose and throat for any pathology. No positive finding was present in these areas. Diagnosis: The diagnosis in this case was impacted wax in the right ear. Differential Diagnosis: 1- Otomycosis, where wet newspaper like mass is seen in the canal. 2- Traumatic perforation of the ear drum. 3- Otitis externa or boil. It is very painful and tenderness is present over tragus and pinna. Investigations: No laboratory investigation was done in this case as the diagnosis was clear. In case of suspected otomycosis, removed debris from the ear should be sent for fungal smear. Treatment: Wax was first softened by instilling a softening agent like 2% soda glycerine ear drops thrice daily for 2 days and suction cleaning was done later on (fig 3.2). After suction cleaning of the ear, tympanic membrane and external auditory canal were found to be normal and patient had normal hearing.

Fig. 3.2 Suction cleaning of the wax

Discussion: Wax or cerumen is the mixture of the secretions of ceruminous and pilo-sebaceous glands. These glands are present only in the cartilagenous portion of external auditory canal.The proportions in the mixture determine the consistency of the wax. When secreted it is thick and golden brown in colour, which becomes darker and hard on drying. Normally it is expelled from the canal in flakes, aided by the movement of jaw. Plug formation is encouraged by excessive formation of wax and its retention by stiff hairs, 10

Clinical scenarios in oto-rhino-laryngology

exostosis, desquamation and other stenosing conditions. The options for the removal of soft wax are: 1- Syringing (fig. 3.3) 2- Suction cleaning (fig. 3.2)

Fig. 3.3 Syringing of the ear in another patient

TEST YOURSELF Read the clinical scenario given at the beginning and answers the following questions 12345-

What are the differential diagnosis in this case? How will you manage this case? What is wax and how it is formed? What are the signs and symptoms of a patient having impacted wax in the ear? What are the different options for removal of impacted wax in the ear?

11

Clinical scenarios in oto-rhino-laryngology

Case 4 Clinical Scenario A mother brought her 10 years old daughter with the complaint that she had a small hole in front of her ear on both the sides since birth (fig 4.1). There was history of repeated discharge often white or yellow in colour from these openings, which settled down with medication from local general practitioner. Discharge was often associated with pain and redness in front of the ears.

Fig. 4.1 Left ear showing a small hole or opening with redness and swelling in front of the crus helix

Important points in history taking: 1- Whether it was present at birth. In this case it was present at birth. 2- Any history of discharge from the opening or redness and pain of the surrounding area. In this case there was history of occassional discharge with redness and pain around the opening for which she takes medicines. 3- Unilateral or bilateral. In this case it was bilateral. Important points in clinical examination: 1- Assess whether it is infected or not. At time of presentation there was no sign of infection except skin was slightly red and congested. 2- Any other opening in external auditory canal. There was no other opening. 3- Assess for any other congenital abnormality of the ear. All other areas were within normal limits. Diagnosis: This was a case of ‘Pre-auricular sinus’. Investigations: 1- Pus or discharge for C/S. This patient when presented has a dry opening so C/S was not done. 12

Clinical scenarios in oto-rhino-laryngology

2- Other baseline investigations for general anaesthesia when planned for surgery like blood complete picture, ESR, random blood sugar, urine D/R and X-ray chest (PA view). 3- Sinogram: It is done to delineate the whole sinus and its tract. In routine cases it is not indicated, so it was not done in this case. Treatment: Surgery was planned after investigation. Under general anaesthesia an elliptical incision was given (fig. 4.2). Whole tract and opening of the sinus was excised and wound was closed in two layers.

Fig. 4.2 Elliptical incision was marked before surgery

Discussion: Pre-auricular sinus is a congenital condition and is due to failure of complete fusion between the first and second branchial arch elements in the auricle. External opening of pre-auricular sinus is situated between the tragus and crus helix.

TEST YOURSELF

Read the clinical scenario given at the beginning and answers the following questions 1234-

What is your diagnosis in this case? How will you manage this case? What is a pre-auricular sinus and how is it formed? What is the location of external opening of pre-auricular sinus?

13

Clinical scenarios in oto-rhino-laryngology

Case 5 Clinical Scenario A 24 years old male patient came with the complaints of severe itching and discharge from his right ear for last 10 to 12 days. On clinical examination, otoscopic findings are shown in fig. 5.1.

Fig. 5.1 Otoscopic findings of the right ear

Important points in history taking: 1- Detailed history regarding itching and discharge. Itching was severe and continuous present all the time. Discharge was scanty, creamish to yellow in colour, thick and often contained blackish spots. It was never blood stained. 2- Previous history of discharge. There was no history of discharge in the past. 3- History of swimming. He went to a picnic spot where he did swimming in a small swimming pool about three weeks back. He had not used ear plugs during swimming. 4- Any history of pain. There was no pain in the ear. 5- Any history of deafness or blockage of the ear. According to him his right ear was blocked since the start of these symptoms. 6- Any history of diabetes mellitus or any immuno-compromised state. There was no such history. 7- Habit of scratching the ears with different objects. There was no such history in this case. Important points in clinical examination: 1- Examination of the external auditory canal and tympanic membrane. External auditory canal was full of creamish yellow debris with brownish black spot (fig. 5.1). Tympanic membrane was not visible. 14

Clinical scenarios in oto-rhino-laryngology

2- Voice test. Mild deafness was present in the right ear. 3- Tuning fork tests. Rinne’s test was negative in the right ear and positive in the left ear. Weber’s test was lateralized towards the right side and Schwabach’s test was equal to the examiner on both sides. 4- Suction cleaning of the debris done and examination of the external auditory canal and tympanic membrane was done, which were both oedematous and congested. Differential Diagnosis: 1- Otomycosis 2- Impacted wax in the ear 3- Other types of otitis externa Investigations: 1- Debris removed from the external auditory canal was sent for fungal smear, which showed presence of fungal hyphae, confirming the diagnosis. Diagnosis: This was a case of otomycosis or fungal infection of the external auditory canal. Treatment: Suction cleaning of the external auditory canal was done completely. Clotrimazole lotion (anti-fungal drops) was given for topical application in the right ear, three times a day. Patient was advised for dry mopping of the external auditory canal before instilling lotion in the ear. Regular follow-up was done and once again suction cleaning was done after 4 days. Patient’s ear became normal and dry within ten days. Discussion: Otomycosis is the fungal infection of the lining skin of external auditory canal. Swimming in the dirty water or continuous discharge due to otitis media, are important predisposing factors. Aspergillus is the most common type of fungus causing this condition but in some cases candida albicans is the causative organism. Secondary bacterial infection is very common which causes pain. On examination the external auditory canal is filled with a wet news paper or blotting paper like mass and its colour depends upon the type of fungus.

TEST YOURSELF

Read the clinical scenario given at the beginning and answers the following questions 1- What is your diagnosis in this case? 2- How will you manage this patient? 3- What are the different varieties of fungus causing otomycosis? 15

Clinical scenarios in oto-rhino-laryngology

Case 6 Clinical Scenario A father brought his 2 years old son with the complaints of severe pain, discharge and bleeding from the right ear for last 2 to 3 days (fig. 6.1). He took medicines from his family physician but there was no relief and the condition was worsening.

Fig. 6.1 Patient with severe pain, discharge and bleeding from the right ear

Important points in history taking: 1- Detailed history regarding his symptoms. According to his father, patient was completely alright 3 days back, when he complained of pain in his right ear. Next day he also had some discharge from his right ear along with pain. He took some medicines from his family physician and pain subsided. Next day patient again had severe pain and discharge along with blood. 2- Previous history of discharge. There was no history of discharge in the past. 3- Any history of trauma, scratching or foreign body insertion. There was no such history. 4- Any history of cold, fever, sore throat or respiratory tract infection. There was no such history. 5- Socio-economic and hygienic condition. They lived in a village with very poor hygienic condition with lots of house flies and mosquitos. Important points in clinical examination: 1- Examination of the right ear. There was discharge and blood coming out from the right external auditory canal (fig. 6.1). Patient had severe tenderness over the pinna and surrrounding area. All discharge and blood cleaned from the external ear and external auditory canal. There were many maggots present in the external auditory canal (fig. 6.2). 16

Clinical scenarios in oto-rhino-laryngology

Fig. 6.2 Maggots removed from the patient’s right ear

Diagnosis: This was a case of ‘Maggots in the ear’. Treatment: Patient was admitted in the hospital. Few drops of ‘maggot oil’ were instilled in the right ear and maggots removed. All debris and discharge also cleaned from the external auditory canal and pack soaked in antiseptic ointment was applied. Broad spectrum parenteral antibiotic was started along with analgesic. Daily dressing and cleaning of the external auditory canal was done. Subsequent recovery was uneventful. Discussion: Maggots are the larvae of housefly. These flies are attracted by the foul smelling discharge present in the ears or nose and lay their eggs into the external auditory canal or nasal cavity. Within 24 hours these eggs hatched out into larvae or maggots. Maggots produces severe pain, irritation, swelling, foul smelling and blood stained discharge. On examination maggots are visible crawling in the external auditory canal. They may cause extensive soft tissue necrosis. Treatment consist of removal of all the maggots with forceps but they are usually firmly attached to the meatal wall. Maggot oil (turpentine oil) or chloroform water is instilled which causes asphyxia and killing of maggots thus facilitating their removal.

TEST YOURSELF

Read the clinical scenario given at the beginning and answers the following questions 1- How will you manage this patient? 2- What are maggots? 3- Outline clinical features of maggots in the ear.

17

Clinical scenarios in oto-rhino-laryngology

Case 7 Clinical Scenario A mother brought her 7 years old son with the complaint that his son was not looking well since yesterday, when he went to bed early. At midnight he woke up with the complaint that he has severe pain in his right ear. She gave him syrup paracetamol and put some eardrops in his right ear, after that he slept again. Next morning when he woke up he was again complaining of severe pain in his right ear. Important points in history taking: 1- Detailed history regarding pain like character, nature, site, severity, radiation, aggravating and relieving factors etc. 2- Any history of fever. In this case there was history of fever since yesterday. 3- Any history of present or previous discharge from the ear. There was no such history in this case. 4- Any history of deafness or hearing impairment. On inquiry child mentioned about the blockage or hearing impairment in his right ear. 5- Any history of sore throat, cold, nasal obstruction, nasal discharge, postnasal dripping etc. In this case there was history of common cold for last two days. 6- Any history of scratching of the external auditory canal, foreign body insertion, swimming or entry of water during bathing. In this case there was no such history. Important points in clinical examination: 1- Inspection of the pinna, external auditory canal, mastoid area, pre-auricular region along with palpation for tenderness in these areas. All were within normal limits. 2- Examination of tympanic membrane with speculum and otoscope. In this case tympanic membrane was congested and bulging, more prominently

Fig. 7.1 Otoscopic picture of tympanic membrane on right side showing congested and bulging tympanic membrane

18

Clinical scenarios in oto-rhino-laryngology

in its posterior half (fig. 7.1). 3- Tuning fork tests. It was not done in this case because of severe pain and anxiety. 4- Examination of the nose and throat. In this case both nasal cavity and throat were congested with secretions in both nasal cavities. Diagnosis: This was a typical case of acute suppurative otitis media. Pain usually starts in the night during sleeping, when the ear is in dependent position along with venous stasis and reduced eustachian tube opening. Differential Diagnosis: In a child of 7 years following conditions have to be differentiated with acute suppurative otitis media: 1- Diffuse otitis externa and boil in the ear. These two conditions may present with acute pain but the pain is not deep seated as in case of acute suppurative otitis media. In addition there will be tenderness on the tragus and pinna with oedema or swelling in the external auditory canal. Tympanic membrane will be normal with no hearing loss. 2- Referred earache. In such cases tympanic membrane and external auditory canal are normal with no deafness or discharge. Look the other areas for referred earache like oral cavity, tonsils, pharynx, teeth, nose and neck. Investigations: No investigation was done in this case as the diagnosis was clear. Pus is sent for culture and sensitivity in cases of tympanic membrane perforation with discharge or in cases where myringotomy is done for evacuation of pus. Treatment: The patient was planned for medical treatment first. Following treatment was given and the patient was followed up for improvement. 1- Antibiotic (amoxicillin with clavulanic acid) in suspension form was given according to body weight of the patient. 2- Syrup ibuprofen with psuedoephedrine. 3- Steam inhalation, twice daily for 10 minutes each. Patient responded well on the above mentioned treatment and there was no need for myringotomy. Discussion: Acute suppurative otitis media is the acute inflammation of the lining mucous membrane of the middle ear cleft. Clinically it is divided into the following four stages: 1- Stage of tubo-tympanitis 2- Stage of catarrhal inflammation 3- Stage of suppuration 19

Clinical scenarios in oto-rhino-laryngology

4- Stage of resolution or complication This patient presented in the stage of suppuration where frank pus was present in the middle ear with bulging of the tympanic membrane. Myringotomy is often needed in this stage to evacuate the pus from the middle ear when bulging of tympanic membrane is more or in cases where no improvement occurs with medical treatment. The common micro-organism responsible for acute suppurative otitis media are: 1- Streptococci 2- Pneumococci 3- Haemophilus influenzae 4- Morexella catarrhalis The final outcome or sequelae of acute suppurative otitis media may be: 1- Infection may halt at any stage with complete resolution. 2- Ruptured tympanic membrane may heal with return of normal hearing. 3- Ruptured tympanic membrane may heal with thin paper like membrane with scarring and some residual hearing loss. 4- Ruptured tympanic membrane may not heal and residual dry perforation remains with conductive hearing loss. 5- Acute inflammation may change into chronic suppurative otitis media (tubo-tympanic type). 6- If the condition is not treated properly, complication may arise due to spread of infection to other sites.

TEST YOURSELF

Read the clinical scenario given at the beginning and answers the following questions 1234567-

What are the important points in history taking and clinical examination in this patient? What is the most likely diagnosis in this case? What are the differential diagnosis in this case? How will you manage this case? What are the different stages of acute suppurative otitis media? What are the common micro-organisms responsible for acute suppurative otitis media? What are the possible outcomes or sequelae of acute suppurative otitis media?

20

Clinical scenarios in oto-rhino-laryngology

Case 8 Clinical Scenario A 16 years old boy came in the OPD with complaint of some mass in the left ear for last few months. On inquiry he said that there was history of discharge from the left ear for last many years. Now he also had marked deafness from his left ear. On clinical examination a reddish mass was seen coming out from the left external auditory canal along with profuse purulent discharge (fig. 8.1).

Fig. 8.1 A reddish mass coming out from the external auditory canal along with purulent discharge

Important points in history taking: 1- Detailed history about discharge from the ear i.e. onset, continuous or intermittent, amount, colour, foul smelling, blood stained, aggravating and relieving factors etc. In this patient, discharge was present for the last many years. It was almost continuous, profuse, yellow in colour and foul smelling. Discharge reduces in amount whenever he takes medicine from the general practitioner for few days and then after becomes the same. 2- Detailed history about the mass like its onset and progression. He told that few months back he noticed heaviness and something in his left ear canal. Gradually that mass increased in size and later it came out of the ear canal upto its present size. There was occassional bleeding from the mass whenever he tried to clean the mass. 3- Detailed history regarding deafness and pain. Deafness was present since the start of discharge but initially it was mild and it increased gradually and now he has marked hearing loss. Regarding pain, it occurred off and on and relieved by taking medicines. 4- Any history of fever, headache, altered conciousness, vomiting, neck stiffness or any other neurological symptom. In this case there was no such history. 21

Clinical scenarios in oto-rhino-laryngology

Important points in clinical examination: 1- General physical examination. It showed that the patient was a young, average built boy sitting comfortably and fully oriented in time, space and person. 2- Inspection of the external ear. It showed a reddish, irregular, smooth surface, shiny mass filling the external auditory meatus and coming out from the canal. There was also yellowish muco-purulent discharge around the mass and adjacent pinna. 3- Examination of the post-aural region. There was no significant finding and this region was normal. 4- Probe test of the mass. It showed that the mass is soft, mobile and appears that it was not attached to the external auditory canal and pedicle was deep seated. It did not bleed on touch. 5- Examination of the discharge. It was present around the mass in the external auditory canal and adjacent pinna. Pus was profuse, yellow in colour, mixed with mucous, foul smelling but not blood stained. Pus was collected on a sterile swab and sent for culture and sensitivity. 6- Examination of the tympanic membrane. It was not visible because of the mass. 7- Voice test, showed moderate degree of hearing loss. 8- Tuning fork tests. Rinne’s test was negative in the left ear and positive in right ear. Weber’s test was lateralized towards the left side and Schwabach’s test was equal to the examiner in both ears. 9- Fistula test. It was negative in both the ears. 10- Vestibular function tests. All appeared to be within normal limits. 11- Examination of the facial nerve. It was found to be intact. 12- Examination of the nose and throat. These regions were within normal limits. Investigations: 1- Pus for culture and sensitivity. It showed mixed growth of pseudomonas aeroginosa and staphylococcus aureus. 2- Pure tone audiogram. It showed moderate to severe conductive deafness.

Fig. 8.2 X-ray mastoid (Law’s view)

22

Clinical scenarios in oto-rhino-laryngology

3- Plain X-ray mastoid (Law’s view). It showed haziness or opacification in the mastoid region along with a soft tissue mass in the external auditory canal (fig. 8.2). 4- CT scan of the mastoid bone. This was not done because of un-affordibility by the patient. Diagnosis: This was a case of aural polyp, a complication of chronic suppurative otitis media most likely tubo-tympanic type. Treatment: The patient was planned for aural polypectomy and mastoid exploration under general anaesthesia. The pedicle of the polyp was lying deep, it was hold and cut by a crocodile forceps and the polyp removed completely. Tympanic membrane was found to have a large sized central perforation. Through a post-aural approach mastoid antrum was opened and cortical mastoidectomy was done. Disease was cleared from the mastoid antrum and mastoid air cells. Myringoplasty was also done at the same time by using temporalis fascia graft. Post-operative recovery was uneventful. Discussion: Chronic suppurative otitis media is the chronic inflammation of mucosa of the middle ear cleft. This is conventionally divided into two main clinical types: 1- Tubo-tympanic type 2- Attico-antral type Tubo-tympanic type is virtually always a complication of acute suppurative otitis media. It is the safe variety and relatively more common than the attico-antral type. Serious complications are rare in tubo-tympanic type. With prolonged discharge a polyp may form in the middle ear and comes out through the perforation. Some times polyp is so large that it come out through the external auditory meatus like in this case. Polyp is formed because of extensive oedema in the mucous membrane as a result of chronic inflammation.

TEST YOURSELF

Read the clinical scenario given at the beginning and answers the following questions 123455-

What is the most likely diagnosis in this case? How will you manage this case? See fig. 8.1 and describe its findings. See X-ray in fig. 8.2 and describe its findings. What are the different types of chronic suppurative otitis media? What is an aural polyp? and how it is formed? 23

Clinical scenarios in oto-rhino-laryngology

Case 9 Clinical Scenario A 24 years old male patient came in OPD with the complaint of thick, foul smelling and occasional blood stained discharge from his right ear for last one year. He also had marked hearing impairment and for last few days he had moderate to severe pain in his right ear. Important points in history taking: 1- Detailed history about discharge from the ear i.e. onset, continuous or intermittent, amount, colour, foul smelling, blood stained, aggravating and relieving factors etc. 2- Detailed history regarding deafness and pain in the ear. Deafness was present since the start of discharge but initially it was mild and it increased gradually and now he has marked hearing loss. Regarding pain, it occurred off and on and relieved by taking medicines but now for last few days it is continuous and moderate to severe which was not relieved by taking analgesic drugs. 3- Any history of fever, headache, altered conciousness, vomiting, neck stiffness or any other neurological symptom. In this case there was no such history. Important points in clinical examination: 1- General physical examination. It showed that the patient was a young, average built person sitting comfortably and fully oriented in time, space and person. 2- Examination of the discharge. It was present in the external auditory canal, scanty in amount, thick yellowish in colour, purulent, foul smelling and blood stained. Pus was cleaned completely from external auditory canal and sent for culture and sensitivity. 3- Examination of the tympanic membrane. It showed postero-superior marginal perforation with whitish material (most likely cholesteatoma). Granulation tissues were also seen anterior to it. There was also involvement of the attic region or pars flaccida (fig. 9.1) 4- Examination of the post-aural sulcus and mastoid region for any swelling, redness, sinus or tenderness etc. These areas were appeared to be within normal limits. 5- Voice test. It showed moderate degree of hearing loss. 6- Tuning fork tests. Rinne’s test was negative in right ear and positive in left ear. Weber’s test was lateralized towards the right side and Schwabach’s test was equal to the examiner in both ears. 7- Fistula test. It was negative in both the ears. 24

Clinical scenarios in oto-rhino-laryngology

Fig. 9.1 Tympanic membrane showing posterosuperior marginal perforation with cholesteatoma and granulation tissues

8- Vestibular function tests. All appeared to be within normal limits. 9- Examination of the facial nerve. It was found to be intact. Investigations: 1- Pus for culture and sensitivity. It showed heavy growth of pseudomonas aeroginosa. 2- Pure tone audiogram. It showed moderate conductive hearing loss (about 40-50 dB) on the right side (fig. 9.2) 3- Plain X-ray mastoid (Law’s view). Showed a lytic lesion (most likely cholesteatoma) in the mastoid bone (fig. 9.3). 4- CT scan of the temporal bone and brain. This was not done because of un-affordibility by the patient. Diagnosis: This was a case of chronic suppurative otitis media most likely with cholesteatoma (attico-antral type), causing bone and ossicular erosion. Differential Diagnosis: 1- Tubo-tympanic type of chronic suppurative otitis media. 2- Chronic suppurative otitis media with complication.

Fig. 9.2 Pure tone audiogram showing moderate conductive hearing loss

25

Clinical scenarios in oto-rhino-laryngology

Fig. 9.3 Plain X-ray mastoid (Law’s view) showing cholesteatoma

Treatment: The patient was planned for mastoid exploration under general anaesthesia. Through post-aural approach mastoid antrum was opened. Extensive cholesteatoma was found involving middle ear, additus, mastoid antrum and mastoid air cells. Modified radical mastoidectomy (canal wall down) was done and all the disease was removed. Long process of incus was found to be necrosed by the cholesteatoma but other ossicles were found intact. Tympanoplasty was also done at the same time. Post-operative recovery was un-eventful. Discussion: Chronic suppurative otitis media is the chronic inflammation of mucosa of the middle ear cleft. This is conventionally divided into two main clinical types.: 1- Tubo-tympanic type 2- Attico-antral type Tubo-tympanic type is virtually always a complication of acute suppurative otitis media. It is the safe variety and relatively more common than the attico-antral type. Attico-antral type is considred as dangerous variety because of its aggresive nature and presence of cholesteatoma. Cholesteatoma is a bag of stratified squamous epithelium which contains keratin debris, shed epithelium and bacteria. It has tendency of expansion and causing necrosis of the neighbouring structures and bones. There are four theories for the formation of cholesteatoma: 1- Congenital cell rest theory 2- Metaplastic theory 3- In-growth of squamous epithelium theory 4- Retraction pocket theory Cholesteatoma if not treated may give rise to following complications, which are broadly classified into: 1- Extra-cranial complications: a- Mastoiditis b- Labyrinthitis 26

Clinical scenarios in oto-rhino-laryngology

cdefg-

Facial nerve paralysis Petrositis Otitis externa Thrombosis of internal jugular vein Chronic adhesive otitis media

2- Intra-cranial complications: a- Extra-dural abscess b- Sub-dural abscess c- Brain abscess d- Meningitis e- Sigmoid sinus thrombosis f- Otitic hydrocephalus

TEST YOURSELF

Read the clinical scenario given at the beginning and answers the following questions 12345678-

What is the most likely diagnosis in this case? What are the differential diagnosis in this case? How will you investigate this case? How will you manage this case? See fig. 9.1 and describe its findings. See pure tone audiogram in fig. 9.2 and describe its findings. See X-ray in fig. 9.3 and describe its findings. What is cholesteatoma? and how it is formed?

27

Clinical scenarios in oto-rhino-laryngology

Case 10 Clinical Scenario A 33 years old male patient came in OPD with complaint of asymmetry of his face for the last 2 weeks. On examination he had facial nerve paralysis involving the whole left side (fig. 10.1). On inquiry he also complained of discharge from the left ear for last one year.

Fig. 10.1 Patient with left sided facial nerve paralysis

Important points in history taking: 1- Detailed history regarding onset, progression, associated factors, aggravating and relieving factors etc. Facial paralysis started insidiously about two weeks back. Initially it was very slight and the patient ignored it but gradually it increased to its present condition. There was no aggravating and relieving factor with no significant associated factor. 2- Any history of ear disease or discharge from the ear. He had history of discharge and deafness from his left ear for the last one year. Discharge was usually scanty, thick, foul smelling and often blood stained. He has never consulted to a doctor for his ear complaints. 3- Any history of trauma or head injury. There was no such history. 4- Any history of pain in the ear or headache. There was no history of pain or headache in this patient. 5- Any history of change in taste sensations on the tongue. He has not noticed any change. 6- Any history of vertigo. No history of vertigo. 7- Motor or sensory weakness in any other part of the body or limbs. There was no such problem. Important points in clinical examination: 1- General physical examination. It showed that the patient was a young adult of average built, sitting comfortably and fully oriented in time, 28

Clinical scenarios in oto-rhino-laryngology

space and person. His vital signs were all within normal limits. 2- Examination of motor part of the facial nerve. There was whole left sided facial paralysis (fig. 10.1). 3- Examination of the ears. There was scanty, thick and foul smelling purulent discharge in the left external auditory canal. After cleaning the pus, tympanic membrane was examined. It showed a large posterosuperior marginal perforation with some whitish material in the middle ear (probably cholesteatoma). Right ear was within normal limits. 4- Examination of the post-aural region. There was no significant finding and this region was normal. 5- Voice test, showed moderate degree of hearing loss of the left side. 6- Tuning fork tests. Rinne’s test was negative in the left ear and positive in right ear. Weber’s test was lateralized towards the left side and Schwabach’s test was equal to the examiner in both ears. 7- Fistula test. It was negative in both the ears. 8- Vestibular function tests. All appeared to be within normal limits. 9- Taste sensations on the tongue. It showed a decrease in taste sensations on the left anterior two-third of the tongue. 10- Examination of the nose and throat. These regions were within normal limits. Investigations: 1- Pus for culture and sensitivity from the left external auditory canal. It showed heavy growth of pseudomonas aeroginosa. 2- Pure tone audiogram. It showed moderate to severe conductive deafness on the left side with normal hearing threshold on the right side. 3- Plain X-ray mastoid (Law’s view). It showed haziness in the mastoid region along with bony erosion and cavity formation (finding consistant with a cholesteatoma). 4- CT scan of the mastoid bone. This was not done because of un-affordibility by the patient. 5- Electro-diagnostic tests for facial nerve like ‘minimal nerve excitability test’, ‘electromyography’ and ‘electroneuronography’. These tests were not done because of un-affordibility by the patient. Diagnosis: This was a case of facial nerve paralysis as a complication of atticoantral type of chronic suppurative otitis media (CSOM with cholesteatoma). Treatment: The patient was planned for mastoid exploration and facial nerve decompression under general anaesthesia. Through a post-aural approach mastoid antrum was opened. Extensive cholesteatoma was present in the mastoid antrum and mastoid air cells. All the cholesteatoma was cleared from these area. The bridge was lowered and modified radical mastoidectomy was done. Cholesteatoma was found to be causing facial canal erosion and 29

Clinical scenarios in oto-rhino-laryngology

compression of the facial nerve at its second genu. All the disease cleared from this area and the facial canal was opened around the site of erosion. Nerve was found to be intact so it was covered with a temporalis fascia graft. Post-operative recovery was un-eventful and facial nerve showed improvement. Discussion: The facial nerve is the seventh cranial nerve and it is a mixed nerve contains motor, sensory and secretomotor fibres. The cause of facial nerve paralysis is classified into: 1- Supra-nuclear paralysis: In this type, only the lower half of the face is affected while the upper half escapes paralysis. It is because of the reason that facial nucleus receives fibres from both side of the cortex controlling upper half of the face. 2- Nuclear paralysis: The facial motor nucleus is affected and the clinical picture is similar to that of infra-nuclear type. 3- Infra-nuclear paralysis: The whole side of the face is affected along with other structures supplied by the facial nerve. According to the site of involvement it is further classified into: i- Intra-cranial ii- Intra-temporal iii- Extra-temporal or Extra-cranial The facial nerve is intimately related with the ear, so many ear diseases can cause facial nerve paralysis. Cholesteatoma because of its bone erosion capability, is one of the important cause of facial nerve paralysis.

TEST YOURSELF

Read the clinical scenario given at the beginning and answers the following questions 1234-

Look at fig. 10.1 and state facial nerve paralysis is present on which side of the face? How will you manage this case? Give the classification of facial nerve paralysis. What are the important causes of otogenic facial nerve paralysis?

30

Clinical scenarios in oto-rhino-laryngology

Case 11 Clinical Scenario A 24 years adult male patient came in the OPD with complaint of recurrent discharge from his right ear for many years. He was fond of swimming and developed recurrent ear discharge whenever water went into his right ear during swimming. He has to take antibiotic for few days to make his ear dry. At the time of presentation his ear was dry and otoscopic picture of his right ear is shown in fig. 11.1.

Fig. 11.1 Dry small central perforation involving antero-inferior quadrant of pars tensa

Important points in history taking: 1- Detailed history about the first onset of symptoms i.e how it started? In this patient he did not remember about the first episode of discharge from the ear and how it started. 2- Detailed history regarding discharge i.e. how often, amount, colour, foul smelling, blood stained etc. In this patient ear remained dry for most of the time and discharge usually started after swimming and becames dry again after taking medicines for few days. 3- Detailed history about the hearing impairment and other ear symptoms. In this case he has slight feeling of hearing impairment and occassional tinnitus on right side. 4- Any history of nasal or throat problem/symptom. In this case there was no symptom related with nose or throat. Important points in clinical examination: 1- Assessment of perforation. In this patient, perforation was small, rounded, central and dry involving antero-inferior quadrant of pars tensa (fig. 11.1). There was no discharge. 2- Voice test. He heared whisper in both ears i.e. hearing appears to be within normal limits. 31

Clinical scenarios in oto-rhino-laryngology

3- Tuning fork tests. Rinne’s test was positive in both ears, Weber’s test was lateralized towards the right side and Schwabach’s test was equal to the examiner on both sides. 4- Any signs of middle ear infection. In this patient ear was completely dry with no congestion of tympanic membrane. Middle ear mucosa as seen through the perforation also appeared normal. 5- Any pathology in the nose and throat. In this case both nose and throat were within normal limits with no significant finding. Diagnosis: This was a case of small and dry central perforation of pars tensa. Investigations: 1- Pure tone audiometry. It showed a conductive loss of 15 to 20 dB especially in the lower frequencies (fig 11.2). 2- Pus for C/S is done when the discharge is present. This patient when presented has a dry ear so C/S was not done. 3- Other baseline investigations for general anaesthesia like blood complete picture, ESR, random blood sugar, urine D/R and X-ray chest (PA view). All were within normal limits in this patient.

Fig. 11.2 Pure tone audiogram of the right ear

Treatment: Patient was planned for myringoplasty under general anaesthesia. As this was a small perforation, fat-plug myringoplasty was planned. Margins of the perforation were freshened by a needle and fat plug taken from the lobule was tucked in the perforation. Spongestone was applied all around and external auditory canal was packed with ribbon guaze soaked in BIPP. Post-operative recovery was uneventful and perforation healed completely (fig. 11.3). Discussion: The causes of dry perforation of pars tensa includes: 1- Previous acute otitis media with perforation where healing is incomplete resulting in persistant small perforation. 32

Clinical scenarios in oto-rhino-laryngology

Fig. 11.3 Tympanic membrane at 6 weeks after myringoplasty

2- Persistant perforation after myringotomy and grommet insertion. 3- Tubo-tympanic type of chronic suppurative otitis media where infection had been settled with medical or surgical treatment but perforation remains there. 4- Traumatic perforation where healing is incomplete. Many patients live with small tympanic membrane perforation that is entirely without symptoms. Symptoms of small and dry perforation include audible whistling sounds during sneezing and nose blowing, decreased hearing and a tendency to infection during cold or when water goes into the ear. Hearing loss in small perforation is usually mild and may be unnoticeable by the patients. In swimmers, divers and other water sports enthusiasts, recurrent ear infection may occur. Medical therapy for small pars tensa perforation is directed at controlling ear infection when discharge is present by giving appropriate systemic antibiotics, aural toilet and antibiotic ear drops. Surgical treatment is repair of the tympanic membrane (myringoplasty). The options for myringoplasty in such a case are: 1- Overlay or underlay myringoplasty with graft material like temporalis fascia. But these two techniques of myringoplasty are more suitable for medium size to large perforations. 2- Fat-plug myringoplasty is very suitable for small perforations like this. 3- Repeated cauterization of margins of the perforation with chemicals.

TEST YOURSELF

Read the clinical scenario given at the beginning and answers the following questions 12345-

What are the important points in history taking and clinical examination in this patient? What investigations will you order in such a case? How will you manage this patient? What are the different options for myringoplasty? What are the advantages of fat-plug myringoplasty? 33

Clinical scenarios in oto-rhino-laryngology

Case 12 Clinical Scenario A 17 years old male patient came with the complaint of impaired hearing from the left ear for last 7 to 8 months. It was often associated with crackling and bubbling noises with sensation of fluid in his left ear especially on changing head position. There was no history of discharge or pain in the ear. On examination, otoscopic findings of his left ear is shown in fig 12.1.

Fig. 12.1 Tympanic membrane of left side showing fluid in the middle ear with fluid level

Important points in history taking: 1- Detailed history regarding deafness. In this patient, deafness was gradual in onset, continuous but fluctuating in severity and sound appeared quieter but not distorted. 2- Any history of discharge or pain. In this case there was no history of discharge or pain in his left ear. 3- Any symptom related with nose or throat. He had history of recurrent nasal discharge and post-nasal dripping, which was usually relieved by taking medicines for few days. Important points in clinical examination: 1- Examination of the tympanic membrane. It showed fluid in the middle ear with fluid level. Tympanic membrane appeared retracted and somewhat congested with absence of cone of light (fig. 12.1). 2- Valsalva’s manuever. There was no affect of Valsalva’s manuever on the tympanic membrane of left side i.e. eustachian tube patency was absent. 3- Siegel’s pneumatic otoscopy. It showed no movement of tympanic membrane. 4- Voice test. It showed moderate degree of deafness on the left side. 5- Tuning fork tests. Rinne’s test was negative on the left side and positive 34

Clinical scenarios in oto-rhino-laryngology

on the right side. Weber’s test was lateralized towards the left side and Schwabach’s test was equal to the examiner on both sides. 6- Examination of the nose and throat. It did not showed any significant finding. Investigations: 1- Pure tone audiogram. It showed moderate conductive hearing loss (3040 dB in the left ear (fig. 12.2). Right ear showed hearing within normal limits.

Fig. 12.2 Pure tone audiogram of the left ear showing moderate conductive hearing loss

2- Speech audiogram. It showed 100% speech discrimination score on both the side. 3- Tympanogram. It showed type B graph on the left side (fig. 12.3) and type A graph on the right side. 4- Plain X-ray soft tissue nasopharynx (lateral view) and X-ray PNS (water’s view) for any pathology in the nasopharynx, nose and PNS and these were within normal limits. 5- Other baseline investigations for general anaesthesia. All were within normal limits. Diagnosis: This was a case of otitis media with effusion in the left ear.

Fig. 12.3 Tympanogram of the left side showing type B graph

35

Clinical scenarios in oto-rhino-laryngology

Treatment: Patient was planned for myringotomy and grommet insertion under general anaesthesia. Radial incision was given with myringotomy knife in the antero-inferior quadrant of the pars tensa. Mucoid fluid came out, which was removed by suction and the grommet inserted (fig. 12.4). Post-operatively antibiotic, analgesic/NSAID and systemic decongestant were given for 2 weeks. Patient’s symptoms improved markedly. Grommet was removed after 3 months.

Fig. 12.4 Tympanic membrane after myringotomy and grommet insertion

Discussion: The term ‘otitis media with effusion’ or ‘non-suppurative otitis media’ is applied to the clinical condition characterized by the presence of nonpurulent fluid in the middle ear cleft. The fluid may be serous, mucoid or sometimes hemorrhagic in nature. Acute and chronic forms can sometimes be distinguished according to the mode of onset or by duration, but the distinction may not always be clear and the condition is often recurrent. In acute form of otitis media with effusion, duration of the disease is less than 3 weeks and in chronic form duration is longer than 3 months. From time to time different terminologies have been proposed for this disease. In 1886 Politzer first described the term “otitis media catarrhalis”. After the end of the second world war the term “glue ear” was first introduced by Jordan. The other commonly used synonyms for this condition are secretory otitis media, serous otitis media, mucinous otitis media, catarrhal otitis media, exudative otitis media and glue ear. It is considered as the most important cause of deafness in the children world over. The exact aetiology of this condition is unknown. The following factors are described as the aetiological factors for this condition. 1- Occlusion of the eustachian tube 2- Allergy 3- Viral infection 4- Unresolved acute otitis media 5- Cleft palate 36

Clinical scenarios in oto-rhino-laryngology

TEST YOURSELF

Read the clinical scenario given at the beginning and answers the following questions 12345678-

What is your diagnosis in this case? What investigations will you order in this case? How will you treat this case? Briefly outline the pathology of otitis media with effusion. Describe the findings of tympanic membrane as shown in fig. 12.1. Describe the findings of pure tone audiogram in fig. 12.2. Which type of tympanogram graph is seen in fig. 12.3. What is the site of incision for myringotomy in cases of otitis media with effusion?

37

Clinical scenarios in oto-rhino-laryngology

Case 13 Clinical Scenario A 28 years old female patient presented with the complaint of impaired hearing from both ears for the last 10 to 12 years. She had more problem in hearing in calm and quiet environment as compared to noisy places. There was no history of pain or discharge from the ears. On examination her external auditory canal and tympanic membrane were appeared normal in both ears. Important points in history taking: 1- Detailed history about deafness i.e. onset, progression, associated factors, aggravating and relieving factors. According to patient onset was insidious, initially hearing impairment was mild but it increased gradually to the current situation. Deafness was continuous with no fluctuation in severity, with no agrravating or relieving factor. Occasionally it was associated with ringing sensations in both ears. 2- Hears better in quiet room or in noisy places. In this patient she had better hearing in noisy places (paracussis Wallisi). 3- Age of onset. In this patient complaint had started when she was about 16 to 17 years of age. 4- Effect of pregnancy on hearing. She had two childrens and according to her, deafness had increased during pregnancy in both the times. 5- History of deafness in the family. According to patient her mother and her elder sister had also similar complaints. 6- Any history of pain or discharge from the ears. In this case there was no history of pain or discharge from the ears. 7- Any complaint related with the nose or throat. In this patient there was no such complaint. Important points in clinical examination: 1- General physical examination. She is a young, fair coloured lady of average built and fully oriented. 2- Examination of the external ear, external auditory canal and tympanic membrane, all appear to be within normal limits. 3- Voice test, showing moderate degree of hearing loss. 4- Tuning fork tests. Rinne’s test was negative in both the ears, Weber’s test was centralized and Schwabach’s test was equal to the examiner in both ears. 5- Valsalva’s maneuver and Siegel’s pneumatic otoscopy. It showed patent eustachian tube and tympanic membrane was mobile. 6- Vestibular function tests, all were within normal limits. 7- Examination of the nose and throat. No significant finding on it. 38

Clinical scenarios in oto-rhino-laryngology

Investigations: 1- Pure tone audiogram. It showed moderate degree of conductive deafness in both ears. There was a dip in bone conduction at 2000 Hz (Carhart’s notch) in both ears (fig. 13.1 and 13.2). 2- Speech audiogram. Speech discrimination score was 98% in right ear and 96% in the left ear. 3- Tympanogram, showing type As graph in both the ears (fig. 13.3 and 13.4).

Fig. 13.1 Pure tone audiogram of right ear

Fig. 13.2 Pure tone audiogram of left ear

Fig. 13.3 Tympanogram of right ear

39

Clinical scenarios in oto-rhino-laryngology

Fig. 13.4 Tympanogram of left ear

Diagnosis: This was a case of otosclerosis involving both ears. Differential Diagnosis: The other causes of conductive deafness due to pathology in the middle ear, should be considered in differential diagnosis, like: 1- Congenital fixation of the footplate of stapes. Deafness is present since birth in this condition. 2- Adhesive otitis media. Tympanic membrane will not appear normal looking and mobile. 3- Tympanosclerosis. White chalky patches are seen on tympanic membrane. 4- Ossicular dislocation. Type AD tympanogram will be seen in this condition. Treatment: The patient was planned for stapedotomy and teflon piston insertion under general anaesthesia on the right side first. Rosen’s incision was given and tympanomeatal flap raised. Chorda tympani nerve was reflected downwards and postero-superior bony buttress removed. Tendon of stapedius muscle was cut and supra-structure of stapes removed after fracture. A hole is made in the footplate of stapes and teflon piston (Shea’s piston) was

Fig. 13.5 Stapedotomy operation with insertion of teflon piston

40

Clinical scenarios in oto-rhino-laryngology

inserted (fig. 13.5). Post-operative recovery was uneventful with improvement in hearing. Discussion: Otosclerosis or Otospongiosis is a localized disease of the otic capsule. There is formation of new spongy bone, which causes ankylosis of the footplate of stapes to the margins of the oval window or may invade to involve the cochlea. This process occurs in the endochondral layer of the bony otic capsule. These bony changes occur at one or more constant sites of otic capsule. The commonest site is anterior to the oval window (fissula antefenestrum) causing ankylosis of the footplate of stapes to the margins of oval window. Abnormal bone may be present at other sites of otic capsule but usually causes no clinical manifestation.

TEST YOURSELF

Read the clinical scenario given at the beginning and answers the following questions 12345678-

What is the most likely diagnosis in this case? What are the differential diagnosis in this case? How will you investigate this case? How will you treat this case? What are the findings on pure tone audiogram in fig 13.1 and 13.2? What are the findings on tympanogram in fig. 13.3 and 13.4? Outline the steps of stapedotomy operation. What is the pathology of otosclerosis?

41

Clinical scenarios in oto-rhino-laryngology

Case 14 Clinical Scenario A 68 years old retired bank manager came with the complaint of progressive bilateral hearing impairment for the last few years. He had marked difficulty in understanding speech especially in the presence of background noises. On examination his external ears and tympanic membranes appeared to be normal. Important points in history taking: 1- Detailed history about deafness i.e. its onset, continuous or intermittent, progressive or not, unilateral or bilateral, hears better in noisy places or in quiet room, associated factors. In this case patient had bilateral, progressive hearing loss and difficulty in understanding speech in the presence of background noises (this is typically seen in cases of sensorineural deafness). 2- Any history of discharge from the ears. In this case there was no history of discharge from the ears. 3- Any history of diabetes mellitus. In this case no history of diabetes mellitus. 4- Occupational history and exposure to loud sound. In this case he was a retired bank manager and there was no history of any occupational noise exposure or sudden sound like bomb blast or firearm. 5- Any history of using any ototoxic drug. In this case there was no history of using any drug having such affects. Important points in clinical examination: 1- Examination of the external ear, external auditory canal and tympanic membrane. All appeared to be within normal limits. 2- Voice test. It showed moderate degree of hearing loss. 3- Tuning fork tests. Rinne’s test was positive in both the ears, Weber’s test was centralized and Schwabach’s test was less than the examiner in both ear (it means patient has bilateral and equal sensori-neural loss). 4- Vestibular function tests. All were within normal limits. 5- Examination of the nose and throat. No significant finding on it. Investigations: 1- Pure tone audiogram. It showed moderate to severe slopping type (i.e. more pronounced in higher frequencies) sensori-neural deafness in both ears (fig. 14.1 and 14.2). 2- Speech audiogram. Speech discrimination score was 69% in right ear and 72% in the left ear. 3- Tympanogram, showed type A graph in both the ears. 42

Clinical scenarios in oto-rhino-laryngology

Fig. 14.1 Pure tone audiogram of right ear

Fig. 14.2 Pure tone audiogram of left ear

Diagnosis: This was a case of bilateral sensori-neural hearing loss, most probably senile deafness or presbyacusis. Differential Diagnosis: The other causes of sensori-neural deafness should be considered in differential diagnosis. Treatment: This was a case of bilateral moderate to severe sensori-neural hearing loss because of ageing, so following treatment was offered to the patient: 1- Hearing aid. Patient was fitted with behind the ear type (BTE), digital programmable hearing aids in both the ears (fig. 14.3). 2- Lip reading and auditory training. Discussion: The term presbyacusis or senile deafness is used to described hearing loss resulting from degenerative changes due to ageing. Deafness is characteristically bilateral, symmetrical and slowly progressive. It affects both 43

Clinical scenarios in oto-rhino-laryngology

Fig. 14.3 Behind the ear (BTE) type hearing aid

the sexes equally. Degenerative changes occur as a result of vascular insufficiency due to sclerosis, thrombosis or atherosclerosis and involves: 1- Hair cells in the organ of Corti 2- Neural tissues in the spiral ganglion 3- Stria vascularis 4- Basal membrane

TEST YOURSELF

Read the clinical scenario given at the beginning and answers the following questions 12345-

What is the most likely diagnosis in this case? What are the differential diagnosis in this case? How will you investigate this case? How will you manage this case? What is senile deafness or presbyacusis?

44

Clinical scenarios in oto-rhino-laryngology

Case 15 Clinical Scenario A 48 years old textile factory worker came with the complaint of impaired hearing from both ears for the past many years. He had marked difficulty in understanding speech especially in noisy places. There was no history of pain or discharge from the ears but occassionally he also had ringing sounds in both ears. On clinical examination pinna, external auditory canal and tympanic membrane were all normal. Important points in history taking: 1- Detailed history about deafness i.e. its onset, continuous or intermittent, progressive or not, unilateral or bilateral, hears better in noisy places or in quiet room, associated factors. In this case, deafness was insidious in onset, continuous, progressive, bilateral and equal in both ears. 2- Occupational history and exposure to loud sound. He was a textile factory worker and had been working on heavy machines with loud sound 8 to 10 hours a day for the past 30 years. There was no facility of any sort for protection of the ears from loud sound in his factory. 3- Any history of diabetes mellitus. In this case no history of diabetes mellitus. 4- Any history of trauma to the ears or head. There was no such history. 5- Any history of using any ototoxic drug. In this case there was no history of using any drug having such affects. Important points in clinical examination: 1- Examination of the external ear, external auditory canal and tympanic membrane. All appeared to be within normal limits. 2- Voice test, showed moderate degree of hearing loss. 3- Tuning fork tests. Rinne’s test was positive in both the ears, Weber’s test was centralized and Schwabach’s test was less than the examiner in both ears (it means patient has bilateral and equal sensori-neural loss). 4- Vestibular function tests. All were within normal limits. 5- Examination of the nose and throat. No significant finding on it. Investigations: 1- Pure tone audiogram. It showed moderate to severe slopping type (i.e. more pronounced in higher frequencies) sensori-neural deafness in both ears (fig. 15.1 and 15.2). 2- Speech audiogram. Speech discrimination score was 70% in right ear and 74% in the left ear. 3- Tympanogram, showed type A graph in both the ears. 45

Clinical scenarios in oto-rhino-laryngology

Fig. 15.1 Pure tone audiogram of right ear

Fig. 15.2 Pure tone audiogram of left ear

Diagnosis: This was a case of bilateral sensori-neural hearing loss, probably noise induced hearing loss (NIHL). Differential Diagnosis: The other causes of sensori-neural deafness should be considered in differential diagnosis. Treatment: This was a case of bilateral moderate to severe noise induced hearing loss, so following treatment was offered to the patient. 1- Hearing aid. Patient was fitted with behind the ear type (BTE), digital programmable hearing aids in both the ears. 2- Lip reading and auditory training. 3- Protection of the ears from loud sound during working, to prevent further noise induced hearing loss. Discussion: Noise induced deafness or hearing loss (NIHL) is caused by prolong exposure to loud sound. The degree of deafness is proportional to sound intensity 46

Clinical scenarios in oto-rhino-laryngology

and duration of daily exposure, though there is a marked variation in individual susceptibility. OSHA (Occupational Safety and Health Administration, USA) standard for the safe exposure to different sound intensity levels per day for five days a week is listed below. These levels are referred to as the permissible exposure level (PEL). - 16 hours 85 dB - 8 hours 90 dB - 6 hours 92 dB - 4 hours 95 dB - 3 hours 97 dB - 2 hours 100 dB - 1.5 hours 102 dB - 1.0 hour 105 dB - 30 minutes 110 dB - 15 minutes 115 dB

TEST YOURSELF

Read the clinical scenario given at the beginning and answers the following questions 123456-

What is the most likely diagnosis in this case? What are the differential diagnosis in this case? How will you investigate this case? How will you manage this case? What are the causes of noise induced hearing loss? What is permissible exposure level (PEL) of sound?

47

Clinical scenarios in oto-rhino-laryngology

Case 16 Clinical Scenario A 48 years old male patient came in OPD with the complaints of severe vertigo for last one day. Along with vertigo he has ringing sounds and hearing loss in his right ear. He had history of similar attacks, three or four times before during the last three years, which improved by medication and he remained normal between the attacks. Important points in history taking: 1- Detailed history about the vertigo. According to the patient it started suddenly yesterday and since then it was continuous. There was no affect of posture i.e. it remain same on standing, sitting or lying down. Patient was feeling that all things around him was rotating. There was no specific aggravating or relieving factor. It was associated with nausea and vomitting as well. 2- Detailed history about hearing loss and tinnitus. According to him, both symptoms started immediately after start of vertigo. Hearing loss was in right ear, mild to moderate, continuous with no aggravating and relieving factors. He has ringing sounds in his right ear, which was constant and same with no aggravating or relieving factors. 3- History of similar attacks in the past. According to patient during the last three years he had three to four similar attacks before which lasted for a weak or so and completely improved by medication. There was no complaint or symptom in between the attacks. 4- Any history of head trauma, headache or any other neurological symptom. There was no such history. 5- Any history of using any vestibulo-toxic drug. In this case there was no history of using any drug having such affects. 6- Any history of allergy. Patient has allergy to multiple things. Important points in clinical examination: 1- General physical examination. The patient was a middle aged man of average height and obese built, looking anxious but well oriented in time, space and person. His vital signs were within normal range. 2- Examination of the external ear, external auditory canal and tympanic membrane. All appeared to be within normal limits. 3- Voice test, showed moderate degree of hearing loss on the right side. 4- Tuning fork tests. Rinne’s test was positive in both the ears, Weber’s test was lateralized towards the left ear and Schwabach’s test was less than the examiner in right ear and equal to the examiner in left ear. 5- Fistula test. It was negative in both the ears. 6- Facial nerve examination. It was intact. 48

Clinical scenarios in oto-rhino-laryngology

7- Spontaneous nystagmus. It was positive with the fast component towards the left side. 8- Gait. Patient deviated towards the right side when he was walking with eyes closed. 9- Romberg’s test. Patient swayed on the right side when he was standing with eyes closed. 10- Cerebellar functions tests like finger nose test, rapid alternating movements (dysdiadochokinesia) and rebound phenomenon, were all within normal limits. 11- Examination of other cranial nerves. All were intact. 12- Examination of the nose and throat. No significant finding on it. Investigations: 1- Pure tone audiogram. Showing moderate sensori-neural type of deafness, equal in all frequencies on the right side and normal hearing on the left side (fig. 16.1). 2- Speech audiogram. Speech discrimination score was 70% in right ear and 96% in the left ear. 3- Recruitment test. It was positive on the right side, showing a cochlear lesion. 4- Caloric test. It was not done because of acute attack of severe vertigo. Diagnosis: This was a case of Meneire’s disease. Differential Diagnosis: The other causes of vertigo should be considered in differential diagnosis, like: 1- Vestibular neuronitis. It usually follows an attack of influenza and hearing remains normal in this condition. 2- Labyrinthitis. It is mostly a complication of chronic suppurative otitis media or ear surgery. Sometimes labyrinthitis of viral origin can occur.

Fig. 16.1 Pure tone audiogram of right ear

49

Clinical scenarios in oto-rhino-laryngology

3- Benign paroxysmal positional vertigo (BPPV). In this condition vertigo only occurs in certain specific head positions. 4- Central causes of vertigo. Treatment: During the acute attack of vertigo, patient was advised strict bed rest and antivertiginous drugs or labyrinthine sedatives were prescribed. In addition patient was advised for: 1- Low salt diet 2- Avoid excessive intake of water 3- Quit smoking 4- Avoid stress and bring a change in life style 5- Avoid things that are causing allergic reaction The patient became symptom free in ten days. Regular follow-up was advised and after taking above mentioned precautions, there was no further attack of vertigo in the follow-up period of last one year. Discussion: Meniere’s disease is the disorder of endolymphatic labyrinth. It is characterized by sudden paroxysmal attacks of vertigo, deafness and tinnitus. It involves both the cochlear and the vestibular components of the inner ear. The most consistent histological finding in Meniere’s disease is dilatation of the endolymphatic compartment of the inner ear. The basic defect is either over production or diminished absorption of endolymph. As a result of this scala media is distended with endolymph. Due to over distension of scala media, rupture of Reissner’s membrane occurs. This leads to mixing of endolymph and perilymph, which disturbs the cochlear microphonics and action potentials of the nerves. This occurs because endolymph is a potassium rich fluid and perilymph is a sodium rich fluid. The attack continues till the ruptured membrane is healed and local biochemistry is corrected. The same type of attack occurs again when the scala media is over distended.

TEST YOURSELF

Read the clinical scenario given at the beginning and answers the following questions 12345-

What is the most likely diagnosis in this case? What are the differential diagnosis in this case? How will you investigate this case? How will you treat this patient? What is the pathology of Meneire’s disease and why it occurs in paroxysmal attacks?

50

Clinical scenarios in oto-rhino-laryngology

Case 17 Clinical Scenario A 46 years old female patient came with the complaints of repeated vertigo, nausea and occasional vomiting, lasting for 2 to 3 minutes for last 3 months. The vertigo was exacerbated whenever she turned her head to the left while lying in bed or rising from the bed. She has no history of deafness and tinnitus. Important points in history taking: 1- Detailed history regarding vertigo. According to the patient, her symptoms started after a road traffic accident when she got minor head injury. She has feeling of rotation whenever she turned her head to the left while lying down or rising from the bed. There was no problem on standing, sitting or walking. She also has feeling of nausea and occasionally vomiting during the attack of vertigo which remained there for few minutes and she became normal after few minutes. The symptoms were constant and same since starting with no aggravating or relieving factors except head movement. 2- Any history of discharge, deafness, tinnitus or any symptom related with the ear. There was no other complaint. 3- Any history of neck pain or stiffness. There was no such history but occasionally she felt dizziness on rapid head movement. 4- Any history of trauma or head injury. There was a definite history of road traffic accident and head injury and all the symptoms started after that. 5- Past medical history for diseases like hypertension, diabetes mellitus, syphillis etc. There was no history of such diseases. 6- Any other neurologic complaint. There was no such history. 7- Drug history. She has used different medicines for these complaints with no relief and she did not know their names. There was no history of using any drug before the start of these symptoms. 8- Any history of psychiatric illness. There was no such history. Important points in clinical examination: 1- General physical examination. The patient was a middle aged woman of average height and built, sitting comfortably and fully oriented. Her vital signs were within normal limits and no other positive finding on general physical examination. 2- Examination of the ears. Pinna, external auditory canal and tympanic membrane were all normal on both the sides. 3- Voice test and tuning fork tests. All were within normal limits. 4- Examination of all cranial nerves. All were intact. 5- Examination for spontaneous nystagmus. It was absent. 51

Clinical scenarios in oto-rhino-laryngology

6- Gait without and with closed eyes. It was within normal limit. 7- Cerebellar function tests. All were within normal limits. 8- Dix-Hallpike test. There was nystagmus and feeling of vertigo when the patient was lowered with head turned towards the left. The nystagmus was delayed in onset and showed fatigability. There was no nystagmus with head turned towards the right side. Differential Diagnosis: The first step in the differential diagnosis of a vestibular disorder is to delineate between a peripheral or central lesion. The following disorders should be included in the differential diagnosis of this patient: 1- Benign paroxysmal positional vertigo 2- Traumatic injury to the labyrinth 3- Vestibular neuronitis 4- Labyrinthitis 5- Meniere’s disease 6- Perilymph fistula 7- Acoustic neuroma 8- Postural hypotension 9- Multiple sclerosis 10- Psychiatric illness or use of drugs like tranqulizers Investigations: 1- Baseline investigations like complete blood picture, blood sugar, serum electrolytes, renal function tests. All were within normal limits. 2- Pure tone and speech audiogram. Both were within normal limits. 3- Tympanogram. Type A graph was seen on both the sides. 4- Caloric test. It was within normal limits on both sides with cold and warm water. Diagnosis: This was a case of benign paroxysmal positional vertigo (BPPV), affected ear being the left side. Treatment: After all clinical workup, patient was planned for ‘particle repositioning procedure’ or ‘Epley’s maneuver’. It involves sequential movement of the head into different positions, staying in each position for roughly 30 seconds (fig. 17.1). Initially patient was advised to sit on a couch (position 1). As the affected ear was left, patient’s head was rotated to 45o on the left and then head was lowered (position 2). Then keeping the patient supine, her head was rotated 90o towards the right (position 3). Again after 30 seconds, patient’s whole body was rolled or rotated towards right, so that her face was 45o towards the ground (position 4). Patient was then advised to sit again from this position (position 5). Patient responded well after this maneuver and became symptom free. 52

Clinical scenarios in oto-rhino-laryngology

Fig. 17.1 Method of performing Epley’s maneuver

Discussion: Benign paroxysmal positional vertigo (BPPV) is a common cause of dizziness and about 20% of all cases of dizziness are due to this. This incidence increases with age and in older people upto 50% of the cases of dizzines are due to BPPV. In this condition there are recurrent, paroxysmal and short-lived attacks of vertigo in certain head positions. Exact aetiology of this condition is unknown but head injury is supposed to be an important factor. Due to head injury otoconia or calcium crystals present in the utricle are dislodged and settle in the posterior semi-circular canal. When the head is reoriented relative to gravity, the gravity-dependent movement of the heavier otoconial debris within the affected semicircular canal causes abnormal endolymph displacement and a resultant sensation of vertigo. The head position, which provokes the vertigo, should be avoided and it is often all that is required. Role of antivertiginous drug is controversial and may get relief in some patients. Many of the cases may settle spontaneously within a few months but some may persist for years. There are two office procedures to treat BPPV, both are intended to move otoconial debris from the posterior semi-circular canal to some less sensitive areas. These include ‘Epleys’ maneuver’ and ‘Semont maneuver’. Surgical treatment is indicated in cases of persistant disease for years not responding to any other treatment. It include ‘canal plugging’, singulare nerve section, vestibular nerve section and labyrinthectomy.

TEST YOURSELF

Read the clinical scenario given at the beginning and answers the following questions 12345-

What are the important points in history taking and clinical examination in this case? How will you investigate this case? How will you manage this case? What is the pathophysiology of benign paroxysmal positional vertigo? Describe the method of Epley’s maneuver. 53

Clinical scenarios in oto-rhino-laryngology

Case 18 Clinical Scenario A mother brought her one year old son with the complaint that he did not pay attention to any kind of sound and his condition was same since birth. She thought that he was deaf and she came here for assessment and consultation. Important points in history taking: 1- Detailed history regarding presenting complaints. According to his mother he did not respond to any sound stimuli and she thought that he was completely deaf. 2- History about the development of the child and other milestones. All other milestones were normal and otherwise he was perfectly normal. 3- History during pregnancy for any disease, use of drugs, viral infections, exposure to radiation etc. In this case there was no significant history during pregnancy. 4- Detailed history about his birth like type of delivery, time duration of delivery, hypoxia of the child during or after delivery, prematurity of the child, birth weight of the child etc. 5- Detailed history of events after child birth like, jaundice, Rh factor incompatibility, hypothyroidism etc. 6- Consanguinity, whether his mother and father are cousins or close relatives. In this case his mother and father were first cousins. 7- Condition of the other siblings. He has one elder sister and one brother and both were absolutely normal. 8- History of congenital deafness in the other family members and close relatives of both maternal and paternal side. In this case there was history of congenital deafness in many persons among close relatives of both maternal and paternal side. Important points in clinical examination: 1- General physical examination. The child was active and otherwise normal looking according to his age. 2- Inspection of the pinna, external auditory canal and tympanic membrane. All were normal. 3- Distraction test. There was no response of the child in distraction test, means he was not listening to the sound stimuli. 4- Other ENT examination. It was within normal limits. 5- Other systemic examination for any other abnormality in the body. In this case no other abnormality was detected. Investigations: 1- Brainstem evoked response audiometry (BERA). No reproducible wave 54

Clinical scenarios in oto-rhino-laryngology

form pattern was obtained even on maximum sound stimuli, showing profound deafness in both the ears. 2- CT scan of the whole ear and mastoid area of both sides. No significant positive finding was seen on CT scan. Diagnosis: The patient was a case of congenital deafness which was profound and bilateral. The most probable cause may be genetic because of consanguinity and similar cases in the family. Treatment: After all clinical workup, patient was planned for cochlear implant. Surgery was done successfully and post-operative recovery was uneventful. After cochlear implant auditory rehabilitation was started and patient showed good improvement. Discussion: Around the globe, deafness is a common public health problem and it is estimated that 200 million people on earth are suffering from this problem. Majority of them belong to the developing countries. The causes of congenital deafness are classified into genetic and non-genetic or environmental. Worldwide severe to profound deafness affect 1 in 2,000 newborns and half of them have genetic etiology. Inherited deafness is more common in population where consanguineous marriages are common like in Pakistan. The only option for treatment in severe to profound bilateral deafness is cochlear implant. Cochlear implant is the device which performs the function of cochlea i.e. it converts sound energy into electrical impulses and stimulates the cochlear nerve fibres. At present cochlear implant is indicated in a patient having bilateral profound sensori-neural deafness with no measurable speech discrimination and even the most powerful hearing aid is not effective. It is most useful in post-lingually deaf adults and children i.e. where hearing loss occur after acquisition of speech. Pre-lingually deaf adults may also get benefit of cochlear implant. Congenitally deaf children get minimum benefit from cochlear implant.

TEST YOURSELF

Read the clinical scenario given at the beginning and answers the following questions 123456-

What are the important points in history taking in this case? What are the important points in clinical examination in this case? How will you investigate this case? How will you manage this case? What is a cochlear implant and how it works? Which type of patient can get the maximum benefit from cochlear implant? 55

Clinical scenarios in oto-rhino-laryngology

Section II

NOSE & PARANASAL SINUSES Case 19Case 20Case 21Case 22Case 23Case 24Case 25Case 26Case 27Case 28Case 29Case 30-

Deviated nasal septum Nasal trauma with fracture Antro-choanal polyp Septal haematoma Ethmoidal polypi Septal adhesion Allergic rhinitis Foreign body of the nose Boil in the nose Epistaxis Chronic rhino-sinusitis Nasopharyngeal angiofibroma

57

59 62 66 70 73 76 78 81 84 86 89 92

Clinical scenarios in oto-rhino-laryngology

Case 19 Clinical Scenario A 32 years old male patient came in the OPD with the complaint of nasal obstruction for the last 3 years. It was associated with post-nasal dripping and thick nasal discharge off and on. Nasal obstruction was almost continuous and more on the left side. Anterior rhinoscopy with nasal speculum showed deviated nasal septum on the left side (fig 19.1).

Fig. 19.1 Anterior rhinoscopy with nasal speculum showing deviated nasal septum

Important points in history taking: 1- Effect of posture on nasal obstruction. On lying down dependent nasal cavity was blocked and patient was unable to lie on his right lateral position. 2- History of excessive sneezing and watery rhinorrhoea. These were absent in this case. 3- Facial pain and headache. These were absent in this case. 4- Other symptoms of allergy like itching or watering of the eyes, skin rashes etc. All were absent. 5- Aggravating and relieving factors. Nasal obstruction and post-nasal dripping were aggravated by exposure to cold and relieved by taking medicines from local general practitioner. 6- Previous history of trauma to nose. There was a history of fall from the stairs and trauma on the nose many years back. Important points in clinical examination: 1- External inspection of the nose for deformity. Slight deviation of tip of the nose was present towards the left side. 2- Anterior rhinoscopy. It showed a deviated nasal septum with convexity 59

Clinical scenarios in oto-rhino-laryngology

on the left side and a sharp spur on the left side (fig. 19.1). 3- Nasal patency test. It was reduced on the right side and absent on the left side. 4- Palpation of the sinuses for tenderness. Not tender in this case. 5- Posterior rhinoscopy. Nothing significant found on this examination in this case. 6- Examination of the throat and ears for any pathology related with his nasal disease. 7- Rigid nasal endoscopy under local anaesthesia in the OPD. It showed gross deviation of the nasal septum on left side with a spur. There is also hypertrophy of the inferior turbinate on the right side. At the time of this examination no muco-purulent or purulent secretion seen in nasal cavity or in any meati. Diagnosis: This was a case of deviated nasal septum. History was also suggestive of recurrent rhino-sinusitis, but at the time of presentation there were no symptoms or signs of infection. Investigations: 1- Blood CP: For total and differential leucocyte count, TLC was 13,000/mm3, neutrophils were 70% and eosinophils were 3%. 2- X-ray PNS (Water’s view or occipito-mental view): It showed that all the sinuses were clear. Nasal septum deviation on the left with a spur was clearly visible (fig. 19.2) 3- Serum IgE level: It was within normal limits. 4- CT scan of nose and PNS in both axial and coronal views. It was not done in this case because of its cost. 5- Other baseline investigations for G/A fitness like X-ray chest (PA view), blood sugar, ECG, urine D/R. All were within normal limits in this case. Treatment: After all investigations, patient was planned for septal surgery

Fig. 19.2 X-ray PNS (Water’s view) showing DNS on the left side with a spur

60

Clinical scenarios in oto-rhino-laryngology

(septoplasty) under general anaesthesia. Discussion: Symptomatic deviated nasal septum requires surgery. Two surgical procedures are described for deviated nasal septum, sub-mucous resection (SMR) and septoplasty. SMR was the classical operation for DNS in adults at one time but now a days septoplasty is the preferred choice. Septoplasty can be performed in children having deviated nasal septum producing symptoms.

TEST YOURSELF

Read the clinical scenario given at the beginning and answers the following questions 1234-

How will you investigate this case? What are the surgical options for treating this patient? Compare differences between SMR and septoplasty. Enlist complications of septal surgery.

61

Clinical scenarios in oto-rhino-laryngology

Case 20 Clinical Scenario A 16 years old boy came with the complaint that he fell down from the stairs about five days before and got trauma on his face and nose, which resulted in nasal deformity. There was bleeding from the nose at that time which stopped spontaneously after applying ice packs. Now he has marked nasal obstruction mainly on the right side. Important points in history taking: 1- Exact nature of the trauma. 2- Injuries at other parts of the body. There was no injury to other body parts. 3- History of unconsciousness at the time of fall. This patient remained conscious after fall. 4- Amount of blood loss. Only small amount of blood was lost in this patient. 5- Shape of the nose before trauma. If any previous photograph is available, then shape of the nose can be compared before and after the trauma. 6- Any previous history of nasal obstruction or other nasal symptom before trauma. There was no problem related with nose before trauma. Important points in clinical examination: 1- General physical examination regarding level of consciousness, vital signs, anaemia and trauma to the other parts of the body. 2- Examination for shape of the nose from front, sides and above by standing behind the patient. In this patient there was deformity of the external nose lateral displacement of the nasal bones on right side and deviation of tip of the nose on left side (fig. 20.1). 3- Anterior rhinoscopy and examination of the nasal vestibule. It showed grossly deviated nasal septum on the right side almost touching the lateral wall (fig. 20.2).

Fig. 20.1 Patient with trauma to the nose causing external deformity with deviation of the nasal bridge to right side

62

Clinical scenarios in oto-rhino-laryngology

Fig. 20.2 Nasal septum is grossly deviated towards the right side

4- Nasal patency test: It was almost absent on the right side. 5- Any other active bleeding site, clot or mucosal tears in the nasal cavity. 6- Other injuries around the nose, eyes and face. There are few ecchymosis below left eye and on the face (fig 20.1). Investigations: 1- X-ray nasal bone (lateral view): This X-ray has a great medico-legal importance especially in cases of assault. In this case multiple fractures were present in nasal bones of both sides (fig. 20.3). 2- X-ray PNS (Water’s view): This view is important to assess condition of the nasal septum especially bony septum and the sinuses. 3- CT scan: It is helpful in cases where multiple bone fractures and soft tissue injuries of the head and face are present. It was not done in this case. 4- Baseline investigations for general anaesthesia like blood CP, prothrombin time, activated partial thromboplastin time and urine D/R etc. Diagnosis: This was a case of class II nasal bone fracture with fracture and deviation of the nasal septum.

Fig. 20.3 X-ray nasal bone (lateral view) showing fracture of nasal bones

63

Clinical scenarios in oto-rhino-laryngology

Treatment: The patient was planned for surgery of correction of nasal bone fracture along with correction of the nasal septum (septo-rhinoplasty) under general anaesthesia. Displaced nasal bone fracture of short duration (like five days in this case) can be corrected by manipulation with the Walsham’s forceps. When the patient comes late after healing of the displaced fracture, osteotomies are required for correction of nasal deformity. In this case correction of the displaced nasal bone was done with Walsham’s forceps and for septal deviation septoplasty was done (fig 20.4).

Fig. 20.4 Post-operative photograph of the patient after three weeks

Discussion: Fracture of the nasal bone is very common and it is often associated with fractures of other bones of the face and base of skull. It is classified into following three types: 1- Class I fracture: In this type distal thin part of the nasal bone is fractured, which may be depressed or displaced. This type of fracture occurs because of trauma from the front and is associated with vertical fracture of the nasal septum. 2- Class II fracture: In this type along with the fracture of nasal bone, fracture of the frontal process of maxilla also occurs. This type of fracture is caused by medium velocity trauma from the lateral side. It is associated with horizontal or ‘C’ shaped fracture of the nasal septum along with perpendicular plate of ethmoid. 3- Class III fracture: In this type there is also involvement and fracture of the ethmoid labyrinth. There is marked depression of the nasal bones which are pushed under the frontal bones and there is an apparent widening of space between the two eyes (telecanthus).

64

Clinical scenarios in oto-rhino-laryngology

TEST YOURSELF

Read the clinical scenario given at the beginning and answers the following questions

1234-

What are the important points in history taking and clinical examination in this case? How will you investigate this patient? How will you manage this patient? What are the different types of nasal bone fracture?

65

Clinical scenarios in oto-rhino-laryngology

Case 21 Clinical Scenario A 36 years old male patient presented with the complaints of nasal obstruction which was mainly on the left side for last one year. It was often associated with left sided facial pain, frontal headache and thick nasal discharge. For these complaints he repeatedly visited his family physician who gave him medications and his symptoms were relieved upto some extent for a short time period. Anterior rhinoscopy showed a smooth, soft and pale mass in the left nasal cavity (fig. 21.1). Important points in history taking: 1- Detailed history about the nasal obstruction including onset, duration, progression, unilateral or bilateral, continuous or intermittent, aggravating and relieving factors etc. In this case the onset was insidious. Nasal obstruction was almost continuous, mostly on the left side and progressive with no specific relieving or aggravating factors. 2- Any history of blood stained discharge or epistaxis. In this case there was no such history. 3- Any history of nasal allergy or asthma like excessive sneezing, watery rhinorrhoea, dyspnoea etc. In this case no such history. 4- Watering of the eyes. In this case there was occasional watering from the left eye. 5- Any history of previous nasal surgery. In this case there was no history of previous surgery. Important points in clinical examination: 1- Anterior rhinoscopy. It revealed a mass in the left nasal cavity, completely filling it (fig. 21.1). There was a slight deviated nasal septum on the right side.

Fig. 21.1 Patient with mass in the left nasal cavity

66

Clinical scenarios in oto-rhino-laryngology

2- Probe test: It showed that the mass was soft, mobile, polypoidal, insensitive to touch and did not bled on touch. 3- Nasal patency test: It was almost absent on the left side. 4- Posterior rhinoscopy. It was difficult to perform posterior rhinoscopy on this patient but the mass was not visible. Investigations: 1- X-ray PNS (Water’s view). It showed haziness or opacification in the left maxillary sinus with soft tissue in the left nasal cavity (fig. 21.2). 2- CT scan of nose and PNS in both axial and coronal views. It showed a soft tissue mass arising from the left maxillary sinus and involving left nasal cavity and nasopharynx (fig. 21.3 and 21.4). The other paranasal sinuses were clear. 3- Baseline investigations for general anaesthesia like blood CP, prothrombin time, activated partial thromboplastin time and urine D/R etc. All were within normal limits. Diagnosis: This was a case of antro-choanal polyp involving left maxillary sinus, nasal cavity and nasopharynx.

Fig. 21.2 X-ray PNS (Water’s view) showing haziness in left maxillary sinus and nasal cavity

Fig. 21.3 CT scan (axial view) showing polyp in left maxillary sinus, nasal cavity and nasopharynx

67

Clinical scenarios in oto-rhino-laryngology

Fig. 21.4 CT scan (coronal view) showing polyp in left maxillary sinus and nasal cavity

Differential Diagnosis: 1- Inverted papilloma: It is more common in old age patient and appears more solid or firm than a polyp. 2- Hypertrophied inferior turbinate. Grossly enlarged inferior turbinate may have the appearance similar to a polyp. Probe test can differentiate a polyp with the enlarged inferior turbinate. Treatment: The patient was planned for surgery and offered the options of conventional intranasal polypectomy or functional endoscopic sinus surgery. Patient preferred the option of conventional intranasal polypectomy because of cost. Under general anaesthesia complete removal of the polyp was done. Antral part was also removed intranasally (fig 21.5). Post-operatively nasal cavity was packed for 48 hours with ribbon gauze soaked in BIPP. Removed polyp was sent for histopathology which confirmed the diagnosis. Discussion: Antro-choanal polyp arises from the maxillary antrum and prolapsed through the ostium of the sinus in the middle meatus. Initially it hangs in the nasal cavity and grows towards the choana thus it has three parts, antral,

Fig. 21.5 Antro-choanal polyp after removal

68

Clinical scenarios in oto-rhino-laryngology

choanal and nasal. The antral part is present in the maxillary antrum and attached with the other two parts through a thin stalk. The choanal part of the polyp may be seen in the oropharynx where it pushes the soft palate downward and forward. The aetiology of antro-choanal polyp is exactly unknown but it is supposed to be due to sinus infection. Antro-choanal polypi are much less common than the ethmoidal polypi. They are more common in males and can occur at any age but mostly before 40 years. It is mostly unilateral but rarely may be bilateral. Histologically polyp is covered with normal respiratory epithelium. The submucosa is grossly oedematous and similar in appearance with the ethmoidal polyp except that there is no eosinophilia.

TEST YOURSELF

Read the clinical scenario given at the beginning and answers the following questions 123456-

What is the most likely diagnosis in this case? What are the differential diagnosis in this case? What are the important points in history taking and clinical examination in this case? How will you investigate this patient? How will you manage this patients? What are nasal polyp and what are its types?

69

Clinical scenarios in oto-rhino-laryngology

Case 22 Clinical Scenario A 42 years old female patient came in OPD with complaints of swelling over the nose with pain and bilateral nasal obstruction for one day (fig 22.1). She also had history of trauma over her face and nose by a child’s head. Local examination of the nose showed bilateral bulging on the nasal septum, which was soft, fluctuant and tender (fig 22.2).

Fig. 22.1 Patient showing swelling over the nose

Fig. 22.2 Bilateral bulging on the nasal septum

Important points in history taking: 1- Detailed history regarding the nature and severity of trauma. 2- Detailed history regarding the pain. In this case pain was mild to moderate and more on touching the nose. It was continuous and localized with no other specific aggravating or relieving factors. 3- Any history of bleeding or clotting disorders. In this patient there was 70

Clinical scenarios in oto-rhino-laryngology

no such history. 4- History of fever. There was no history of fever. 5- History of any nasal problem before the trauma. In this case there was no such previous history. Important points in clinical examination: 1- General physical examination with vital signs especially temperature. There was no fever. 2- Examination and palpation of the external nose. Swelling was tender. Nasal bone palpation was normal. 3- Examination of the nasal cavity by elevating tip of the nose. It showed a bilateral, reddish, swelling or bulge on the nasal septum. 4- Palpation of the swelling with probe (probe test). The swelling was soft and fluctuant. 5- Anterior rhinoscopy. It was not possible to perform anterior rhinoscopy because of swelling and tenderness. 6- Examination of the eyes. There was no significant positive finding. 7- Examination of the throat especially for post-nasal bleeding. Throat was clear. 8- Palpation of the draining lymph nodes. No palpable lymph node found. Diagnosis: The most probable diagnosis in this case was septal haematoma. Differential Diagnosis: 1- Septal abscess: Sometimes clinically distinction between the septal haematoma and abscess is not evident clearly. Septal abscess is more painful and tender and fever is usually present. The colour of well formed abscess may be pale as compared to haematoma which is usually red or bluish. Tender and palpable draining lymph nodes suggest septal abscess. 2- Associated nasal bone fracture: Because of trauma to the nose fracture of the nasal bone may be present along with septal haematoma. 3- Nasal boil with surrounding cellulitis. Investigations: 1- Blood complete picture. It was within normal limits. 2- Investigations for bleeding or clotting disorders like bleeding time, clotting time, prothrombin time and activated partial thromboplastin time. All were within normal limits. 3- Plain X-ray nasal bone (lateral view) to exclude nasal bone fracture. Treatment: The patient was admitted in hospital and planned for incision and drainage under local anaesthesia. 4% xylocaine pack was applied in the nasal cavity for 15 to 20 minutes. After all aseptic measures incision was given on one side of the swelling, clotted blood came out, all blood and necrotic tissues 71

Clinical scenarios in oto-rhino-laryngology

were evacuated. A corrugated drain was put and both nasal cavities were packed with polyfax soaked ribbon gauze. Prophylactic antibiotic was given post-operatively and the pack was removed after 48 hours. Discussion: Septal haematoma is collection of the blood under muco-perichondrium or muco-periosteum of the nasal septum. In most of the cases it results from trauma to the nasal septum but sometimes it may occur spontaneously. The trauma to the nasal septum may be accidental e.g. road traffic accident, blow or fall on the nose and surgical trauma e.g. after SMR and septoplasty operation. The spontaneous haematoma formation may occur in bleeding disorders like haemophilia, purpura, leukaemia etc. Organization of the septal haematoma may occur if it is not drained, leading to permanent thickening of the nasal septum. Infection usually follows the septal haematoma leading to septal abscess formation. Larger haematoma and septal abscess causes necrosis of the septal cartilage because nutrition of the cartilage is affected due to the separation of muco-perichondrium. Necrosis of the septal cartilage may result in supra tip depression or saddle nose deformity.

TEST YOURSELF

Read the clinical scenario given at the beginning and answers the following questions 123456-

What is the likely diagnosis in this case? What are the differential diagnosis in this case? How will you manage this case? What is septal haematoma? What are the causes of septal haematoma? What are the complications of septal haematoma if it is not treated appropriately?

72

Clinical scenarios in oto-rhino-laryngology

Case 23 Clinical Scenario A 28 years old female patient came with the complaints of bilateral nasal obstruction, excessive sneezing and watery rhinorrhoea for past 8 to 10 years. Now nasal obstruction has increased markedly to became almost continuous and she can not breath through her nose. On clinical examination of the nose, pale, multiple and bilateral polypi were present in the nasal cavities (fig. 23.1).

Fig. 23.1 Patient with multiple polypi in both nasal cavities

Important points in history taking: 1- Detailed history about the nasal obstruction including onset, duration, progression, unilateral or bilateral, continuous or intermittent, aggravating and relieving factors etc. In this case the onset was insidious. Nasal obstruction was almost continuous, bilateral and progressive with no specific relieving or aggravating factors. 2- Any history of nasal allergy or asthma like excessive sneezing, watery rhinorrhoea, dyspnoea etc. All were present. Patient was allergic to multiple things but allergy was severe with house dust. 3- Watering of the eyes. In this case there was watering from both the eyes. 4- Any history of previous nasal surgery. In this case there was no history of previous surgery. 5- About vision and other eye problems. There was no other complaint related with the eyes except watering of eyes and vision was normal. 6- Any history of blood stained discharge or epistaxis. In this case there was no such history. Important points in clinical examination: 1- External examination of the nose, face and eyes. There was no positive findings on examination of these structures. 73

Clinical scenarios in oto-rhino-laryngology

1- Anterior rhinoscopy. It revealed multiple, pale, smooth and shiny grape like polypi completely filling both nasal cavities (fig. 23.1). 2- Probe test: It showed that the polypi were soft, mobile, insensitive to touch and did not bleed on touch. 3- Nasal patency test: It was almost absent on both the side. 4- Posterior rhinoscopy. Nasopharynx was clear on posterior rhinoscopy. 5- Examination of the eyes for vision, proptosis and intercanthal distance. All were within normal limits. Investigations: 1- CT scan of nose and PNS in both axial and coronal views. It showed presence of polypi in both nasal cavities with involvement of both maxillary sinuses, all ethmoidal air cells and sphenoid sinus (fig. 23.2 and 23.3). 2- Investigations for allergy including peripheral eosinophil count and total serum IgE level, both were increased. 3- Baseline investigations for general anaesthesia like blood CP, prothrombin time, activated partial thromboplastin time and urine D/R etc. All were within normal limits.

Fig. 23.2 CT scan nose and paranasal sinuses (axial view)

Fig. 23.3 CT scan nose and paranasal sinuses (coronal view)

74

Clinical scenarios in oto-rhino-laryngology

Diagnosis: This was a case of bilateral ethmoidal nasal polypi. Differential Diagnosis: 1- Allergic fungal sinusitis. In this condition there is characteristic presence of allergic mucin and double density shadows on CT scan. Treatment: The patient was planned for functional endoscopic sinus surgery under general anaesthesia. Clearance of the polypi was done from all the sinuses and nasal cavity using microdebrider. Removed polypi pieces were sent in two separate bottles (one in formalin and other in normal saline) for both histopathology and fungal smear and culture. Histopathology confirmed the diagnosis of nasal polypi while fungal smear and culture did not showed presence/growth of fungus. Discussion: Nasal polyp is a pedunculated swelling arising from the sinus and nasal mucosa. Two distinctive varieties are described: 1- Ethmoidal polypi 2- Antro-choanal polyp Ethmoidal polypi are more common and occurs in more than 70% of cases. Although it is a disease of the ethmoidal sinuses, mucosal changes extend further into the nose and other paranasal sinuses. The maxillary sinuses are affected more than the frontal and sphenoid sinuses. The polyp may arise from uncinate process, bulla ethmoidalis, ostium of the sinuses and medial surface of the middle turbinate. Ethmoidal polypi are mostly multiple, bilateral, pale and whitish in colour. On naked eye examination they are soft, smooth and grape like structures, which moves on probing and insensitive to touch. On microscopic examination it is covered with ciliated columnar epithelium and submucosa shows large intercellular spaces filled with serous fluid. There is marked infiltration of eosinophils and round cells. The covering epithelium may undergo metaplastic changes to transitional and squamous type, when exposed to atmosphere.

TEST YOURSELF

Read the clinical scenario given at the beginning and answers the following questions 12345-

What is the most likely diagnosis in this case? How will you investigate this patient? How will you manage this patient? What are nasal polyp and what are its types? What are the features of ethmoidal polypi? 75

Clinical scenarios in oto-rhino-laryngology

Case 24 Clinical Scenario A 31 years old male patient had undergone septal surgery 8 months back. After surgery he lost for follow-up as he went to his village far away. He has reported again with the complaint of left sided nasal obstruction for last few months. Nasal obstruction was very marked and constant on left side only. On clinical examination, findings on anterior rhinoscopy are shown in fig. 24.1.

Fig. 24.1 Anterior rhinoscopy showing nasal adhesion between the septum and inferior turbinate

Important points in history taking: 1- Detailed history about the nasal obstruction including onset, duration, progression, unilateral or bilateral, continuous or intermittent, aggravating and relieving factors etc. In this case the onset was insidious. Nasal obstruction was almost continuous, mostly on the left side and progressive with no specific relieving or aggravating factors. 2- Any history of nasal discharge or post-nasal dripping. In this case there was no such history. 3- Any complaint related with the throat or ears. In this case there was no complaint. Important points in clinical examination: 1- External examination of the nose for shape and any pathology like saddle nose deformity, columellar retraction or other external deformity. 2- Anterior rhinoscopy. It revealed adhesion formation between the nasal septum and anterior end of inferior turbinate on the left side (fig. 24.1). 3- Nasal patency test. It was almost absent on the left side and normal on the right side. 4- Posterior rhinoscopy. It was within normal limits. 5- Examination of the throat for any pathology like post-nasal dripping and signs of inflammation. 76

Clinical scenarios in oto-rhino-laryngology

Diagnosis: This was a case of nasal adhesion formation between the nasal septum and anterior end of the inferior turbinate after septal surgery. Investigations: Baseline investigations for general anaesthesia like blood complete picture, prothrombin time, activated partial thromboplastin time and urine D/R etc. All were within normal limits. Treatment: The patient was planned for excision of the nasal adhesion. Patient was not willing for procedure to be done under local anaesthesia, so surgery was planned under general anaesthesia. Fibrous tissues were cut with diathermy completely. Splint was applied to prevent further adhesion formation and nasal cavity was packed for 24 hours. Splint was removed after 14 days. Patient was symptom free after that and was followed up for some time. Discussion: The causes of nasal obstruction after septal surgery can be either due to persistent septal deviation or due to complication like nasal adhesions, septal perforation etc. Nasal adhesions are fibrous bands that are formed between the nasal septum and lateral wall of the nose especially inferior turbinate. Chances of adhesion formation are increased when surgery on the lateral wall is also performed with septoplasty or SMR, like turbinectomy, sub-mucous diathermy or electric cautery of the inferior turbinate. Nasal adhesion is treated by excision of the fibrous band and keeping splint between the nasal septum and inferior turbinate for some time, so that healing of the overlying mucosa is complete.

TEST YOURSELF

Read the clinical scenario given at the beginning and answers the following questions

1234-

What is your diagnosis in this case? How will you manage this patients? How the nasal adhesions are formed after septal surgery? How can adhesion formation be prevented after septal surgery?

77

Clinical scenarios in oto-rhino-laryngology

Case 25 Clinical Scenario A 28 years old female patient came in OPD with the complaints of excessive sneezing, watery rhinorrhoea and nasal obstruction off and on for last 10 to 12 years. On clinical examination, findings on anterior rhinoscopy are shown in fig. 25.1.

Fig. 25.1 Anterior rhinoscopy showing pale and enlarged inferior turbinate

Important points in history taking: 1- Onset of symptoms. It was insidious in onset and she did not know when she was completely alright. 2- Frequency and severity of symptoms. According to her, frequency and severity of all symptoms was mild initially but it increased gradually with time. Then sneezing and rhinorrhoea occurred almost daily throughout the year. There was no seasonal variation. 3- Aggravating and relieving factors. Sneezing and rhinorrhoea started whenever she had exposure to dust, smoke and sometime perfumes. Symptoms usually improved whenever she took anti-allergy drugs by herself but relief was only temporary and started again after stopping the medicine. 4- Detailed history about the nasal obstruction. Initially it was mild, intermittent and usually on one side at one time but then it increased to became almost continuous and one nose was alternatively blocked all the time. 5- Any symptom related with the eyes. She also had irritation and watering from both eyes off and on. 6- Any history of skin or other allergy. There was no such history. 7- Any history of asthma or aspirin sensitivity. There was no such history. Important points in clinical examination: 1- Anterior rhinoscopy. The findings are shown in fig. 25.1. Nasal mucosa 78

Clinical scenarios in oto-rhino-laryngology

2345-

was oedematous and pale with hypertrophy of inferior turbinates on both sides. Thin watery secretions were also present in the nasal cavity. Nasal patency test. It was markedly reduced on both the sides. Posterior rhinoscopy. It was within normal limits. Probe test to differentiate between enlarged inferior turbinate and a polyp. Examination of the throat for any pathology. It was within normal limits.

Investigations: 1- Blood complete picture, showed raised eosinophil count of 12% in the peripheral blood. 2- Total serum IgE level. It was markedly raised. 3- X-ray PNS (Water’s view). It showed mucosal thickening in the maxillary sinuses and enlarged inferior turbinates on both sides (fig. 25.2). 4- Skin tests with different allergens. Patient was found to be allergic with multiple allergens. 5- Other baseline investigations for general anaesthesia. All were within normal limits.

Fig. 25.2 Plain X-ray PNS (Water’s view) showing soft tissues of enlarged inferior turbinates and mucosal thickening in maxillary sinuses

Diagnosis: This was a case of allergic rhinitis with enlarged inferior turbinates. Treatment: The patient was planned for reducing the size of inferior turbinate by using microdebrider under general anaesthesia. Xylocaine with adrenaline was injected in the inferior turbinate. A small submucosal pocket was formed by pushing the microdebrider at the anterior end of inferior turbinate. Submucosal tissues were removed by microdebrider keeping the overlying mucosa intact. Post-operatively nasal cavity was packed for 24 hours. Good nasal patency was achieved after surgery. Patient was advised to keep her nasal allergy in control by taking medicines and regular follow-up. Postoperatively following medicines were prescribed: 1- Oral fexofenadine 120 mg once daily. 79

Clinical scenarios in oto-rhino-laryngology

2- Oral montelukast 10 mg once daily. 3- Topical beclomethasone spray twice daily. 4- Prophylactic oral antibiotic for 7 days. Discussion: Allergy is an abnormal reaction of the tissue to certain substances. It is mediated by immunoglobulin E (IgE) and is classified as a type-I hypersensitivity reaction. Two clinical forms of allergic rhinitis are well known, ‘seasonal’ and ‘non-seasonal or perennial’. In seasonal form, symptoms appear only during specific season in the specific areas of the world for e.g. in pollen season. In non-seasonal or perennial form the symptoms are usually not very marked as in the seasonal variety but remains throughout the year. Typically in allergic rhinitis, nasal mucosa appears to be pale, swollen, and sometimes bluish in colour. Nasal cavity may be seen full of watery and thin secretions and inferior turbinates are swollen and hypertrophied. Sometimes nasal polypi may be present. Hypertrophied inferior turbinate size can be reduced surgically by the use of: 1- Submucosal diathermy 2- Electric cautery 3- CO2 Laser 4- Submucosal resection by microdebrider 5- Coblation

TEST YOURSELF

Read the clinical scenario given at the beginning and answers the following questions

1234-

What is your diagnosis in this case? How will you manage this patient? What is the pathophysiology of nasal allergy? What are different surgical options available for reducing the size of inferior turbinate?

80

Clinical scenarios in oto-rhino-laryngology

Case 26 Clinical Scenario A father brought his 4 years old son with the complaint that he had inserted something in his nose during playing about 4 hours back. After insertion he was complaining of pain in his nose along with some blood stained discharge from his right nostril. Child was very anxious and non-cooperative, it was not possible to perform anterior rhinoscopy properly and foreign body was not visible on clinical examination. Important points in history taking: 1- Inquire about nature of the foreign body, whether it was vegetative or non-vegetative, metallic or non-metallic, smooth or sharp, rounded or irregular etc. In this case nature of foreign body was not known. 2- Duration of foreign body insertion. It was inserted about 4 hours back. 3- Any attempt of removal by family member or family doctor. Unskilled attempt for its removal may cause further pushing of foreign body deeper and trauma to the surrounding structures. In this case there was no attempt for removal. 4- Any bleeding from the nose. Only blood stained discharge was present. 5- Pain in the nose. Patient was complaining of pain in the nose. Important points in clinical examination: 1- General physical examination. Child was very anxious and crying. His vital signs were within normal limits. 2- External examination of the nose, showed slight blood stained discharge coming out from his right nostril. 3- Anterior rhinoscopy. It was not possible to perform anterior rhinoscopy and assess the foreign body, its nature and site of impaction. Investigations: 1- Plain X-ray nasal cavity (lateral view). It showed a rounded, radiopaque foreign body in the nasal cavity (fig. 26.1). Diagnosis: This was a case of foreign body in right nasal cavity. Treatment: Patient was admitted for removal of foreign body under general anaesthesia, as he was crying and very anxious, even not allowing proper examination. Under general anaesthesia foreign body was removed by passing a probe beyond the foreign body and pulling it out (fig. 26.2). The foreign body was a lithium battery (fig. 26.3). 81

Clinical scenarios in oto-rhino-laryngology

Fig. 26.1 X-ray nasal cavity (lateral view) showing a rounded radiopaque foreign body

Fig. 26.2 Method for removal of rounded foreign body from the nose

Fig. 26.3 Removed foreign body from the nose, a lithium battery

Discussion: Foreign bodies in the nose are much more common in children and mentally retarded persons. It may enter through one of the following routes: 1- Through the anterior nares 2- Through the posterior nares 3- Through penetration of its walls 82

Clinical scenarios in oto-rhino-laryngology

Most of the foreign bodies are impacted near the floor between the nasal septum and inferior turbinate. Plain X-ray of the nose in lateral and antero-posterior view may show the presence of radiopaque foreign body. Soft tissue shadow may be seen in cases of radiolucent foreign body. If the foreign body is not removed and remains unnoticed for few days it will give rise to local reaction, oedema with superadded infection. This local reaction occurs much earlier and more intense in cases of lithium battery because of leakage of chemicals. It may cause ulceration in the nasal mucosa and necrosis of the nasal septum. Calcium deposition will occur over the foreign body leading to rhinolith formation. Rarely foreign body of the nose may slip into the nasopharynx spontaneously and gets impacted in aerodigestive tract lower down.

TEST YOURSELF

Read the clinical scenario given at the beginning and answers the following questions

1234-

How will you manage this case? What are the different routes for entry of foreign body in the nose? What is the commonest site of foreign body impaction in the nasal cavity? What complications can arise if the foreign body nose is not removed?

83

Clinical scenarios in oto-rhino-laryngology

Case 27 Clinical Scenario A 30 years old female patient came in the OPD with complaint of pain and swelling at right nostril for one day. When she woke up in the morning a day before, she had mild pain in the right nostril and she noticed some redness there but soon pain increased and she had severe pain along with marked redness and swelling. Examination findings of the nose at the time of presentation are shown in fig. 27.1.

Fig. 27.1 Patient with swelling and redness at the right nostril

Important points in history taking: 1- Any previous history of similar swelling on the nose. There was no such history. 2- History of fever. There was no history of fever. 3- History of diabetes mellitus or other immuno-compromised states. In this case no such history. Important points in clinical examination: 1- External examination of the nose. It showed a small, localized swelling present near the columella on right nostril (fig. 27.1). Swelling was red in colour along with oedema and redness of the surrounding skin. A small opening was present over its surface and thick pus was coming out from the opening. It was very tender. 2- Anterior rhinoscopy. Difficult to perform on the right side but it showed redness and oedema of the surrounding nasal mucosa. Left nasal cavity was normal. Diagnosis: The diagnosis of this case was ‘Boil in the nose’. 84

Clinical scenarios in oto-rhino-laryngology

Investigations: No investigation was done in this case. Treatment: This was a case of a boil in the nose with spontaneous rupture. Pus was thick and the opening was small, so the opening was made larger with an artery forceps and all the pus drained out. Pus was sent for culture and sensitivity. Antibiotic against staphylococcus aureus was given orally (amoxicillin with clavulanic acid) along with analgesics. Pus C/S report showed heavy growth of staphylococcus aureus. The organisms were found to be sensitive to amoxicillin with clavulanic acid, so the same antibiotic was continued for 7 days. The patient’s recovery was uneventful. Discussion: Nasal boil or furuncle is the staphylococcal infection of a hair follicle. In the nose hair follicles are present in the vestibule of the nasal cavity. Nasal vestibule is present in the dangerous area of the face and its inflammation may spread to cause intracranial complications. Boil in the nose is usually single but multiple boils may occur. Like boil in the ear, nasal boil is also very painful as skin of the nasal vestibule is tightly adherent to the underline structures. Recurrence of nasal boil is common. The predisposing factors are diabetes mellitus, general debilitating diseases, scratching and nose picking.

TEST YOURSELF

Read the clinical scenario given at the beginning and answers the following questions 1234-

What is the most likely diagnosis in this case? How will you manage this case? What are the predisposing factors of nasal boil? Why nasal boil can cause intracranial extension of infection?

85

Clinical scenarios in oto-rhino-laryngology

Case 28 Clinical Scenario A 47 years old male patient came in emergency department with complaint of severe bleeding from the nose, which started suddenly half an hour back while he was in his office. He tried to stop bleeding by pinching his nose and applying ice packs on forehead and nose, but all measures failed. Immediate management: 1- Examination of the nose showed profuse bleeding from the right nostril. 2- Anterior nasal packing was done immediately with a ribbon gauze soaked with antibiotic ointment (polyfax) and xylocaine jelly in both the nasal cavities. 3- Bleeding stopped after anterior nasal packing. 4- Throat was examined for any posterior bleeding. There was no posterior bleeding and throat was clear after nasal packing. 5- Vital signs were checked. His pulse was 90/minute, blood pressure was 180/110 mm. of Hg., respiratory rate was 20/minute and temperature was 98.8oF. 6- Sublingual anti-hypertensive agent (captopril 25 mg.) was given immediately to lower blood pressure. 7- Intravenous line was maintained with 5% dextrose water. 8- Prophylactic antibiotic was started parenterally. 9- Intravenous tranexamic acid was started. 10- Oral analgesics and anxiolytic agents were started. 11- Patient was admitted for further management. Important points in history taking: 1- Any history of epistaxis before. According to the patient he had previous history of recurrent epistaxis but it was always mild and stopped spontaneously by pinching of the nose for 10 to 15 minutes. 2- Any history of bleeding from other site. There was no such history. 3- Any history of bleeding or clotting disorder. There was no such history. 4- Any history of fever. There was no history of fever. 5- Any history of trauma to nose. There was no such history. 6- History of hypertension. He was a known hypertensive patient for last 7 to 8 years and has been taking anti-hypertensive drugs irregularly. Blood pressure often remained elevated and at such time bleeding also occurred from the nose. 7- History of using of anticoagulant or platelet aggregation inhibitor drugs like aspirin. There was no such history.

86

Clinical scenario in oto-rhino-laryngology

Important points in clinical examination: 1- General physical examination. The patient was a middle aged person of obese built and average height. He was fully oriented in time, space and person. 2- Assess the amount of blood loss and check for any signs of shock, hypovolumia or anaemia. Investigations: 1- Blood complete picture with platelet count. Haemoglobin was 11.2 gm/dl. while all other were within normal limits. 2- Bleeding and clotting profile. All were within normal limits. 3- Tests for blood sugar, liver and renal functions. All were within normal limits. Diagnosis: This was a case of severe epistaxis, most probably due to hypertension. Further management: 1- Patient was shifted to the ward. 2- Vital signs monitoring including blood pressure. His blood pressure became stable after few hours. 3- Physician’s opinion for control of hypertension. 4- Anterior nasal packing was removed after 48 hours in the operation theatre and nasal cavity was examined for any local pathology. There was no bleeding after removal of the pack and no significant finding was noted on examination of the nasal cavities. 5- Patient was kept for observation in the hospital for one day more and discharged next day. Discussion: Epistaxis is a very common event which is often unpleasant for the sufferer and can occasionally be fatal. Epistaxis is mentioned in the medical literature dating back to very early times. Bleeding may occur from any site but in 90% of cases epistaxis occurs from the Little’s area. Little’s area is situated in the antero-inferior part of the nasal septum, where the anastomosis of four blood vessels is present, called ‘Kiesselbach’s plexus’. These arteries include anterior ethmoidal, septal branch of superior labial, sphenopalatine and greater palatine arteries. The other sites of bleeding include above the middle turbinate from the ethmoidal vessels, below the middle turbinate from branches of sphenopalatine artery and the nasopharynx. Venous bleeding may occur especially from the retro-columellar vein which runs vertically just behind the columella. Hypertension is a common cause of bleeding from the nose in middle aged and old persons. The important step in the management of epistaxis due to hypertension is the control of blood pressure. When the blood pressure is properly controlled, epistaxis usually stops by 24 to 48 hours of nasal packing. 87

Clinical scenarios in oto-rhino-laryngology

TEST YOURSELF

Read the clinical scenario given at the beginning and answers the following questions

1234-

How will you manage this case? What is the commonest site for epistaxis? What is Kiesselbach’s plexus? What are the different causes of epistaxis?

88

Clinical scenarios in oto-rhino-laryngology

Case 29 Clinical Scenario A 30 years old male patient came in the OPD with complaints of nasal discharge, post-nasal dripping, facial pain and headache for last 8 to 10 months. He repeatedly took medicines from his family physician for these complaints during this period but there was no relief. On clinical examination, thick muco-purulent secretion was present with hypertrophied inferior turbinates and congested mucosa in both nasal cavities. Important points in history taking: 1- Detailed history regarding nasal discharge and post-nasal dripping. According to the patient he was in his usual state of health about 10 months back when he had an attack of common cold. He took medicines for this, his fever was relieved but the nasal discharge and post-nasal dripping continued. These symptoms were almost continuous but relieved to some extent for a short period after taking medicines. Nasal discharge was thick and yellowish to green in colour. 2- Detailed history regarding headache and facial pain. These symptoms also started 10 months back along with the other complaints. Headache was mainly in the frontal region or sometimes between the eyes. These symptoms were intermittent and relieved by taking medicines but reappeared again after stopping the treatment. 3- Any history of watery rhinorrhoea, excessive sneezing, irritation in the nose or eyes or any other symptom of nasal allergy. There was no previous history of nasal allergy. 4- Any history of nasal obstruction. According to him, he had often nasal obstruction of one nostril or the other, especially on lying in lateral position lower dependent nose was blocked. 5- Any history of repeated sore throat. Because of post-nasal dripping he often had irritation in the throat and he had to clear his throat repeatedly. 6- Drug history. He did not know all the names of medicines which he had used during this period, but he had repeatedly taken different antibiotics, analgesics, decongestants, nasal sprays, steam inhalation etc. 7- Any history of asthma or skin allergies. There was no such history. Important points in clinical examination: 1- General physical examination. There was nothing significant on general physical examination. 2- External examination of the nose, face and eyes. All were normal. 3- Anterior rhinoscopy. It showed a slightly deviated ‘S’ shaped nasal septum. Muco-purulent secretion was present in both nasal cavities with congested mucosa and hypertrophied inferior turbinates. 89

Clinical scenarios in oto-rhino-laryngology

4- Posterior rhinoscopy. It showed presence of post-nasal dripping. 5- Nasal endoscopy. It was done in OPD under local anaesthesia. All the findings of anterior rhinoscopy were confirmed. There was oedema, mucosal thickening and polypoidal mucosa in the area of osteo-meatal complex in the middle meatus. Investigations: 1- CT scan of the nose and PNS (both axial and coronal views). It showed blockage in the osteo-meatal complex on both the sides, mucosal thickening in the ethmoidal air cells and both maxillary sinuses. There was retained secretions in the left maxillary sinus (fig. 29.1 and 29.2). Frontal and sphenoid sinuses were within normal limits. Diagnosis: This was a case of chronic rhino-sinusitis. Treatment: As patient had already taken all the medicines with no relief, so patient was admitted for functional endoscopic sinus surgery under general

Fig. 29.1 CT scan of nose & PNS in coronal view

Fig. 29.2 CT scan of nose & PNS in axial view

90

Clinical scenarios in oto-rhino-laryngology

anaesthesia. All the polypoidal mucosa present in the middle meatus was cleared. Uncinate process was removed and middle meatus antrostomy done. All the ethmoidal air cells were cleared (ethmoidectomy done). Nasal cavity was packed for 24 hours. Post-operatively patient was put on medical treatment and patient showed good improvement in all his symptoms. Discussion: Chronic sinusitis is the chronic inflammation of the paranasal sinuses which lasts for months and sometime years. Acute exacerbations are common and in between the intervals, symptoms may be reduced. Most of the cases of chronic sinusitis are due to failure of the acute infection to resolve. It may follow single or repeated attacks of acute sinusitis. Oedema of the sinus mucosa is present ranging from slight thickening to gross polyposis. Chronic inflammatory cellular infiltration is present with glandular hypertrophy. The surface epithelium may show desquamation, regeneration, ulceration or metaplasia. The organisms are usually mixed and include streptococci, pneumococci, B. proteus and anaerobes.

TEST YOURSELF

Read the clinical scenario given at the beginning and answers the following questions

12345-

What is the most likely diagnosis in this case? What are the important points in history taking and clinical examination in this patient? How will you investigate this patient? How will you manage this patient? Describe the pathology of chronic rhino-sinusitis.

91

Clinical scenarios in oto-rhino-laryngology

Case 30 Clinical Scenario A 14 years old boy came with the complaints of left sided nasal obstruction, recurrent epistaxis and hyposmia for last 6 to 7 months. He had also noticed a mass coming out from his left nostril for last one month with more frequent nasal bleeding (fig. 30.1).

Fig. 30.1 Patient with mass in the left nasal cavity

Important points in history taking: 1- Detailed history regarding his symptoms. According to the patient he was in his usual state of health 6 to 7 months before, when he developed nasal obstruction mainly on the left side. Initially it was intermittent but soon it became continuous. Along with nasal obstruction he had decreased sense of smell and recurrent bleeding from left side of the nose, amount ranging from few drops to profuse bleeding and stopped spontaneously most of the time. Last month he also noticed a mass coming out from his left nostril which was increasing progressively. 2- Any history of nasal packing or admission in the hospital due to epistaxis. He was never admitted in a hospital for nasal packing before. 3- Any history of blood transfusion. There was no such history. 4- Any history of rhinorrhoea, sneezing or nasal allergy. There was no such history. 5- Any complaint related with the eyes. There was no complaint. 4- Any complaint related with the ear. According to the patient he had noticed some blockage and occasional discomfort in his left ear. Important points in clinical examination: 1- General physical examination. Patient was a young adolescent boy of average height and built, fully conscious and oriented. His vital signs 92

Clinical scenarios in oto-rhino-laryngology

2-

2345-

were within normal limits. He was slightly anaemic with no other positive finding on general physical examination. Examination of the nose. It revealed a mass coming out from the left nasal cavity, completely filling it and bleeding on touch (fig. 30.1). The outer surface of the mass was irregular, ulcerated and red. There was deviation of the nasal septum towards the right side due to the mass. Probe test. It showed that the mass was soft to firm in consistency, mobile, polypoidal, insensitive to touch and was bleeding on touch. Nasal patency test. It was absent on the left side and decreased on the right side. Posterior rhinoscopy. It was difficult to perform posterior rhinoscopy on this patient but a similar mass was visible in the nasopharynx. Examination of the eyes. There was no apparent proptosis, displacement, or telecanthus. Eye movements and vision were also normal.

Differential Diagnosis: Following conditions must be considered in the differential diagnosis: 1- Nasopharyngeal angiofibroma. 2- Inverted papilloma. It is more common in old age patient. 3- Antrochoanal polyp. It is smooth, soft, pale and glistening. 4- Fungal rhino-sinusitis. Investigations: 1- CT scan of nose and PNS in both axial and coronal views with contarst. It showed a soft tissue mass filling the nasopharynx, left nasal cavity, left maxillary sinus and anterior ethmoid air cell. It was pushing the nasal septum on the right with remoulding of the surrounding bones. It appeared to be vascular on contrast media (fig. 30.2 and 30.3). 2- Angiography with embolization. Four vessels angiography was performed which showed that the mass was very vascular and recieving its blood supply from the branches of left external carotid artery. Embolization of the feeding vessels were done with gelfoam (fig. 30.4). 3- Baseline investigations for general anaesthesia like blood CP, prothrombin

Fig. 30.2 CT scan of nose & PNS in coronal view

93

Clinical scenarios in oto-rhino-laryngology

Fig. 30.3 CT scan of the nose & PNS in axial view

Fig. 30.4 Angiography of the left external carotid artery before and after embolization

time (PT), activated partial thromboplastin time (APTT) and urine D/R etc. All were within normal limits except his haemoglobin was 9.2 gm/dl. He was transfused with two packed cells unit preoperatively. Diagnosis: This was a case of nasopharyngeal angiofibroma involving nasopharynx, left nasal cavity, left maxillary sinus and left anterior ethmoidal air cells. Treatment: The patient was planned for surgical removal of the angiofibroma. Angiography with embolization was already done one day before surgery. A combined lateral rhinotomy and sub-labial transantral approach was planned (fig. 30.5). Complete excision of the tumour was done with this approach. Tumour removed was sent for histopathology which confirmed the diagnosis of nasopharyngeal angiofibroma. Post-operative recovery was uneventful and during follow-up patient was alright with no recurrence. Discussion: Nasopharyngeal angiofibroma is a benign but locally aggressive 94

Clinical scenarios in oto-rhino-laryngology

Fig. 21.5 Tumour visible through sub-labial transantral approach

tumour. It is the commonest benign tumour of the nasopharynx. It occurs mostly in adolescent males so it is also called juvenile angiofibroma. The usual age of onset is in the second decade. It occurs almost exclusively in males. The exact aetiology of the tumour is unknown but it is thought to be hormonal dependent (male sex hormones). Histologically angiofibroma is composed of fibrous connective tissues interspersed with variable proportion of endothelium lined blood spaces. The ratio of fibrous and vascular component may vary. Mostly the vessels are just endothelium lined spaces with no muscle coat. That is why severe bleeding may occur on taking biopsy and surgical removal, as these vessels cannot contract to stop bleeding. The tissue of origin remains unknown, although various theories have been proposed. The site of origin of the tumour is also a matter of dispute. Previously it was thought that it arises from the roof or posterior wall of the nasopharynx. Now it is believed that it arises from the posterior part of lateral wall of the nose close to sphenopalatine foramen. From here the tumour extends into the nasal cavity, nasopharynx, paranasal sinuses and pterygopalatine fossa. Nasopharyngeal angiofibroma is a benign tumour but locally it is invasive and destroys the adjoining structures. From the site of its origin near sphenopalatine foramen the tumour grows in different directions. Anteriorly it extends into the nasal cavity and may invade the paranasal sinuses including maxillary, ethmoid and sphenoid sinuses. Laterally it may extend into the pterygopalatine fossa and from there to infra-temporal fossa and cheek. It may enter into the orbit through inferior and superior orbital fissures. It may extend above into the middle and anterior cranial fossa. In middle cranial fossa it enters through foramen lacerum or through sphenoid sinus. In anterior cranial fossa it enters through ethmoidal sinuses or through cribriform plate.

95

Clinical scenarios in oto-rhino-laryngology

TEST YOURSELF

Read the clinical scenario given at the beginning and answers the following questions 1234567-

What is the most likely diagnosis in this case? What are the differential diagnosis in this case? What are the important points in history taking and clinical examination in this case? How will you investigate this patient? How will you manage this patients? What is the site of origin of nasopharyngeal angiofibroma? Why nasopharyngeal angiofibroma is common in adolescent males?

96

Clinical scenarios in oto-rhino-laryngology

Section III

ORAL CAVITY & PHARYNX Case 31Case 32Case 33Case 34Case 35Case 36Case 37Case 38Case 39-

Chronic tonsillitis Post-tonsillectomy haemorrhage Quinsy Enlarged adenoids Papilloma of the cheek Carcinoma of the cheek Carcinoma of the tongue Foreign body of the throat Ranula

97

99 102 104 107 110 113 117 120 123

Clinical scenarios in oto-rhino-laryngology

Case 31 Clinical Scenario A 23 years old female patient came in OPD with complaints of recurrent attacks of sore throat, odynophagia and fever for the last 10 to 12 years. She used to take medicines from her family physician and symptoms usually relieved within few days. Initially these attacks were very infrequent but with the passage of time, frequency of attacks had increased and it occured every one to two months. Important points in history taking: 1- Detailed history regarding onset, duration, frequency, aggravating and relieving factors of an attack of sore throat. 2- Fever, whether high grade or low grade. In this case fever was mostly in the range of 101 to 102o F. 3- Any associated nasal complaints like nasal obstruction, post-nasal dripping, rhinorrhoea etc. In this case there was no nasal complaint. 4- About gastro-oesophageal reflux disease (GERD). In adults it is one of the common cause of recurrent or chronic sore throat. In this patient there was no history of GERD. 5- About any symptom related with rheumatic fever, glomerulonephritis or sub-acute bacterial endocarditis. 6- History of sleep apnoea or snoring. In this patient there was history of snoring and occasional sleep apnoea. 7- Any history of bleeding or clotting disorder. This is important in patient who is undergoing tonsillectomy. Important points in clinical examination: 1- Assessment of the palatine tonsils and pillars. In this case tonsils were congested, hypertrophied with prominent crypts and congested anterior pillars (fig 31.1).

Fig. 31.1 Patient with congested and hypertrophied tonsils and congested anterior pillars

99

Clinical scenarios in oto-rhino-laryngology

2- Palpation of jugulo-digastric lymph nodes. These were palpable and tender on both the sides in this patient. 3- Anterior and posterior rhinoscopy to rule out any nasal disease. In this patient it was within normal limits. Diagnosis: This was a case of chronic or recurrent tonsillitis with hypertrophied tonsils. Investigations: 1- Blood complete picture with platelet count. It was within normal limits. 2- Bleeding and clotting profile including PT and APTT. 3- Anti-streptolysin O titre (ASO titre). In this case it was increased (300). 4- X-ray chest (PA view) for general anaesthesia. 5- Urine D/R. 6- Hepatitis B and C screening. Treatment: The patient was planned for tonsillectomy under general anaesthesia and admitted in the hospital in morning with overnight nil per orally (NPO). Tonsillectomy was done by diathermy method (fig 31.2) and patient was discharged next day. Post-operatively she was given antibiotic and analgesics for 7 days. Post-operative recovery was uneventful (fig 31.3). Discussion: In chronic tonsillitis there are small micro-abscesses within the core of the tonsils and micro-organisms are present in these abscesses. These abscesses are surrounded by fibrous tissues and antibiotics do not reach within the abscesses in proper concentration. Thus eradication of microorganisms within the core of the tonsil is difficult. One way of eradication is to remove the tonsil itself. That is why tonsillectomy is performed in cases of chronic or recurrent tonsillitis. Three clinical forms of chronic tonsillitis are recognized:

Fig. 31.2 Tonsillectomy by diathermy method in Rose’s position

100

Clinical scenarios in oto-rhino-laryngology

Fig. 31.3 Post-operative photograph of the patient after 5 days of tonsillectomy showing slough in the tonsillar fossa

1- Chronic follicular tonsillitis 2- Chronic parenchymatous tonsillitis 3- Chronic fibroid tonsillitis In cases of chronic or recurrent tonsillitis, tonsillectomy is indicated when there are: 1- Seven attacks of tonsillitis per year for one year. 2- Four to six attacks of tonsillitis per year for two consecutive years. 3- Three attacks of tonsillitis per year for three consecutive years. There are different methods for performing tonsillectomy including: 1- Dissection or cold steel method 2- Diathermy 3- Diode or CO2 laser 4- Harmonic scalpel 5- Coblation 6- Radiofrequency ablation 7- Microdebrider

TEST YOURSELF

Read the clinical scenario given at the beginning and answers the following questions 12345-

What is the most likely diagnosis in this case? How will you manage this case? What investigations are required in a patient undergoing tonsillectomy? What are the indications for tonsillectomy other than chronic or recurrent tonsillitis? What are the different methods for tonsillectomy?

101

Clinical scenarios in oto-rhino-laryngology

Case 32 Clinical Scenario A father brought his 9 years old son in emergency whose tonsillectomy had been performed 5 days back with the complaint that he had vomited out a large amount of blood 1 hour back. After that fresh blood was coming out from the throat. Important points in history taking: 1- Detailed history regarding the amount and nature of vomiting and bleeding. The vomited material contained dark coloured blood and was about half a litre in amount. After that he was continuously spitting fresh blood mixed with saliva at regular interval. 2- Any history of fever. He had a spike of high grade fever last night which was relieved by taking syrup paracetamol. 3- Was he taking antibiotic regularly? According to his father, he was not taking the medicines regularly and he had missed many doses. In addition he was also not using antiseptic mouth wash regularly. Important points in clinical examination: 1- General physical examination. The child was looking lethargic and weak but he was fully oriented in time, space and person. His pulse was 100/minute, blood pressure was 110/70 mm of Hg., respiratory rate was 20/minute and temperature was 100oF. Signs of dehydration were positive showing mild to moderate dehydration. 2- Assess the amount of blood loss and check for any signs of shock, hypovolumia or anaemia. 3- Examination of the oral cavity and throat. It showed a clot present in his right tonsillar fossa. There was no active bleeding. Oral hygiene was very poor. Immediate management: 1- Patient was admitted in hospital and was advised not to take any thing by mouth (NPO) till further orders. 2- Intravenous line was maintained with dextrose saline infusion. 3- Parenteral antibiotic (intravenous ceftrixone) was started according to the body weight. 4- Blood was sent for blood complete picture, prothrombin time (PT) and activated partial thromboplastin time (APTT). Diagnosis: This was a case of secondary post-tonsillectomy haemorrhage. 102

Clinical scenario in oto-rhino-laryngology

Further management: 1- Patient was admitted in the ward for further management and monitoring. 2- Vital signs monitoring. Patient was haemodynamically stable. 3- Maintain good oral hygiene with regular antiseptic gargles and mouth wash. 4- Blood reports showed that his haemoglobin was 10.2gm/dl while PT and APTT were within normal limits. 5- Regular examination of the throat. There was no further bleeding and clot size started to regress and disappeared completely next day. 6- Oral syrup paracetamol started and fever settled down with no further spike. 7- Soft diet allowed from next day. 8- Patient was kept in hospital for 2 days more for observation and parenteral antibiotic and then discharged on oral antibiotic. Discussion: Haemorrhage is the most important complication after tonsillectomy which can be fatal sometimes. It is conventionally divided into, primary, reactionary and secondary haemorrhage. Primary haemorrhage occurs on the operation table after removal of the tonsils. Reactionary haemorrhage occurs within 24 hour after the operation. Secondary haemorrhage occurs after 24 hours till 14 days, when the healing in tonsillar fossa is completed. Usually it occurs on 5th post-operative day or afterwards. The cause of secondary haemorrhage is infection of the tonsillar bed with sloughing and opening of small blood vessels in its bed. The haemorrhage is usually not profuse and is associated with fever. It is treated by giving proper antibiotics, rest, sedation and observation in the hospital. In severe and profuse secondary haemorrhage not responding to medical treatment, fossa is packed and the pillars are stitched together under general anaesthesia and the pack is removed after few days, when the condition is settled.

TEST YOURSELF

Read the clinical scenario given at the beginning and answers the following questions

12345-

How will you manage this case? What are the different types of post-tonsillectomy haemorrhage? What is the cause of secondary post-tonsillectomy haemorrhage? What is the usual time for secondary post-tonsillectomy haemorrhage? What is the treatment of secondary post-tonsillectomy haemorrhage?

103

Clinical scenarios in oto-rhino-laryngology

Case 33 Clinical Scenario A 17 years old male patient came in the OPD with complaints of severe pain in the throat on left side, difficulty in swallowing, high grade fever and restricted mouth opening for last 2 days. Examination findings of his throat and oral cavity are shown in fig. 33.1.

Fig. 33.1 Patient with marked bulging and redness over the soft palate with restricted mouth opening

Important points in history taking: 1- Onset and progression of the symptoms. The onset was sudden two days back. Initially all the symptoms were mild in severity but increased rapidly. 2- Detailed history regarding pain especially unilateral or bilateral. Initially pain was bilateral and mild but soon it became more on the left side. Then it was severe, mainly on the left side, continuous, localized and throbbing in nature. It was aggravated by swallowing and relieved to some extent by taking analgesics. 3- Detailed history about the fever. It was sudden in onset, almost continuous, high grade, not associated with chills and relieved to some extent by taking antipyretic drugs. 4- Detailed history about the odynophagia. There was severe pain on swallowing especially on left side of the throat. Pain was so severe that he was unable to swallow his saliva as well. 5- Detailed history of restricted mouth opening or trismus. There was severe pain on the left side on mouth opening along with trismus. 6- Any history of voice change. Voice has changed to became thick and muffled. 7- Any history of respiratory obstruction or stridor. There was no such history. 8- Any history of earache. There was mild earache on the left side but his 104

Clinical scenarios in oto-rhino-laryngology

hearing was unaffected. 9- Previous history of repeated sore throat and fever. He had recurrent attacks of sore throat with fever since childhood. Usually it occurred after every two to three months. Important points in clinical examination: 1- General physical examination. Patient was a teenager of average built and height, looking very ill and restless. His pulse was 110/minute, blood pressure was 110/75 mm of Hg., respiratory rate was 20/minute and temperature was 102oF. His voice was thick and muffled. Rest of the general physical examination was unremarkable. 2- Examination of the oral cavity and throat. There was restricted mouth opening with bad smell coming out and tongue was coated. There was a bulge involving left side of the soft palate and tonsil which was red and congested (fig. 33.1). The uvula was swollen and pushed towards the right side. 3- Examination of the lymph nodes showed palpable and tender jugulodigastric lymph node on the left side. 4- Examination of the nose and ears. It was within normal limits. Diagnosis: The diagnosis in this case was peri-tonsillar abscess or quinsy. Differential Diagnosis: This condition has to be differentiated from: 1- Parapharyngeal abscess. 2- Neoplasia of the tonsils. 3- Retropharyngeal abscess. Investigations: No pre-operative investigation was required in this case. Treatment: The patient was admitted for incision and drainage. He was very cooperative so incision and drainage was decided to be done under local anaesthesia. 4% xylocaine was used as gargles for local anaesthesia. In sitting position incision was given on the bulging soft palate, at the junction of two imaginary lines as shown in fig 33.2. Thick pus came out which was sent for culture and sensitivity. Parenteral broad spectrum antibiotic was started along with oral anti-inflammatory and analgesic drugs. Pus C/S report showed heavy growth of mixed gram +ve organisms, sensitive to the given antibiotic. The patient’s recovery was uneventful. Discussion: Quinsy or peritonsillar abscess is the collection of pus in the peritonsillar space between the capsule of the tonsil and adjacent lateral pharyngeal wall. 105

Clinical scenarios in oto-rhino-laryngology

Fig. 33.2 Two imaginary line, horizontal along the base of uvula and vertical along the anterior pillar

It usually follows an acute attack of tonsillitis. The route of infection is probably via a crypt, mostly crypta magna. Before development of the abscess, inflammation is set up in the peritonsillar region outside the tonsillar capsule and is called stage of peritonsillitis. As the inflammation increases, frank pus is collected between the tonsillar capsule and superior constrictor muscle. As the route of infection in majority of the cases is crypta magna, the peritonsillar abscess lies mostly near the upper pole of the tonsils. In majority of the cases, abscess is unilateral and most frequently affects young, adult male but may occur at any age. If the peri-tonsillar abscess or quinsy is not treated properly, spontaneous rupture of the abscess can occur leading to aspiration pneumonia and lung abscess. If incision and drainage of the abscess is planned under general anaesthesia, great care should be taken during endotracheal intubation as spontaneous rupture can occur during this procedure.

TEST YOURSELF

Read the clinical scenario given at the beginning and answers the following questions 1234567-

What is the most likely diagnosis in this case? What are the important points in history taking in this patient? What are the important points in clinical examination of this patient? What are the differential diagnosis in this case? How will you manage this case? What is the pathology of peri-tonsillar abscess or quinsy? What complications can occur if quinsy is not treated early and properly?

106

Clinical scenarios in oto-rhino-laryngology

Case 34 Clinical Scenario A mother brought her 8 years old son with the complaints of mouth breathing, snoring, bed wetting and recurrent earache for the last two to three years. All these complaints were increased in severity during the last 6 months. He also had history of recurrent fever off and on.

Fig. 34.1 Patient with open mouth and prominent incisors

Important points in history taking: 1- Detailed history about the symptoms, whether these are continuous or intermittent, its severity, aggravating and relieving factors etc. According to the patient’s mother, these symptoms were developed about three years back and were continuous and progressively increasing. There was no specific aggravating or relieving factor. 2- Ask about the voice change. His voice had a nasal tone in voice. 3- Other ear symptoms like deafness and discharge. He often complained of earache but he had no complaint regarding hearing. 4- Ask about the pain in the throat, odynophagia etc. He had recurrent attacks of sore throat often associated with high grade fever. 5- History about any allergy. There was no history of asthma or any other allergy. Important points in clinical examination: 1- General physical examination. It showed open mouth, prominent incisors and pinched nostrils (fig 34.1). Rest of the general physical examination was unremarkable. 2- Examination of the nose for any pathology like deviated nasal septum, nasal polyp, enlarged turbinates. In this patient nose was congested with presence of muco-purulent secretions in the nose. 107

Clinical scenarios in oto-rhino-laryngology

3- Check for nasal patency. Nasal patency was almost absent on both the sides. 4- Posterior rhinoscopy. It is usually very difficult to perform in a child. Similarly this child also did not co-operate for posterior rhinoscopy. 5- Examination of the oral cavity and oropharynx especially palatine tonsils, faucial pillars. Tonsils were congested with slight enlargement. 6- Examination of the ear especially condition of the tympanic membrane and tuning fork tests and Valsalva’s maneuver. No significant finding was present at the time of examination. 7- Nasal and nasopharyngeal examination with rigid endoscope, if available. 8- Palpation of cervical lymph nodes especially jugulo-digastric lymph nodes, which were not palpable. Diagnosis: The most probable diagnosis was ‘enlarged adenoids’ in this case. Differential Diagnosis: All other cause of nasal obstruction in children should be included in differential diagnosis like deviated nasal septum, nasal polyp, hypertrophied inferior turbinate, foreign body or rhinolith, choanal atresia, etc. Investigations: 1- X-ray soft tissue of the nasopharynx (lateral view). This is the most important investigation for the diagnosis of enlarged adenoids. Nasopharyngeal airway was assessed in this view, which showed grossly enlarged adenoids with only slit like nasopharyngeal airway (Fig. 34.2). 2- Other baseline investigations for general anaesthesia when planned for surgery like blood complete picture, ESR, random blood sugar, urine D/R and X-ray chest (PA view). Treatment: Patient was planned for adenoidectomy under general anaesthesia. Oral endotracheal intubation was done. In Rose’s position Boyle’s Davis

Fig. 34.2 X-ray soft tissue nasopharynx (lateral view) showing enlarged adenoids with airway narrowing

108

Clinical scenarios in oto-rhino-laryngology

mouth gag was applied and digital palpation of the nasopharynx was done and adenoids enlargement was confirmed (fig 34.3). Adenoids removed by adenoid curette.

Fig. 34.3 Patient in Rose’s position with Boyle’s Davis mouth gag applied

Discussion: The adenoids are enlarged and hypertrophied nasopharyngeal tonsils, sufficient to produce symptoms. Nasopharyngeal tonsil is present in the nasopharynx at the junction of its roof and posterior wall. This is composed of vertical ridges of lymphoid tissues, separated by deep cleft and covered by ciliated columnar epithelium. It is present at birth, show physiological enlargement and tends to start atrophy at puberty. Hypertrophy sufficient to produce symptoms occur most commonly between the ages of three to seven years. Inflammatory changes occur in adenoids as a result of infection in it alone or in association with rhinitis and tonsillitis. Allergy of the upper respiratory tract may also contribute to the enlargement of adenoids. The symptoms of adenoids enlargement are produced due to respiratory obstruction and blockage of the eustachian tube. Signs and symptoms depend upon the relative size of the adenoids with that of nasopharynx.

TEST YOURSELF

Read the clinical scenario given at the beginning and answers the following questions 12345-

What is the most likely diagnosis in this case? What are the differential diagnosis in this case? What investigations will you order in this case? How will you treat this case? Briefly outline the steps of adenoidectomy operation.

109

Clinical scenarios in oto-rhino-laryngology

Case 35 Clinical Scenario A 58 years old male patient presented with a growth on the inner surface of his right cheek for last 5 to 6 months (fig 35.1). Initially it was very small but it increased in size rapidly and within this period it became of the present size.

Fig. 35.1 Patient with a growth on the inner surface of right cheek

Important points in history taking: 1- Detailed history regarding the growth. According to the patient, it was present for last 5 to 6 months and he did not know how it started. Initially it was very small and he ignored it. Soon it started to increase in size till its present form (fig. 35.1). There was no aggravating or relieving factor known to him and there was also no associated factor. 2- Any history of pain in the swelling or burning sensation in the mouth. There was no history of pain in the growth or surrounding area and no history of burning sensation in the mouth. 3- Any history of trauma to the cheek especially due to sharp tooth etc. There was no such history. 4- Any history of using beetle nut, pan, tobacco, smoking or alcohol. He used to eat pan with some tobacco but in very less quantity, 2 to 3 pans per day for the last 10 to 12 years. There was no history of consumption of alcohol or smoking. 5- Any history of restricted mouth opening. According to him there was some degree of restricted mouth opening especially after the start of growth. 6- Any swelling in the neck or below mandible. There was no such history. 7- Any history of fever, anorexia or weight loss. There was no such history. Important points in clinical examination: 1- General physical examination. Patient was a middle aged person of 110

Clinical scenarios in oto-rhino-laryngology

2-

345-

average height and slightly obese built, fully oriented in time, space and person. His pulse was 80/minute, blood pressure was 130/85 mm of Hg., respiratory rate was 22/minute and temperature was 98.6oF. Examination of the growth. There was a growth present on the inner surface of the cheek, irregular in shape, 3x4 cms. in size on the right side. The surface was formed by multiple small finger like projections, partly white and partly pink in colour. The margins were irregular but well defined and adjacent mucosa was slightly congested and red. On palpation it was non-tender, firm in consistency with no induration of the surrounding area or the base and there was no bleeding on touch. Examination of the cervical lymph nodes. There was no palpable lymph node in the neck on any side. Examination of rest of the oral cavity and oropharynx. Oral hygiene was satisfactory, teeth were stained with pan and mouth opening was slightly restricted. Examination of the hypopharynx and larynx. It was within normal limits.

Differential Diagnosis: 1- Benign tumours like squamous papilloma, fibroma, haemangioma or pyogenic granuloma etc. 2- Malignant tumours like verrucous carcinoma, squamous cell carcinoma, adnocarcinoma or adenoid cystic carcinoma etc. Investigations: 1- Punch biopsy. It was done under surface anaesthesia, tissues taken from multiple areas of the swelling and sent for histopathological examination. The histopathology report showed fibrous stroma covered with stratified squamous cells with some infiltration by lymphocytes. No features of malignancy were seen. The diagnosis was squamous cell papilloma. 2- Orthopentomogram (OPG). It was within normal limits. 3- Baseline investigations for general anaesthesia. All were within normal limits. Diagnosis: The histopathological diagnosis in this case was squamous cell papilloma. Treatment: The patient was admitted and planned for excision of the swelling under general anaesthesia. Complete excision of the swelling was done with safe margins all around and sent for histopathology. The raw area formed after the excision was around 5x5 cms., it was covered with a partial thickness skin graft (fig. 35.2). The histopathological report of the specimen after surgery confirmed the pre-operative diagnosis. Post-operative recovery of the patient was uneventful. Healing of the surgical area occurred completely with normal mucosa (fig. 35.3). 111

Clinical scenarios in oto-rhino-laryngology

Fig. 35.2 Partial thickness skin grafting after excision of the swelling

Fig. 35.3 Complete healing of the surgical area as seen after 4 weeks of surgery

Discussion: A squamous cell papilloma is a benign tumour of epithelial origin. It usually presents as a warty white or pink swelling with finger like processes. Histologically it consists of stratified squamous cell epithelium supported by a core of vascular connective tissues. It is treated by complete surgical excision. Recurrence after complete surgical excision is very rare.

TEST YOURSELF

Read the clinical scenario given at the beginning and answers the following questions 1234-

What is the most likely diagnosis in this case? What are the important points in history taking in this patient? What are the important points in clinical examination of this patient? How will you manage this case?

112

Clinical scenarios in oto-rhino-laryngology

Case 36 Clinical Scenario A 51 years old male patient came in OPD with complaint of a non-healing ulcer/growth on inner surface of the left cheek for last 3 to 4 months. He took medicines from his family physician but with no relief, rather it was increasing day by day. There was no history of pain and fever but he had lost some weight during this period. Important points in history taking: 1- Detailed history regarding the ulcer/growth. According to the patient, it was present for last 3 to 4 months. Initially it was very small and he ignored it. Soon it started to increase in size till its present form (fig. 36.1). There was no aggravating or relieving factor known to him and there was no associated factor. 2- Any history of pain in the swelling or burning sensation in the mouth. There was no history of pain in the growth or surrounding area but he was complaining of burning sensation in the mouth and he was unable to take spicy food for a long time. 3- Any history of trauma to the cheek especially due to sharp tooth etc. There was no such history. 4- Any history of using beetle nut, pan, tobacco, smoking or alcohol. He used to eat pan with tobacco, about 14 to 15 pans per day for the last 35 years. He also used to smoke 8 to 10 cigarettes per day for about the same period. There was no history of taking alcohol. 5- Any history of restricted mouth opening. According to him there was some degree of restricted mouth opening especially after the start of growth. 6- Any swelling in the neck or below mandible. There was no such history. 7- Any history of fever, anorexia or weight loss. There was no history of fever but he had history of anorexia and weight loss.

Fig. 36.1 Patient with an ulcerative growth on inner surface of the left cheek

113

Clinical scenarios in oto-rhino-laryngology

Important points in clinical examination: 1- General physical examination. Patient was a middle aged person of average height and built, fully oriented in time, space and person. His pulse was 78/minute, blood pressure was 140/85 mm of Hg., respiratory rate was 22/minute and temperature was 98.8oF. 2- Examination of the growth. There was an ulcerative growth present on the inner surface of the cheek, irregular in shape, about 5 cms. in size in its maximum diameter on the left side, reaching upto the lower gingival sulcus. Posteriorly it was reaching upto the posterior limit of the teeth. The surface was irregular with ulceration and the margins were also irregular and ill defined. The surrounding mucosa was showing marked sub-mucous fibrosis and leucoplakia patches. 3- Palpation of the growth. On palpation it was non-tender, firm in consistency with marked induration of the surrounding area and the base. There was no bleeding on touch. 4- Examination of rest of the oral cavity. There was marked trismus due to oral sub-mucous fibrosis. Oral hygiene was poor and the teeth were badly stained with pan. 5- Examination of the cervical lymph nodes. There was no palpable lymph node in the neck on any side. 6- Examination of the nose, hypopharynx and larynx. It was within normal limits. Differential Diagnosis: 1- Squamous cell carcinoma 2- Other malignant tumours like adenocarcinoma, adenoid cystic carcinoma or sarcoma 3- Chronic granulomatous diseases like tuberculosis, syphilis. 4- Pyogenic granuloma 5- Benign tumours Investigations: 1- Punch biopsy. It was done under surface anaesthesia, tissues taken from multiple areas of the swelling and sent for histopathological examination. The histopathology report showed moderately differentiated squamous cell carcinoma. 2- Orthopentomogram (OPG). It was within normal limits with no involvement of the mandible by the disease. 3- Investigations for staging the disease like X-ray chest (PA view), ultrasonography of abdomen. These were within normal limits. 4- CT scan of the neck and oral cavity. It was not done because patient was un-affording. 5 - Baseline investigations for general anaesthesia. All were within normal limits.

114

Clinical scenarios in oto-rhino-laryngology

Diagnosis: The diagnosis in this case was squamous cell carcinoma of the left cheek. The clinical stage of the disease was T3N0M0. Treatment: The patient was admitted and planned for surgery. As the patient had extensive T3 growth with marked mouth opening restriction so intra-oral excision was not possible. Lower lip splitting incision was planned for excision of the growth. Secondly there was no clinically palpable cervical lymph node but the primary tumour was very extensive, so selective supra-omohyoid neck dissection was also planned. Fig. 36.2 shows pre-operative marking for the incision used. Complete excision of the primary tumour was done with a safe margin of 1 cm all around, along with selective supra-omohyoid neck dissection (removal of level I to III lymph nodes). Raw area of the oral cavity was covered with partial split thickness skin graft taken from the thigh. Drain applied and wound closed in layers (fig. 36.3). Specimen removed was sent for histopathology, which re-confirmed the diagnosis of moderately differentiated squamous cell carcinoma. All removed lymph nodes were free from malignancy. Post-operative recovery was uneventful and patient was well and disease free in the follow-up period.

Fig. 36.2 Patient with pre-operative marking for the surgical incision

Fig. 36.3 Patient after completion of the surgery with stitched wound

115

Clinical scenarios in oto-rhino-laryngology

Discussion: Neoplasia of the oral cavity are classified as benign and malignant. Majority of the tumours in the oral cavity are malignant. Among the malignant tumours, squamous cell carcinoma accounts for about 95% of the cases. The rest being salivary gland tumours, lymphoma and melanoma. The incidence of oral squamous cell carcinoma varies worldwide, but its incidence in our region is very high. Aetiological factors responsible for this higher incidence are consumption of pan, chalia (beetle nut) and tobacco in different combination. The other factors are smoking, dietary deficiencies and alcohol use. The role of dietary deficiency is not exactly known but deficiency of vitamin B complex, iron and other antioxidants may be responsible. There is a definite male preponderance with a peak age incidence of 50 to 60 years. According to the UICC classification (Union Internationale Center le Cancer), the ‘T’ staging of oral cancer is according to the size of the tumour: T1 = tumour less than 2 cms in maximum diameter. T2 = tumour between 2 and 4 cms. T3 = tumour between 4 and 6 cms. T4 = tumour more than 6 cms or tumour of any size extending to neighboring structures like bone, muscles, nerves or skin.

TEST YOURSELF

Read the clinical scenario given at the beginning and answers the following questions 1234567-

What is the most likely diagnosis in this case? What are the important points in history taking in this patient? What are the important points in clinical examination of this patient? What investigations are required in this case? How will you treat this case? What are the aetiological factors for squamous cell carcinoma of the oral cavity? Outline TNM classification for oral carcinoma?

116

Clinical scenarios in oto-rhino-laryngology

Case 37 Clinical Scenario A 42 years old male patient presented with a growth on the upper surface of the tongue for last 3 months (fig 37.1). Initially it was very small but it increased in size rapidly. He was a pan chewer and ‘Hukka’ smoker for last 25 years.

Fig. 37.1 Patient with a growth on the tongue involving dorsal surface, tip and lateral margin on the right side

Important points in history taking: 1- Detailed history regarding the growth. According to the patient, it was present for last 3 months and he did not know how it started. Initially it was very small but soon it started to increase in size till its present form (fig. 37.1). There was no aggravating or relieving factor known to him and there was also no associated factor. 2- Any history of pain in the swelling or burning sensation in the mouth. There was no history of pain in the growth or surrounding area but he had history of burning sensation in the mouth. 3- Any history of using beetle nut, pan, tobacco, smoking or alcohol. He used to eat pan with tobacco and he was a ‘Hukka’ smoker for last 25 years. He used to take 8-10 pan per day. There was no history of consumption of alcohol. 4- Any swelling in the neck or below mandible. There was no such history. 5- Any history of fever, anorexia or weight loss. There was no such history. Important points in clinical examination: 1- General physical examination. Patient was a middle aged person of average height and built, fully oriented in time, space and person. His vital signs were within normal limits and no positive finding on general physical examination. 2- Examination of the growth. There was a growth present on the tongue 117

Clinical scenarios in oto-rhino-laryngology

involving dorsal surface, tip and lateral margin on the right side and going on the under surface as well. It was irregular in shape, 4x4 cms. in size, surface was irregular with some small finger like projections. The margins were also irregular but well defined and adjacent mucosa was normal looking. On palpation it was non-tender, firm in consistency with induration of the surrounding area and deeper structures and not bleeding on touch. 3- Examination of the cervical lymph nodes. There was no palpable lymph node in the neck on any side. 4- Examination of rest of the oral cavity and oropharynx. Oral hygiene was poor, teeth were stained with pan and mouth opening was slightly restricted. 5- Examination of the hypopharynx and larynx. It was within normal limits. Differential Diagnosis: 1- Benign tumours like squamous papilloma. 2- Malignant tumours like verrucous carcinoma or squamous cell carcinoma. Investigations: 1- Punch biopsy. It was done under surface anaesthesia, tissues taken from multiple areas of the swelling and sent for histopathological examination. The histopathology report showed verrucous carcinoma. 2- Baseline investigations for general anaesthesia. All were normal. Diagnosis: The histopathological diagnosis in this case was verrucous carcinoma of the tongue. The clinical staging was T2N0M0. Treatment: The patient was admitted and planned for partial glossectomy under general anaesthesia. Complete excision of the swelling was done with safe margins all around and primary closure done. Specimen sent for histopathology which confirmed the pre-operative diagnosis. Post-operative recovery of the patient was uneventful. Patient was disease free in the follow-up period. Discussion: Squamous cell carcinoma is the commonest tumour of the oral cavity and the lips. The incidence of squamous cell carcinoma according to different sites of the oral cavity is as follows: - Tongue 35% Lateral border 31% Tip 2% Dorsum 2% - Floor of the mouth 30% Anterior 25% Lateral 5% 118

Clinical scenarios in oto-rhino-laryngology

- Lower alveolus - Buccal mucosa - Upper alveolus - Hard palate - Retro-molar trigone

15% 10% 5% 3% 2%

Verrucous carcinoma is a variant of squamous cell carcinoma and has a characteristic warty or papillary appearance usually grayish in colour. It is a low grade malignant tumour which rarely metastasizes to regional lymph nodes and never distantly. It has a very good prognosis and treated mainly by surgical excision with safe margins.

TEST YOURSELF

Read the clinical scenario given at the beginning and answers the following questions 123456-

What is the most likely diagnosis in this case? What are the important points in history taking in this patient? What are the important points in clinical examination of this patient? How will you manage this case? What is verrucous carcinoma and what is its overall prognosis? What is the incidence of squamous cell carcinoma at different sites of the oral cavity?

119

Clinical scenarios in oto-rhino-laryngology

Case 38 Clinical Scenario A father brought his 4 years old child in emergency with the complaint that his child had ingested a foreign body (coin) 2 hours back. After ingestion of coin he was complaining of pain in the throat and he is unable to swallow any food. Important points in history taking: 1- Any history of breathlessness or cyanosis. In this case there was no such history. 2- Any history of dysphagia or odynophagia. In this case patient was unable to swallow any food or liquid after ingestion of foreign body. Before this incident the child had no symptoms. Important points in clinical examination: 1- General physical examination. The child was fully conscious but slightly anxious and breathing comfortably. 2- Vital signs examination. All were within normal limits. 3- Check for cyanosis or breathlessness. There was no cyanosis or breathlessness. 4- Check for dehydration. Patients was not dehydrated. 5- Examination of the throat. There was pooling of saliva in the oral cavity, otherwise there was no significant finding. 6- Check for laryngeal crepitus. It was absent. 7- Indirect laryngoscopy. It was not possible to perform indirect laryngoscopy because the patient was an anxious and non-cooperative child. 8- Auscultation of the chest. There was no positive finding on auscultation of the chest. Investigations: 1- Plain X-ray chest and neck (AP view). It showed a radiopaque circular foreign body impacted at the lower part of the neck (fig. 38.1). As the foreign body was a coin and in AP view it was appearing as circular, it means it was impacted in the hypopharynx (not in the larynx). For localization of other foreign bodies, a plain X-ray in the lateral view is also required to diagnose whether it is in hypopharynx or larynx. Diagnosis: This was a case of impacted foreign body (a coin) in the hypopharynx. Treatment: The patient was admitted in the hospital and planned for removal of 120

Clinical scenarios in oto-rhino-laryngology

Fig. 38.1 X-ray chest and neck (AP view) showing a radiopaque circular foreign body (coin) impacted in hypopharynx

foreign body through endoscopy under general anaesthesia. After endotracheal intubation rigid anterior commissure type of direct laryngoscope was passed and hypopharynx examined (fig. 38.2). The coin was visible, impacted at the crico-pharyngeous. Foreign body was removed with the help of crocodile forceps. Post-operative recovery was uneventful and the patient was discharged from the hospital after 6 hours.

Fig. 38.2 Rigid anterior commisure type of direct laryngoscope with crocodile forceps

Discussion: Foreign body impaction in the pharynx and oesophagus is quite common. Children are most commonly the victims, as they have tendency to put different objects in the mouth and during playing these objects get impacted in the throat. In adults foreign body may get impacted accidentally or in psychotic patients by themselves to commit suicide. Commonly encountered foreign bodies in the digestive tract are coins, food bolus (usually meat), bones, pins and dentures etc. Digestive tract foreign bodies are likely to be impacted either at the site of anatomical narrowing or sometimes at pathological narrowing like stricture, stenosis or neoplasia. The commonest site of impaction of ingested foreign bodies is at or above the crico121

Clinical scenarios in oto-rhino-laryngology

pharyngeous sphincter. In most of the cases removal of the foreign body is possible through rigid endoscopy (laryngoscopy or oesophagoscopy) depending upon the site of impaction.

TEST YOURSELF

Read the clinical scenario given at the beginning and answers the following questions 1234-

What are the possible sites of impaction of an ingested foreign body? What is the probable site of impaction in this patient? How will you investigate this case? How will you manage this case?

122

Clinical scenarios in oto-rhino-laryngology

Case 39 Clinical Scenario A 8 years old girl presented with a swelling in floor of the mouth under the tongue for last few months. She also complained of discomfort and difficulty in chewing and swallowing for last one month due to the swelling. Clinical examination showed a smooth, soft and fluctuant swelling in the floor of the mouth (fig. 39.1).

Fig. 39.1 Patient with a swelling in floor of the mouth

Important points in history taking: 1- Detailed history regarding the swelling. According to the patient, she noticed this swelling few months before when it was very small. Then it started to increase in size day by day. 2- Any external swelling in the neck, sub-mental, sub-mandibular region. There was no external swelling. 3- Any change in size of the swelling in relation to meals. There was no such change. 4- Any history of pain in the swelling. It was painless. 5- Any history of bleeding or discharge from the swelling. There was no such history. 6- Any history of fever. There was no history of fever. Important points in clinical examination: 1- Inspection of the swelling. There was a single, smooth, roughly oval shaped, slightly bluish coloured swelling of 4 cms. in maximum diameter was present in floor of the mouth (fig. 39.1). 2- Palpation of the swelling. It was non-tender, soft, fluctuant and nonreducible swelling. 3- Bimanual palpation of floor of the mouth. Nothing significant. 4- Examination of the tongue movements. It was normal. 123

Clinical scenarios in oto-rhino-laryngology

5- Palpation of cervical lymph nodes. No palpable lymph nodes. Differential Diagnosis: 1- Ranula 2- Haemangioma 3- Sialoadenitis Investigations: 1- Baseline investigations for general anaesthesia. All were normal. Diagnosis: This was a case of simple ranula. Treatment: The patient was admitted and planned for marsupialization under general anaesthesia, where roof of the ranula was removed and the inner lining was stitched with the oral mucosa. Post-operative recovery was uneventful. Discussion: Ranula is a retention cyst in the floor of the mouth arising from the mucous gland or from sub-mandibular or sub-lingual salivary gland or its ducts. There are two types of ranula: 1- Simple ranula: The retention cyst is limited to the floor of mouth. 2- Plunging ranula: The cyst may extend into the tissues of the neck and presents externally in sub-mental or sub-mandibular region. It is due to extravasation of its content into the soft tissues below mylohyoid muscle.

TEST YOURSELF

Read the clinical scenario given at the beginning and answers the following questions 12345-

What is the most likely diagnosis in this case? What are the important points in history taking and clinical examination in this patient? What are the differential diagnosis in this case? How will you manage this case? What is ranula and what are its different types?

124

Clinical scenarios in oto-rhino-laryngology

Section IV

LARYNX & TRACHEA Case 40Case 41Case 42Case 43-

Vocal nodules Tracheostomy Carcinoma of the larynx Foreign body of the bronchus

125

127 130 133 138

Clinical scenarios in oto-rhino-laryngology

Case 40 Clinical Scenario A 38 years old male primary school teacher by profession came in OPD with the complaints of voice change/hoarseness for the past six to seven months. Hoarseness was continuous and increasing day by day slowly. There was no definite history of fever or pain in the throat. He had never done smoking in his life. Important points in history taking: 1- Occupation/vocal abuse. There was definite history of vocal abuse as he was a primary school teacher by profession. 2- Detailed history regarding hoarseness. According to him the onset of hoarseness was insidious. It was continuous and progressively increasing. It was aggravated by vocal abuse and relieved to some extent by voice rest. It was not associated with respiratory distress, stridor or any other airway problem. 3- History of neck trauma or surgery. There was no such history. 4- Smoking/alcohol/exposure to chemicals or fumes. He was a non-smoker and had no history of alcohol consumption or exposure to chemicals. 5- Past history of tuberculosis/ tuberculosis in the family. There was no such history. 6- Fever. There was no history of fever. 7- Weight loss/anorexia/dysphagia. There was no such history in this case. Important points in clinical examination: 1- General physical examination. Only positive finding on general physical examination was hoarse voice. 2- Indirect laryngoscopy. It is very important to examine the interior of the larynx especially vocal cords. This patient was very anxious during indirect laryngoscopy so the vocal cords could not be seen. 3- Flexible fiber-optic laryngoscopy. This is of much help especially in anxious persons where indirect laryngoscopy is difficult. On flexible laryngoscopy two nodular thickenings were found at the junction of anterior one-third and posterior two-third of the true vocal cords. Both vocal cords were found to be fully mobile. 4- Neck nodes palpation. In this case there was no palpable cervical lymph node. 5- Other ENT examination. It was within normal limits. Diagnosis: The diagnosis in this case was vocal nodules. 127

Clinical scenarios in oto-rhino-laryngology

Differential Diagnosis: The other causes of chronic hoarseness should be considered in the differential diagnosis like: 1- Chronic laryngitis 2- Laryngeal tuberculosis 3- Intubation granuloma 4- Benign laryngeal tumour e.g. papilloma 5- Malignant laryngeal tumour e.g. squamous cell carcinoma 6- Vocal cord paralysis e.g. after thyroid surgery, bronchogenic carcinoma 7- Vocal cord polyp 8- Laryngocoele Investigations: 1- Baseline investigations for general anaesthesia when planned for surgery like blood complete picture, ESR, random blood sugar, blood coagulation studies, urine D/R and X-ray chest (PA view). All were within normal limits. Treatment: After flexible laryngoscopy and baseline investigations, patient was planned for micro-laryngoscopy and excision of the vocal nodules. Appearance of the vocal nodules on microlaryngoscopy is shown in fig 40.1. In microlaryngoscopy an operating microscope was used with an objective lens of 400 mm to see and treat laryngeal pathology under magnification (fig 40.2). Complete excision of the vocal nodules were done under microscope. Post-operative recovery of the patient was uneventful. Patient was advised voice rest for 7 to 10 days along with a broad spectrum prophylactic antibiotic. Patient’s voice became normal after that. He was also advised to avoid vocal abuse and shouting thereafter. Discussion: It is a condition which occurs in persons who use their voice excessively with straining or faulty production. It is also called ‘singer’s

Fig. 40.1 Vocal nodules as seen on microlaryngoscopy

128

Clinical scenarios in oto-rhino-laryngology

Fig. 40.2 Micro-laryngoscopy procedure

node’ or ‘screamer’s node’. This condition is seen in singers, teachers, hawkers, actors etc. It is also seen in mothers of young children who shout a lot and persons talking to the deaf. These people sometimes squeeze their voice. As a result of this small haematoma is formed at the point of maximum impact of vocal cords. Due to repeated straining this haematoma may organize and forms nodular thickening. These are always bilateral, grayish white in colour and at the junction of anterior one-third and posterior two-third of the true vocal cords.

TEST YOURSELF

Read the clinical scenario given at the beginning and answers the following questions 12345-

What is the most likely diagnosis in this case? What are the differential diagnosis in this case? How will you investigate this case? How will you manage this case? What is the pathology of vocal nodules?

129

Clinical scenarios in oto-rhino-laryngology

Case 41 Clinical Scenario A call recieved from ICU for tracheostomy in a 29 years old female patient who is on artificial respiration (ventilator) for last 4 days with endotracheal intubation (fig. 41.1). Four days back she had undergone some surgery under general anaesthesia and developed cerebral hypoxia during anaesthesia and is on ventilator after that.

Fig. 41.1 ICU patient on artificial ventilation with endotracheal tube

Important points in history taking and clinical examination: 1- Clinical assessment for how long the patient will remain on ventilator. For short duration artificial ventilation, endotracheal intubation is preferable but for prolonged artificial ventilation (more than 72 hours), tracheostomy is better. This patient was already on endotracheal intubation for the last 4 days and still expected time period for recovery is unknown, so tracheostomy should be done in this patient. 2- Endotracheal tube if remained for a longer time, it causes trauma to lips, oral cavity, pharynx, vocal cords and sub-glottic area. So after performing tracheostomy and removal of endotracheal tube, look for any injury in these areas. Investigations: No special investigation was required in this case. Treatment: Patient was planned for elective standard tracheostomy in operation theatre. In the presence of an anaesthetist with all monitoring and artificial ventilation through endotracheal tube, patient was positioned on operation table with sand bag under the shoulders and head ring under the head. Local 130

Clinical scenarios in oto-rhino-laryngology

anaesthesia was administered and a horizontal incision was given two fingers above the supra-sternal notch from anterior border of one sterno-cleidomastoid muscle to the other. Flap raised, strap muscles seperated in the midline, thyroid isthmus retracted down, trachea opened and Portex tracheostomy tube inserted (fig. 41.2 and 41.3). Before opening the trachea anaesthetist was asked to remove endotracheal tube and after insertion of the tracheostomy tube ventilation circuit was connected to it.

Fig. 41.2 Portex tracheostomy tube with cuff

Fig. 41.3 Patient after tracheostomy

Discussion: In comatosed ICU patient where tracheostomy is needed, we have two options: 1- Standard tracheostomy in operation theatre 2- Percutaneous tracheostomy (PCT) in ICU Shifting of the patient to operation theatre is difficult as these patients are already on ventilator. To avoid this problem we have option of percutaneous tracheostomy in ICU. In this patient we performed tracheostomy in operation theatre because PCT kit is expensive and was not available at that time. In percutaneous tracheostomy (PCT), first a guide wire is introduced percutaneously into the trachea and then dilators of progressive thickness are 131

Clinical scenarios in oto-rhino-laryngology

introduced through this guide wire so as to make a bigger hole. Finally when the hole is large enough tracheostomy tube is passed through the same guide wire. Advantages of PCT are: 1- No need of operation theatre. 2- Hazards of shifting such patients to OT can be avoided. 3- Smaller and more aesthetic wound. 4- Less operative bleeding. 5- Less chances of wound infection. But this procedure is hazardous in the following situations and should not be performed: 1- Patients having short and thick neck. 2- Goitre. 3- Patients below 15 years of age. 4- Previous tracheostomy patients. 5- Previous laryngeal or neck surgery patients.

TEST YOURSELF

Read the clinical scenario given at the beginning and answers the following questions 1234-

What are the options available in this patient for performing tracheostomy? Briefly outline the steps of tracheostomy operation. What is percutaneous tracheostomy? and what are its advantages? What are the contra-indications for percutaneous tracheostomy?

132

Clinical scenarios in oto-rhino-laryngology

Case 42 Clinical Scenario A 49 years old male patient came in OPD with complaints of change of voice, respiratory distress on exertion and sometimes noisy breathing for last 3 to 4 months. He was a smoker for last 35 years taking about 20 to 25 cigarettes per day. Important points in history taking: 1- Detailed history regarding his symptoms. According to the patient he was in his usual state of health 3 to 4 months before when he noticed change in his voice (hoarseness). Onset of hoarseness was insidious and initially he ignored it but after sometime it started to increase for which he consulted his family physician who gave him some medication. Hoarseness was continuous and progressively increasing with no definite aggravating or relieving factors. After sometime he also experienced respiratory distress or dyspnoea on exertion and sometimes noisy breathing as well. It was relieved by taking rest. 2- Any history of dysphagia. According to patient he had no difficulty in swallowing but sometimes he felt that something was obstructing his throat or something was present in his throat. 3- Any history of fever, pain or cough. There was no history of fever or pain but he had cough on and off. 4- Any history of anorexia or weight loss. He had history of anorexia and weight loss for last few months. 5- Any history of neck swelling. There was no such history. 6- Any history of previous surgery. There was no history of any surgery in the past. 7- Any family history of throat diseases. His father had carcinoma of the larynx and he had been operated for it about 20 years back. 8- Occupation of the patient and history of vocal abuse. He was an electrician and there was no history of vocal abuse. Important points in clinical examination: 1- General physical examination. Patient was a middle aged person of average height and lean built, fully oriented in time, space and person. His voice was hoarse and breathing was laboured. On number counting he was not able to count more than 5 or 6 in one breath. His pulse was 86 per minute, respiratory rate was 24 per minute, temperature was 98.7oF and blood pressure was 140/90 mm. of Hg. There was no cyanosis, anaemia or other positive finding on general physical examination. 2- External examination of the neck. There was widening or splaying of the thyroid cartilages (fig. 42.1). Laryngeal crepitus was positive. 133

Clinical scenarios in oto-rhino-laryngology

Fig. 42.1 Patient showing widening or splaying of the thyroid cartilages

3- Examination of the oral cavity and oropharynx. There was nothing significant on it. 4- Indirect laryngoscopy. Indirect laryngoscopy was difficult with overhanging epiglottis. 5- Fiber-optic flexible direct laryngoscopy. It showed an extensive growth on the right side involving full length of true vocal cord and extending both above and down to involve supra-glottic as well as sub-glottic region. Right vocal cord was fixed and the airway was compromised due to growth (fig. 42.5). 6- Examination of cervical lymph nodes. There was no palpable cervical lymph node. 7- Rest of ENT and systemic examination. There was no significant finding. Differential Diagnosis: 1- Squamous cell carcinoma 2- Other malignant tumour 3- Benign tumour 4- Other causes of chronic hoarseness Investigations: 1- Direct laryngoscopy and biopsy. It was done under general anaesthesia and all the findings of flexible laryngoscopy were confirmed. Punch biopsy from multiple areas were taken and sent for histopathology which showed poorly differentiated squamous cell carcinoma. 2- CT scan of the neck. It showed an extensive growth present in the larynx on right side involving supra-glottic, glottic and sub-glottic region causing marked narrowing of the airway (fig. 42.2). 3- Investigations for distant metastasis like X-ray chest (PA view), ultrasonography of the abdomen etc. All were within normal limits. 4- Other baseline investigations including blood complete picture, coagulation profile, liver function tests, renal function tests, serum electrolytes etc. All were within normal limits. 134

Clinical scenarios in oto-rhino-laryngology

Fig. 42.2 CT scan of the neck (axial view) showing growth in larynx with airway narrowing

Diagnosis: This was a case of squamous cell carcinoma of the larynx. Clinical staging was T4N0M0. Treatment: The patient was planned and prepared for total laryngectomy under general anaesthesia. Patient needed pre-operative tracheostomy due to extensive growth, anaesthetist was unable to pass endotracheal tube. ‘U’ shaped modified Gluck/Sorenson’s incision was used (fig. 42.3). Whole of the larynx was removed along with hyoid bone and muscles (fig. 42.4 and 42.5). Pharynx was reconstructed and tracheal opening stitched to the skin. Drain applied and wound closed in layers. Post-operative recovery was uneventful. Discussion: Squamous cell carcinoma of the larynx is common and it is among the list of top ten commonest malignancies in Pakistan. It has a high rate of cure and a good prognosis. Early diagnosis and correct treatment gives a long survival. Laryngeal carcinoma has a definite male preponderance but the sex incidence varies worldwide. In Western countries the incidence of laryngeal

Fig. 42.3 Patient with pre-operative marking for ‘U’ shaped modified Gluck/Sorenson’s incision

135

Clinical scenarios in oto-rhino-laryngology

Fig. 42.4 Patient’s open neck after removal of the larynx

Fig. 42.5 Specimen of the larynx after removal showing growth on the right side

carcinoma in females is increasing, probably because of the smoking habits. The peak age of presentation is between forty and sixty years. Laryngeal carcinoma in childhood is a rare entity. Like most of the other malignancies, the aetiology of laryngeal carcinoma is unknown. A number of possible aetiological factors have been described. Smoking is considered as most important aetiological factor. The risk is directly proportional to the number of cigarettes smoked per day, duration of smoking and use of filter. Tobacco in other forms like hukka, berri, tobacco chewing etc. also has the same effect. Alcohol consumption is another important causative factor especially in western countries. Other factors, which are associated with the increase incidence of laryngeal carcinoma, include environmental pollution, vocal abuse, radiation to the neck and occupational exposure to asbestos and certain other metals. In childhood laryngeal carcinoma radiation is important, as malignant change may occur in juvenile laryngeal papillomatosis treated with radiation. The squamous cell carcinoma develops not only from areas normally covered by squamous epithelium but it may also arise from areas that are lined by ciliated columnar and transitional epithelium. According to the site of origin and involvement, laryngeal carcinoma are classified as: 1- Supra-glottic 136

Clinical scenarios in oto-rhino-laryngology

2- Glottic 3- Sub-glottic 4- Transglottic Most of the carcinomata develop on the vocal cords (glottic carcinoma), about 76% of total cases. Anterior half of the vocal cord is more commonly involved than the posterior half. Supra-glottic carcinoma arises from any part of supraglottis and comprises about 19%. Those that arise from the subglottic region are sub-glottic carcinomata, these are rare about 1-5% of total cases. Transglottic carcinomata are glottic lesions that invade both the supraglottic and sub-glottic regions of the larynx. These tumours have deep extension, which cross the laryngeal ventricle vertically to involve two or more anatomical areas.

TEST YOURSELF

Read the clinical scenario given at the beginning and answers the following questions 123456-

What are the differential diagnosis in this case? What are the important points in history taking and clinical examination in this case? How will you investigate this case? How will you manage this case? What are the aetiological factors of carcinoma of the larynx? What is the commonest site for origin of carcinoma of the larynx?

137

Clinical scenarios in oto-rhino-laryngology

Case 43 Clinical Scenario A 3 years old boy was referred from paediatric department for opinion. This boy first came with the complaint of cough, fever and breathlessness about two weeks back and diagnosed as a case of lower respiratory tract infection and treated accordingly. The condition improved with treatment but after stopping the drugs he again developed same condition. Plain X-ray chest (PA view) was done which showed a radiopaque metallic foreign body (a metallic nail) impacted in the left main bronchus (fig. 43.1).

Fig. 43.1 X-ray chest (PA view) showing a radiopaque foreign body (metallic nail) impacted in the left main bronchus

Important points in history taking: 1- Any history of foreign body ingestion. According to his parents there was no history of foreign body ingestion. Some construction work was going on in their house and the child might have taken this foreign body in their absence. 2- Any history of choking attack. There was no such history. Important points in clinical examination: 1- General physical examination. The child was looking ill but fully conscious and oriented. His pulse was 116 per minute, respiratory rate was 30 per minute, temperature was 101oF. There was no cyanosis or other positive finding on general physical examination. 2- Inspection of the chest. It showed slightly decreased movement of the chest on left side. 3- Auscultation of the chest. Air entry was reduced on the left side with crepts all over the chest. 4- Percussion of the chest. It has a slightly dull note on left side of the chest as compared to right side. 138

Clinical scenarios in oto-rhino-laryngology

Investigations: 1- Plain X-ray chest (PA view). It was already done which showed a radiopaque foreign body (metallic nail) in the left main bronchus with prominent vascular marking (fig. 43.1). 2- Baseline investigations for general anaesthesia. All were within normal limits. Diagnosis: This was a case of impacted foreign body (metallic nail) in the left main bronchus. Treatment: The patient was admitted and planned for removal of foreign body through bronchoscopy under general anaesthesia. Parenteral broad spectrum antibiotic was started immediately with other supportive treatment. Anaesthetist opinion was taken for fitness and planned for elective bronchoscopy next day. Bronchoscope was passed into the trachea till opening of the left main bronchus which showed marked oedema and granulation tissues. With some manipulation foreign body became visible which was removed with a crocodile forceps. Post-operative recovery was uneventful, he remained in hospital for two days and his condition improved very much during this period. Discussion: Most of the foreign bodies entering into the respiratory tract, pass through the larynx and lodge in the trachea or mostly in bronchi. Bronchus on the right side is involved more than the left side, because right bronchus is wider and more in line with the trachea. Clinical features of impacted foreign body in the respiratory tract depend upon the site of impaction, size and nature of the foreign body. Cough and dyspnoea may be present initially at the time of accident. Vegetative foreign bodies like chalia and peanuts produce an intense inflammatory reaction in the mucosa. Symptoms of acute tracheo-bronchitis may be present. Emphysema, atelactasis, lung abscess, pneumonia or even lung collapse may occur. Non-vegetative foreign bodies like whistle, needle, metallic pieces etc. may remain silent for sometimes because of minimal local inflammatory reaction. Atelactasis occurs if the foreign body completely obstructs the lumen. In partial obstruction, obstructive emphysema may develop. Radiopaque foreign body may be seen on plain radiograph. Radiolucent foreign body especially vegetative may produce changes on radiology due to inflammatory reaction or obstruction. Emphysema, pneumonia, lung collapse and bronchitic changes may be seen. Removal of the tracheal and bronchial foreign body is done through rigid type of bronchoscopy. Rigid bronchoscopy is performed under general anaesthesia where a small lumen size endotracheal tube is preferred. Patient lies in supine position with flexion at cervical vertebrae and extension of head 139

Clinical scenarios in oto-rhino-laryngology

Fig. 43.2 Bronchoscopy procedure

at atlanto-occipital joint. This brings the laryngo-tracheal axis in line with the oral axis (fig. 43.2). Bronchoscope is held in the right hand and introduced through the mouth into the larynx. Bronchoscope is then advanced and whole tracheo-bronchial tree is examined. Foreign body can be removed through the rigid bronchoscope by holding it with crocodile forceps. If foreign body is large and it is not possible to pass it through the bronchoscope, then bronchoscope is also withdrawn along with the foreign body.

TEST YOURSELF

Read the clinical scenario given at the beginning and answers the following questions 12345-

What is the probable site of impaction of foreign body in this patient? How will you manage this case? What are the clinical features of impacted foreign body in the trachea or bronchus? How is the foreign body removed from the trachea or bronchus? Outline the steps of rigid bronchoscopy procedure.

140

Clinical scenarios in oto-rhino-laryngology

Section V

HEAD & NECK Case 44Case 45Case 46Case 47Case 48Case 49Case 50-

Ludwig’s angina Branchial cyst Retro-pharyngeal abscess Sub-mandibular salivary calculus Parotid gland pleomorphic adenoma Multinodular goiter (MNG) Tuberculous cervical lymphadenopathy

141

143 146 149 152 155 158 162

Clinical scenarios in oto-rhino-laryngology

Case 44 Clinical Scenario A 42 years old male patient had a history of tooth extraction about 5 days back of the left lower jaw tooth. He presented in OPD with complaint of painful swelling and redness below his chin and lower jaw, which started three days back and increased rapidly in last two days. External examination findings are shown in fig. 32.1.

Fig. 32.1 Patient with marked swelling and redness over the sub-mental and sub-mandibular region, more on the left side

Important points in history taking: 1- Detailed history regarding swelling. According to the patient he noticed a painful swelling in the floor of mouth and below mandible three days back. Rapidly the swelling increased next day along with severe pain. There was no aggravating or relieving factor for the swelling. It was associated with pain on swallowing and excessive salivation. 2- Detailed history regarding pain. Pain started before the swelling. Initially it was mild but soon became moderate to severe. It was continuous, sharp cutting in nature and localized. It aggravated by swallowing and relieved to some extent by taking analgesics. 3- Any history of fever. Patient had history of fever without rigors, around 101 to 102oF. 4- Detailed history regarding his dental problem. According to the patient he had dental carries in multiple teeth. He developed pain and swelling around gums in one tooth on left lower side for which he consulted a dentist. Tooth extraction was done five days back. 5- Any history of respiratory distress, dyspnoea or stridor. There was no such history. Important points in clinical examination: 1- General physical examination. Patient was a middle aged person of 143

Clinical scenarios in oto-rhino-laryngology

2-

3-

456-

average height and slightly obese built, looking ill but fully oriented in time, space and person. His pulse was 100/minute, blood pressure was 130/85 mm of Hg., respiratory rate was 22/minute and temperature was 100.8oF. External examination of the neck. It showed a generalized swelling involving sub-mental and sub-mandibular region on both sides but more marked on left side (fig. 32.1). The skin over the swelling was smooth, red and congested. The margins of the swelling were ill defined. There was no pulsation, discharge, sinus, hyperpigmentation or reducibility. On palpation temperature was raised over the swelling. It was tender, firm, non-fluctuant, smooth with ill defined margins. Trans-illumination was negative. Examination of the oral cavity and throat. It showed restricted tongue movement with swelling in floor of the mouth. Pooling of saliva was also present in the oral cavity. Signs of tooth extraction were present in first molar tooth on the left lower side with swelling around the gums. Bi-manual palpation. It was not done because of severe pain. Any signs of respiratory distress, dyspnoea, cyanosis or stridor. All were absent. Rest of ENT examination was within normal limits.

Diagnosis: The diagnosis in this case was Ludwig’s angina. Investigations: 1- Blood complete picture. It showed increased in the total white cell count with 80% neutrophils. Treatment: The patient was admitted in the hospital for further management and observation. Parenteral antibiotics (amoxicillin with clavulanic acid and metronidazole) were started along with analgesic and anti-inflammatory drug. Patient showed marked improvement within 48 hours. He was discharged on oral medications with uneventful recovery. Discussion: Ludwig’s angina is the infection of sub-mandibular space. This space is bounded above by the mucous membrane of the floor of mouth and tongue and below by deep fascia which extends from the hyoid to the mandible. It is divided into two compartments by the mylohyoid muscle: 1- Sub-lingual: It lies above the mylohyoid muscle. 2- Sub-maxillary and sub-mental: It lies below the mylohyoid muscle. In about 80% of the cases, infection reaches to this space by extension of dental root infection. The other causes are sub-mandibular sialadenitis, penetrating injuries of the floor of mouth and mandibular fractures. Most of the infections of this space are caused by mixed aerobic and anaerobic 144

Clinical scenarios in oto-rhino-laryngology

organisms. Common organisms responsible are streptococci, staphylococci, E. coli and bacteroids. If Ludwig’s angina is not treated properly it can give rise to following complications: 1- Laryngeal oedema 2- Mediastinitis 3- Lower respiratory tract infection 4- Parapharyngeal abscess 5- Retro-pharyngeal abscess

TEST YOURSELF

Read the clinical scenario given at the beginning and answers the following questions 1234567-

What is the diagnosis in this case? What are the important points in history taking in this patient? What are the important points in clinical examination of this patient? How will you manage this case? What is the pathology of Ludwig’s angina? What are the boundries and compartments of sub-mandibular space? What complications can occur if Ludwig’s angina is not treated early?

145

Clinical scenarios in oto-rhino-laryngology

Case 45 Clinical Scenario A 17 years old boy presented with a swelling on lateral aspect of the neck on left side (fig 45.1). According to him this swelling was present since childhood but caused no problem to him as it was very small and painless, but for the last two years it had increased in size. Two months back he had undergone incision and drainage under local anaesthesia by his family physician but swelling reappeared soon.

Fig. 45.1 Patient with swelling on lateral aspect of the neck on left side with redness of overlying skin

Important points in history taking: 1- Detailed history about the swelling. According to the patient this swelling was present since childhood and he did not know how it started. Initially it was very small and barely visible to others and it remained small for a long time. Two years back it started to grow very slowly and during this period it became large to its present size. There was no aggravating or relieving factor and it was not associated with any other complaint. 2- Any history of pain. Patient stated that initially there was no pain but as the swelling increased in size he occasionally felt mild localized pain in the swelling. After incision and drainage of the swelling he developed recurrent pain in the swelling which was mild to moderate in severity. 3- Any history of fever. There was no definite history of fever but occasionally he felt that his body was warm during last two months. 4- Detailed history about the surgical procedure. According to him he consulted his family physician regarding the swelling who did incision and drainage under local anaesthesia. Some thick yellowish material came out and for some times swelling disappeared but after few days it again started to grow and became of same size as before. He also noticed redness of the skin over the swelling and has recurrent pain after surgery. 146

Clinical scenarios in oto-rhino-laryngology

Important points in clinical examination: 1- Examination of the swelling. A rounded, smooth, single swelling of 5 cms. diameter was present on the lateral aspect of neck on the left side just at anterior border of sternocleidomastoid muscle near its middle (fig. 45.1). The margins of the swelling were well defined. The skin over the swelling was reddened with a scar mark of previous surgery. There was no pulsation, discharge, opening or sinus, reducibility, increase of size on coughing and movement of swelling with swallowing. Temperature over the swelling was slightly raised and tenderness was present. The skin over the swelling was mobile and swelling itself was also mobile in both vertical and horizontal direction. The swelling was soft to firm in consistency and transillumination was positive. 2- Examination of the throat, nose and ears. It was within normal limits. Differential Diagnosis: Following conditions should be considered in differential diagnosis of a swelling in the lateral aspect of neck for long duration: 1- Branchial cyst 2- Epidermoid cyst 3- Lymphadenopathy 4- Thyroid nodule 5- Parapharyngeal space tumour Investigations: 1- Ultrasound of the neck. It showed presence of a thick walled cyst of 6 cms. in maximum diameter. Thyroid gland was separate from the swelling and was normal in size and texture. 2- Fine needle aspiration cytology. Aspirate mainly consist of straw coloured fluid which contained few lymphocytes. 2- Baseline investigations for general anaesthesia. All were within normal limits. Diagnosis: The diagnosis of this case was branchial cyst. Treatment: The patient was planned for excision of branchial cyst under general anaesthesia. Horizontal incision was given over the swelling and skin flaps including platysma muscle were raised above and down. Deep fascia was cut anterior to sternocleidomastoid muscle and muscle retracted posteriorly. Whole cyst was separated from surrounding and removed completely. Wound closed in layers. Discussion: The branchial cyst is also known as lateral cervical cyst. It is usually present in lateral part of the neck deep to sternocleidomastoid muscle at the 147

Clinical scenarios in oto-rhino-laryngology

junction of upper one-third and lower two-third. The origin of branchial cyst is debatable and there are four theories of origin: 1- Branchial apparatus theory 2- Cervical sinus theory 3- Thymopharyngeal duct theory 4- Inclusion theory Branchial cyst is mostly lined by stratified squamous epithelium and contains straw coloured fluid with cholesterol crystals. In 80% of the cases, the wall of the cyst contains lymphoid tissues. Males are slightly more affected than females (ratio of 3:2). The peak age incidence for presentation of branchial cyst is third decade. Majority are present in the classical upper lateral neck but some are also present in lower neck, parotid and posterior triangle of the neck.

TEST YOURSELF

Read the clinical scenario given at the beginning and answers the following questions 12345-

What is the most likely diagnosis in this case? What are the differential diagnosis in this case? How will you manage this case? What are the theories of origin of branchial cyst? What is the classical position of branchial cyst in the neck?

148

Clinical scenarios in oto-rhino-laryngology

Case 46 Clinical Scenario A 28 years old male patient came with the complaint that three days back during lunch, a sharp fish bone got penetrated into the posterior wall of the throat, half of it was visible in the throat. He went to his family physician who removed the foreign body. After removal he had some discomfort in the throat which increased next day. After two days he had severe pain in the throat along with difficulty in swallowing. Important points in history taking: 1- Detailed history regarding pain. According to the patient he had discomfort in the throat after removal of the foreign body and next day it increased to moderate pain. It was continuous, localized, dull in character, not radiating to anywhere and progressively increasing and became severe next day. 2- Detailed history regarding difficulty in swallowing. According to him, next day after removal of the foreign body he developed difficulty in swallowing which increased rapidly and he was unable to eat anything and even he was not able to swallow his own saliva. 3- Any history of fever. He had fever without rigors for last one day. He checked his body temperature in the morning and it was 101oF. 4- Any history of neck swelling. There was no such history. 5- Any history of inability to open mouth (trismus). There was no such history. 6- Any previous history of recurrent sore throat. He had no such problem. 7- Any history of diabetes mellitus or any other immuno-compromised state. He was non-diabetic and otherwise healthy. 8- Any history of pain or restriction in neck movements. There was no such complaint. 9- Any history of dyspnoea, stridor or voice change. His voice was muffled after the start of this problem. Important points in clinical examination: 1- General physical examination. Patient was a young male looking ill and lethargic but fully oriented. His pulse was 108/minute, blood pressure was 115/80 mm of Hg., temperature was 101.8oF and respiratory rate was 20/minute. His voice was muffled. 2- Examination of the oral cavity and throat. His oral hygiene was poor with pooling of saliva in the mouth. There was a smooth bulge in the posterior pharyngeal wall. Tonsils, pillars, soft palate and other areas were normal. 3- Examination of the neck. There was no swelling in the neck. Cervical 149

Clinical scenarios in oto-rhino-laryngology

lymph nodes were not palpable. Neck movements were normal. 4- Indirect laryngoscopy. It was not done due to pain in the throat. Differential Diagnosis: 1- Retropharyngeal abscess 2- Parapharyngeal abscess 3- Quinsy 4- Acute tonsillitis/pharyngitis 5- Other causes of acute sore throat Investigations: 1- Plain X-ray neck (lateral view). It showed widening of the prevertebral space with pushing of the larynx and trachea anteriorly (fig. 46.1). There was no visible air fluid level. 2- Blood complete picture. It showed marked leucocytosis of 24,000/mm3 with 85% neutrophils.

Fig. 46.1 Plain X-ray neck (lateral view) showing marked widening of the prevertebral space

Diagnosis: This was a case of acute retropharyngeal abscess. Treatment: Patient was admitted for incision and drainage of the retropharyngeal abscess. Patient was very anxious and was not willing to underwent this procedure under local anaesthesia, so it was done under general anaesthesia. Endotracheal intubation was difficult with risk of spontaneous rupture of the abscess, tube was passed with caution. A vertical incision was given on the posterior pharyngeal wall, thick pus came out. Pus was sent for culture and sensitivity. Post-operatively broad spectrum parenteral antibiotic was given (amoxicillin with clavulanic acid). Culture and sensitivity report showed heavy growth of staphylococcus aureus, sensitive to the given antibiotic. Further recovery was uneventful and patient was discharged from the hospital after two days. 150

Clinical scenarios in oto-rhino-laryngology

Discussion: Retropharyngeal space is a potential space that lies behind the pharynx between the bucco-pharyngeal fascia covering pharyngeal muscles and the prevertebral fascia covering prevertebral muscles. It extends from base of the skull to the posterior mediastinum as far as the level of bifurcation of the trachea. The space is divided by a fibrous raphe in the midline into two lateral halves called ‘space of Gillette’. Each of this space contains retropharyngeal lymph node, which usually disappear by the age of four years. Retropharyngeal space communicates laterally with parapharyngeal space. Retropharyngeal abscess is the collection of pus in this retropharyngeal space and there are two distinct clinical types: 1- Acute retropharyngeal abscess 2- Chronic retropharyngeal abscess Acute retropharyngeal abscess is mostly caused by suppuration in the retropharyngeal lymph nodes. This variety is commonly seen in children below the age of 4 years. In adults acute retropharyngeal abscess may occur as a result of penetrating injury in the posterior pharyngeal wall or cervical oesophagus. Rarely infection may reach here from petrous apex abscess.

TEST YOURSELF

Read the clinical scenario given at the beginning and answers the following questions 12345-

What are the differential diagnosis in this case? How will you investigate this case? How will you manage this case? What is retropharyngeal space? and what are its contents? What are different clinical types of retropharyngeal abscess?

151

Clinical scenarios in oto-rhino-laryngology

Case 47 Clinical Scenario A 36 years old male patient came with the complaints of recurrent pain and swelling in the sub-mandibular region on right side for last many months. Swelling increased in size whenever he used to take meals especially sour substances. He also had pain in the swelling and floor of the mouth during meals, which remained there for one to two hours. Important points in history taking: 1- Detailed history regarding the swelling. According to him he did not know how the swelling started, but first time he noticed the swelling because of pain. Initially it was very small but progressively it increased. Swelling usually appeared during meal time and remained there for sometime. In between meal time it was barely visible. 2- Detailed history regrading pain. According to him he had pain in the sub-mandibular region and floor of the mouth during meals, which was localized, dull, mild to moderate in nature and remained for one to two hours after meals. Pain was associated with increase in size of the swelling. 3- Any history of fever. There was no history of fever. Important points in clinical examination: 1- Examination of the sub-mandibular region. No distinct swelling was present in this region at the time of examination but there was a generalized bulge of the sub-mandibular salivary gland. On palpation gland was palpable and appeared soft to firm in consistency. 2- Examination of the floor of mouth. By asking the patient to elevate the tongue, a diffuse bulge was seen in the floor of the mouth on the right side (fig. 47.1).

Fig. 47.1 A diffuse bulge in floor of the mouth on the right side

152

Clinical scenarios in oto-rhino-laryngology

3- Bimanual palpation of the floor of mouth and sub-mandibular region. In this patient a diffuse enlargement of the sub-mandibular salivary gland was present and a hard substance was palpable in the sub-mandibular duct in its posterior part, which was slightly mobile along the duct. Most probably it was a stone in the duct. 4- Rest of ENT examination. No positive finding on it. Differential Diagnosis: On history and clinical examination this appears to be a case of stone in the sub-mandibular duct causing partial obstruction. The following conditions should be considered as differential diagnosis: 1- Stricture or stenosis of the sub-mandibular duct. 2- A small tumour of the duct or its surrounding structures causing obstruction to the duct. 3- Enlarged sub-mandibular lymph node. 4- Neoplasia of the sub-mandibular salivary gland. Investigations: 1- Plain X-ray floor of the mouth (occlusal view). This showed a radiopaque shadow in the region of the sub-mandibular duct (fig 47.2). 2- Sialogram. It is not done in this case as radiopaque stone was visible on plain X-ray. If the stone was not visible on plain X-ray or some other pathology like stricture was suspected then sialogram is indicated. 3- Other baseline investigations for general anaesthesia including blood complete picture, ESR, random blood sugar, blood coagulation studies, urine D/R and X-ray chest (PA view). All were normal. Diagnosis: This was a case of stone in the sub-mandibular salivary gland duct (salivary calculus). Treatment: Patient was admitted and planned for either removal of the stone

Fig. 47.2 Plain X-ray floor of the mouth (occlusal view) showing a radiopaque shadow along the sub-mandibular duct

153

Clinical scenarios in oto-rhino-laryngology

Fig. 47.3 Specimen of the sub-mandibular salivary gland with the removed stone

intra-orally or removal of the gland with stone through external approach. The sub-mandibular duct stone is possible to be removed intra-orally by opening the duct if it is present in the duct anteriorly. More posteriorly lying stone or stone in the gland itself is not possible to be removed intra-orally. In such cases whole of the sub-mandibular salivary gland is removed along with major part of its duct. In this case, it was not possible to remove the stone intra-orally, so external approach was used to remove the whole gland and its duct with the stone (fig. 47.3). Discussion: Stone formation can occur in the duct or within the salivary gland as a result of calcium deposition on any epithelial debris, mucous or organic matrix. Stone formation is much common in sub-mandibular duct (90%) as compared to the parotid duct. There are several reasons for more incidence of stone formation in sub-mandibular duct. Sub-mandibular secretion is mainly mucous and thick as compared to serous and thin secretion of the parotid gland. In addition the calcium content of the sub-mandibular secretion is higher.

TEST YOURSELF

Read the clinical scenario given at the beginning and answers the following questions 123456-

What are the differential diagnosis in this case? How will you investigate this patient? How will you manage this case? What are the surgical options for removal of sub-mandibular duct stone? In which salivary gland, incidence of stone formation is maximum? Why the incidence of stone formation is more in sub-mandibular gland as compared to parotid gland? 154

Clinical scenarios in oto-rhino-laryngology

Case 48 Clinical Scenario A 44 years old male patient presented with a painless swelling near angle of mandible on the right side for the last many years. Initially the swelling was very small and barely visible but over a period of many years it increased gradually to its present size (fig 48.1).

Fig. 48.1 Patient with a swelling near angle of mandible on the right side

Important points in history taking: 1- Detailed history regarding swelling. According to the patient, swelling was present for last many years and he did not know how it started. Initially it was very small and he ignored it. Very slowly and gradually it started to increase in size till its present form (fig. 48.1). There was no aggravating or relieving factor known to him and there was no associated factor. 2- Any history of sudden increase in size. In this case there was no such history and swelling was increasing very slowly and gradually. 3- Any history of pain in the swelling. He occasionally felt some discomfort in the region of swelling and there was no history of pain. 4- Any history of facial nerve paralysis/paresis or facial asymmetry. There was no such history. 5- Any history of dry mouth and difficulty in chewing or swallowing. There was no such history in this case. Important points in clinical examination: 1- Examination of the swelling. A rounded, smooth, single swelling of 8 cms. diameter was present in the parotid region and angle of mandible on right side. The margins of the swelling were well defined. The skin over the swelling was normal and mobile. There was no pulsation, 155

Clinical scenarios in oto-rhino-laryngology

2345-

discharge, opening or sinus, reducibility or hyper-pigmentation over the swelling. Temperature over the swelling was equal to the surrounding with no tenderness. The swelling was firm in consistency with no fluctuation and transillumination was negative. The swelling was only slightly mobile and appeared to be arising from the parotid gland. Examination of the facial nerve. It was intact. Examination of the oral cavity and oropharynx. It was also within normal limits. Examination of the ear. It was within normal limits. Examination of the cervical lymph nodes. There was no palpable lymph node in the neck on any side.

Investigations: 1- Fine needle aspiration cytology. There was scanty aspirate which showed presence of mixed cells of ductal origin and myo-epithelial cells. No features of malignancy were seen in the cells. Probable diagnosis was pleomorphic adenoma. 2- MRI/CT scan. It was not done as the patient was non-affording. 3- Baseline investigations for general anaesthesia. All were within normal limits. Diagnosis: The diagnosis in this case was pleomorphic adenoma arising from the parotid gland. Treatment: The patient was admitted and planned for superficial parotidectomy under general anaesthesia. Parotidectomy incision (Blair’s incision) as shown in fig. 48.2 was given, which started near the upper part of the pinna, ran in the pre-auricular crease till the point where lobule was attached to the face. It turned backwards till tip of the mastoid process, from there it curved forward parallel to lower margin of the mandible in the neck crease about two fingers

Fig. 48.2 Surgical incision for parotidectomy

156

Clinical scenarios in oto-rhino-laryngology

below the mandible. Flap was raised and reflected anteriorly. Main trunk of the facial nerve was identified which was entering into the parotid gland. Whole of superficial lobe of the gland was removed and wound closed in layers after applying drain. Post-operative recovery of the patient was uneventful and he was followed up regularly with no recurrence. Discussion: Although the incidence of salivary gland tumour varies in different geographical regions, these tumours in general are uncommon. Approximately 80% of all salivary tumours are located in the parotid, 10% in the submandibular and rest of the 10% in sub-lingual and minor salivary glands. In the parotid gland most of the tumours (about 80%) are benign. Pleomorphic adenoma is the commonest among benign tumours of the salivary glands. It is characterized by slow growth and clinically a benign course. It is essentially an epithelial tumour of complex morphology, with both epithelial and myo-epithelial tissues. That is why it is called mixed tumour or pleomorphic. The epithelial and myo-epithelial elements are arranged in various patterns. It is surrounded by a false capsule, which is formed in response to expansion of tumour. This false capsule of compressed parotid tissue varies in thickness and tumour may extend into the capsule in a lobulated or pseudopod pattern. That is why removal of the tumour through its capsule may lead to recurrence thus it is treated by removal of whole superficial lobe. It is usually seen in third and fourth decade with a slight female preponderance. The most important complication of the parotid gland surgery is injury to the facial nerve. Facial nerve after emerging from the stylomastoid foramen, enters into the parotid gland and divides into its five terminal branches within the parotid gland. Thus facial nerve divides the parotid gland into two parts, superficial and deep lobes.

TEST YOURSELF

Read the clinical scenario given at the beginning and answers the following questions 123456-

What is the most likely diagnosis in this case? What are the important points in history taking in this patient? What are the important points in clinical examination of this patient? How will you manage this case? What is the relation of facial nerve with the parotid gland? Classify various tumours of the parotid gland.

157

Clinical scenarios in oto-rhino-laryngology

Case 49 Clinical Scenario A 41 years old female patient came in OPD with the complaint of swelling infront of her neck for last 4 years (fig. 49.1).

Fig. 49.1 Patient with swelling infront of the neck

Important points in history taking: 1- Detailed history about the swelling. Onset of the swelling was insidious and she did not know how it started. Then it started to increase in size very slowly and gradually till it reached to its present form. There was no known aggravating or relieving factor and it was not associated with any other complaint. 2- Any sudden increase in size of the swelling. There was no history of sudden increase in size. 3- Any history of pain in the swelling or neck. There was no such history. 4- Any symptom related with compression of either food or air passage. There was no history of dysphagia, dyspnoea, stridor etc. but she complained that now she could not sleep in supine position for a long time. 5- History of any symptom related with hyper or hypothyroidism like weight loss or weight gain, palpitation, tremors, change in skin or hairs, change in eyes, change in appetite etc. There was no such history. 6- Area where she lived. She was recently migrated to Karachi from northern area of the country (which is endemic for goiter). 7- History of similar problem in the other family members. Many of her close relatives and family members have similar problem. Important points in clinical examination: 1- General physical examination. She was a middle aged women of obese built, looking comfortable and well oriented. Her pulse rate was 78 per 158

Clinical scenarios in oto-rhino-laryngology

2-

345-

minute, blood pressure was 130/84 mm of Hg., respiratory rate was 20 per minute and temperature was 98.8oF. There was no pedal oedema, tremors, sweating or dryness in hands etc. Examination of the swelling. An irregular swelling was present in front and lateral aspect of the neck, measuring 15 cms. at its maximum. The margins of the swelling were ill defined and diffuse. The surface was irregular with nodularity. It moved on swallowing but not on protrusion of the tongue. The skin over the swelling was normal and mobile. It was soft to firm in consistency with no fluctuation and transillumination was negative. On auscultation there was no bruit. Movement of the vocal cords. On indirect laryngoscopy both vocal cords were fully mobile and normal. Examination of the cervical lymph nodes. There was palpable lymph node in the neck. Examination of the eyes for proptosis and movements. It was within normal limits.

Investigations: 1- Fine needle aspiration cytology of the swelling. It showed that aspirate contained mainly colloid fluid with few clusters of follicular cells admixed with foamy histiocytes and hemosiderin laden macrophages. The features were suggestive of benign nodular goiter. 2- Ultrasonography of the swelling. It showed grossly enlarged thyroid gland with multiple cysts of variable sizes in both lobes of the gland. 3- Thyroid function tests. T3, T4 and TSH were all within normal limits. 4- Serum calcium level. It was also within normal limit. 5- Baseline investigations for general anaesthesia. All were within normal limits. Diagnosis: This was a case of multinodular goiter (MNG). Treatment: Patient was planned for thyroidectomy under general anaesthesia. A cervical collar incision was given in a skin crease and flaps elevated. Straps muscles were separated in the midline and retracted. Thyroid gland was exposed and sub-total thyroidectomy was done (fig. 49.2 and 49.3) with preservation of both inferior parathyroid glands with intact blood supply. Drain was applied and wound closed in layers. Post-operative recovery was uneventful. Discussion: Multinodular goiter (MNG) is a commonly used term to describe an enlarged thyroid gland with multiple areas of nodularity. Worldwide MNG is the most common endocrine disorder affecting 500 to 600 million people, where iodine deficiency is often the causative factor. MNG is further classified 159

Clinical scenarios in oto-rhino-laryngology

Fig. 49.2 Multinodular goiter during surgery

Fig. 49.3 Multinodular goiter after sub-total thyroidectomy

into toxic and non-toxic variety and the treatment slightly varies in these two types. In typical non-toxic MNG surgery is done while in toxic MNG medication is given before surgery to make the patient euthyroid. Although the surgical technique is the same for both toxic and non-toxic MNG. In iodine deficient areas, resultant hypothyroidism contributes to the etiology of multinodular goiter. Through the feedback mechanism, hypothyroidism causes an increase in TSH, which stimulates growth of thyroid tissues. The other factors such as genetic influences also play a role in the development of multinodular goiter. It is well established that multinodular goiter may harbor occult malignancy, although the true incidence has been disputed. Patients with multinodular goiter often present with a neck swelling producing cosmetic deformity or sometimes diagnosed by a physician during routine examination. Large goiter may produce compressive symptoms like dysphagia, feeling of something in the throat or dyspnoea especially on lying supine. Controversy remains regarding the extent of thyroid gland removal so as to prevent recurrence in MNG and at the same time to prevent serious complications. At one hand is the sub-total or near total thyroidectomy, where 160

Clinical scenarios in oto-rhino-laryngology

the complications are less but the chances of recurrence are high and resurgery in such cases has more complications. On the other hand is the total thyroidectomy where there is virtually no chance of recurrence but the complications are more than partial surgery. The rate of recurrence in cases of MNG, treated by partial surgery is directly proportional to the volume of remnant thyroid tissues left behind and the amount of multinodularity. In cases of extensive, bilateral and generalized multinodularity, the recurrence rate is higher.

TEST YOURSELF

Read the clinical scenario given at the beginning and answers the following questions 123456-

What are the important points in history taking in this patient? What are the important points in clinical examination in this patient? What is the most likely clinical diagnosis after history and clinical examination? How will you investigate such a patient? How will you treat this patient? What is multinodular goiter (MNG)?

161

Clinical scenarios in oto-rhino-laryngology

Case 50 Clinical Scenario A 14 years old boy came with the complaints of painless swelling on the right side of his neck for last three months which was increasing progressively (fig. 50.1). He was otherwise healthy and had no other symptoms referable to the head and neck area, except he complained of low grade fever and sweating especially in the evening for the same period.

Fig. 50.1 Patient with swelling in posterior triangle of the neck on right side

Important points in history taking: 1- Detailed history regarding the swelling. According to the patient it started insidiously and initially it was very small and barely visible but then it started to increase in size progressively day by day. It was painless but sometimes he felt discomfort in the neck. There was no aggravating or relieving factors and no known associated symptom. 2- Detailed history regarding fever. According to the patient he had frequent low grade fever often associated with chills and sweating especially in the evening and night. 3- Any history of nasal obstruction, epistaxis or other complaint related with the nose. There was no such history. 4- Any complaint related with respiratory or food passage. There was no complaint. 5- Occupation and socio-economic condition. He was a worker with a car mechanic and belonged to a low socio-economic class. He lived in a one room house with eight family members. 6- Any family history of similar problem, tuberculosis or any respiratory disease. According to him his elder brother had tuberculosis of the lungs. 7- Any history of weight loss or anorexia. There was no such history. 162

Clinical scenarios in oto-rhino-laryngology

8- Any history of smoking, pan eating or alcohol intake. There was no such history. Important points in clinical examination: 1- General physical examination. Patient was a young boy of lean built and average height, fully oriented. His vital signs were within normal limits. 1- Examination of the swelling. There was an irregular swelling present in the posterior triangle of the neck around 6x8 cms. The margins were illdefined and irregular, surface was nodular, overlying skin was normal looking with no signs of inflammation. Swelling was not pulsatile and not reducible. On palpation it was firm in consistency, nodular, not fluctuant, slightly mobile over its bed, skin over the swelling was mobile. The temperature of the swelling was same as of surrounding and it was not tender to touch. Transillumination was negative. Clinical impression being multiple lymph nodes matted together. 2- Examination of the nose and nasopharynx. It was within normal limits. 3- Examination of the throat and indirect laryngoscopy. It was within normal limits. 4- Examination of the ears. There was no positive finding on it. Differential Diagnosis: 1- Tuberculous lymphadenopathy 2- Lymphoma/leukaemia 3- Nodal metastasis from a primary in head & neck region 4- Other causes of lymphadenopathy 5- Benign or malignant tumour of mesenchymal origin Investigations: 1- Blood complete picture and ESR. It showed increase in lymphocyte count of 55% and raised ESR of 80 mm in first hour. 2- Plain X-ray chest (PA view). It was within normal limits. 3- Ultrasonography of the neck. It showed multiple solid swellings with central necrosis in few of them. Impression was multiple lymph nodes with central necrosis. Overall size was 7x8 cms. 4- Fine needle aspiration cytology (FNAC). It showed presence of granuloma with multiple giant cell and casseous necrosis, consistent with tuberculosis. 5- Sputum for acid fast bacilli (AFB). It was negative for AFB. 6- Mountoux test (MT). It was strongly positive of 14 mm. Diagnosis: The diagnosis of this case was tuberculous cervical lymphadenopathy. Treatment: The patient was planned for antituberculous therapy. Initially four drug regimen was given for two months including rifampicin, ethambutol, pyrazinamide and isoniazid according to the body weight. After two months 163

Clinical scenarios in oto-rhino-laryngology

three drugs (rifampicin, ethambutol and isoniazid) were given for 6 months. Patients recovery was uneventful with disappearance of all cervical lymph nodes. Discussion: Tuberculosis is primarily a medical disease. If the diagnosis is established in a timely and accurate manner, immediate institution of antituberculous therapy produces dramatic resolution. In the past, surgery has played an important role in both diagnosis as well as treatment of cervical tuberculous lymphadenopathy. The only effective way to make definite diagnosis in such cases is open biopsy. Increasing use of FNAC for evaluating neck mass has revolutionized ability to make early diagnosis, institute appropriate treatment and in many cases to avoid open biopsy. Only 10 to 15% of the patients who present with cervical tuberculous lymphadenopathy, will have chest X-ray findings consistent with pulmonary tuberculosis. However despite normal chest X-ray, some of these patients will have a positive sputum culture for acid fast bacilli (AFB). Therefore even if the radiograph is negative, all patients with cervical tuberculous lymphadenopathy should have obtained sputum culture for AFB.

TEST YOURSELF

Read the clinical scenario given at the beginning and answers the following questions 1234-

What is the most likely diagnosis in this case? How will you investigate this patient? How will you manage this case? What is the role of open biopsy in such cases?

164

Clinical scenarios in oto-rhino-laryngology

INDEX

A

nose, 71, 84 Branchial cyst, 146 Bronchial foreign body, 138 Bronchoscopy, 139

Abscess, brain abscess, 27 extra dural abscess, 27 parapharyngeal, 105, 145, 150 peri-tonsillar, 104 retropharyngeal, 105, 145, 149 septal abscess, 71 sub dural abscess, 27 Acoustic neuroma, 52 Acute, otitis media, 18 retropharyngeal abscess, 149 suppurative otitis media, 18 tonsillitis, 150 Adenoid cystic carcinoma, 111 Adenoidectomy, 108 Adenoids, 107 Adenoid facies, 107 Adenoma, pleomorphic, 156 Allergic fungal sinusitis, 73 Allergic polyp, 73 Allergic rhinitis, 78 Angiofibroma, nasopharyngeal, 92 Angiography, 94 Antro-choanal polyp, 66, 75 Antrostomy, 91 Aspergillus, in otomycosis, 14 Attico antral, type of CSOM, 24, 29 Aural polyp, 21, 41

C

Caloric test, 49, 52 Canal wall down procedures, 26, 29 Canal wall up procedures, 23 Candida albicans, 15 Carcinoma, cheek, 113 larynx, 133 tongue, 117 verrucous, 117 Carhart’s notch, 39 Catarrhal otitis media, 36 Cerebral hypoxia, 130 Cerumen, 9 Cervical lymphadenopathy, 162 Cholesteatoma, 24, 29 Chronic, laryngitis, 128 mastoiditis, 26 retropharyngeal abscess, 149 sinusitis, 89 suppurative otitis media, 23, 24, 29 tonsillitis, 99 Cochlear implant, 55 Cortical mastoidectomy, 23 Cough, 138 CSOM, attico-antral, 24, 29 tubo-tympanic, 23 CT scan, larynx, 135 nose & PNS, 67, 68, 74, 90, 93, 94 Cyst, branchial, 146

B

Benign positional vertigo, 51 BERA, 54 Bimanual palpation, 153 Bleeding, from the ear, 16 from the mouth, 102 from the nose, 62, 86, 92 Blockage, of the ear, 9 Boil, ear, 3

D

Deafness, 3,9,21,24,34,38,42,45,48,54 Deviated nasal septum, 53 Direct laryngoscopy, 121, 134 165

Clinical scenarios in oto-rhino-laryngology

tongue, 117

Discharge, blood stained, 81 from the ear, 3,14,16,21,24,28,31 from the nose, 59, 66, 73, 81 from the sinus, 12 Discomfort, in the ear, 9 in the throat, 149 Dry perforation, of ear drum, 31 Dysphagia, 120, 149 Dyspnoea, 138

H

Haemangioma, 124 Haemorrhage, post-tonsillectomy, 102 Haematoma, septal, 60 Headache, 66, 89 Hearing aid, 44 Hearing impairment, 3, 9, 21, 24, 34, 38, 42, 45, 48, 54 Hoarseness, 127, 133 Hypertrophied inferior turbinate, 68 Hypopharynx, foreign body, 120 Hyposmia, 92

E

Earache, 3, 6, 9, 12, 16, 18, 107 Endolymphatic hydrops, 50 Epistaxis, 86 Epley’s maneuver, 53 Ethmoidal polyp, 73 Ethmoidectomy, 91 Extra cranial complications, of CSOM, 26

I

Intracranial complications of CSOM, 27 Inverted papilloma, 68, 93 Itching, in the ear, 14

J

F

Juvenile, angiofibroma, 92

Facial asymmetry, 28 Facial nerve, 28 Fever, 18, 99, 104, 138, 162 Foreign body, bronchial, 138 ear, 6 nose, 81 hypopharynx, 120 Fracture nasal bone, 62 Functional endoscopic sinus surgery, 68,75 Fungal sinusitis, 93 Furuncle, ear, 3 nose, 71, 84

K

Kiessel Bach’s plexus, 87

L

Labyrinthitis, 49, 52 Laryngeal, carcinoma, 133 oedema, 145 tuberculosis, 128 Laryngectomy, 135 Laryngoscopy, 121, 134 Little’s area, 87 Ludwig’s angina, 143 Lymphadenopathy, 162 Lymphoma, 163

G

Gastro-oesophageal reflux, 99 Glue ear, 36 Goiter, multinodular, 158 Grommet, 36 Growth, cheek, 110, 113 floor of the mouth, 123

M

Maggots, in ear, 16 Maggot oil, 17 Malignant tumours, of larynx, 133 oral cavity, 113, 117 166

Clinical scenarios in oto-rhino-laryngology

neoplasia of, 110, 113 Orthopentomogram, 114 Osteoma. external auditory canal, 4 Osteomeatal complex, 90 Otitis externa, 10, 15 Otitis media, acute suppurative, 18 catarrhal, 36 chronic suppurative, 23, 24, 29 exudative, 36 mucinous, 36 non-suppurative, 36 secretory, 36 serous, 36 with effusion, 36 Otomycosis, 10, 14 Otosclerosis, 38

Marsupialization, 124 Mass, in the external auditory canal, 21 in the nose, 66, 73, 92 Mastoidectomy, 23 Maxillary sinusitis, 89 Mediastinitis, 145 Meniere’s disease, 48 Meningitis, 27 Microlaryngoscopy, 128 Modified radical mastoidectomy, 26, 29 Mouth, breathing, 107 neoplasia of, 113, 117 Multinodular goiter, 158 Myringoplasty, 32 Myringotomy, 36

P

N

Pain, during swallowing, 99, 104 ear, 3, 6, 9, 12, 16, 18, 107 facial, 66, 89 nose, 84 sub-mandibular region, 152 throat, 104, 120, 149 Papilloma, of cheek, 110 Paracusis Willisi, 38 Parapharyngeal abscess, 105, 145, 150 Parotid gland, 156 Parotidectomy, 156 Percutaneous travheostomy, 131 Perforation, tympanic membrane, 31 Peri-tonsillar abscess, see also quinsy, 104 Pleomorphic adenoma, 156 Plunging ranula, 124 Polyp, antro-choanal, 66, 75 aural, 4, 21 ethmoidal, 73 Polypectomy, nasal, 68 Post-nasal dripping, 59, 89 Post-tonsillectomy haemorrhage, 102 Pre-auricular sinus, 12 Presbyacusis, 42

Nasal allergy, 78 Nasal bone fracture, 62 Nasal deformity, 62 Nasal obstruction, 59, 62, 66, 70, 73, 76, 78, 92, 107 Nasal packing, 86 Nasal polyp, 66, 73 Nasopharyngeal, angiofibroma, 92 tonsils, 109 Nasopharynx, neoplasia of, 92 Neck dissection, 115 Neoplasia, of cheek, 110, 113 larynx, 133 oral cavity, 110, 113, 117 salivary gland, 156 tongue, 117 Noise induced deafness, 45 Non suppurative otitis media, 36

O

Odynophagia, 99, 104, 123 Oesophagoscopy, 122 Oral cavity, 167

Clinical scenarios in oto-rhino-laryngology

chin, 143 external auditory canal, 3 floor of the mouth, 123, 147 neck, 146, 158, 162 nose, 70, 84 parotid region, 155 sub-mandibular region, 147 tongue, 117 Syringing, ear, 38

Q

Quinsy, 104

R

Radical mastoidectomy, 29 Ranula, 123 Retropharyngeal abscess, 105, 145, 149 Rhinorrhoea, 56, 66, 73, 78 Rhino-sinusitis, 89 Ringing, in the ear, 34, 45, 48 Rose’s position, 100

T

Tinnitus, 34, 45, 48 Tongue, carcinoma, 117 Tonsillectomy, 100 Tonsillitis, chronic, 99 Total laryngectomy, 135 Tracheostomy, 130 Tracheostomy tube, 131 Traumatic perforation, of ear drum, 10,33 Trismus, 104, 113 Tuberculosis, of cervical lymph nodes, 162 Tubo tympanic, CSOM, 44 Tympanogram, 25 Tympanoplasty, 48

S

Salivary glands, calculus, 153 neoplasia of, 155 Secretory otitis media, 36 Semont’s maneuver, 53 Senile deafness, 42 Septal, adhesions, 76 deviation, 53 haematoma, 70 surgery, 60 Septoplasty, 60 Septo-rhinoplasty, 64 Serous otitis media, 36 Sialoadenitis, 124 Sialogram, 153 Simple ranula, 124 Singer’s node, 128 Sinusitis, 89 Sneezing, 73, 78 Snoring, 107 Squamous cell carcinoma, 111,114,118,134 Squamous cell papilloma, 111 Stapedotomy, 40 Stridor, 133 Sub mucous diathermy, 80 Suction cleaning, of ear, 10 Superficial parotidectomy, 156 Supra-omohyoid neck dissection, 114, Sweating, 162 Swelling, cheek, 110

V

Vertigo, 48, 51 Vertigo, benign paroxysmal positional, 51 Verrucous carcinoma, 117 Vestibular neuronitis, 49, 52 Vocal nodule, 127 Voice change, 127, 133 Voice test, 19

W

Walsham’s forceps, 64 Wax, in ear, 38 Weber’s test, 19

X

X-ray, mastoid, 22, 26 nasal bone, 63 PNS, 60, 67, 79 168

View more...

Comments

Copyright ©2017 KUPDF Inc.
SUPPORT KUPDF